You are on page 1of 338

First

published in 2015 by Gloucester Publishers Limited, Northburgh House,


10 Northburgh Street, London EC1V 0AT

Copyright © 2015 Lorin D’Costa

The right of Lorin D’Costa to be identified as the author of this work has been
asserted in accordance with the Copyrights, Designs and Patents Act 1988.

All rights reserved. No part of this publication may be reproduced, stored in a


retrieval system or transmitted in any form or by any means, electronic,
electrostatic, magnetic tape, photocopying, recording or otherwise,
without prior permission of the publisher.

British Library Cataloguing-in-Publication Data


A catalogue record for this book is available from the British Library.

ISBN Kindle: 978-1-78194-292-5


ISBN epub: 978-1-78194-293-2

Distributed in North America by National Book Network,


15200 NBN Way, Blue Ridge Summit, PA 17214. Ph: 717.794.3800.

Distributed in Europe by Central Books Ltd.,


99 Wallis Road, London E9 5LN. Ph 44(0)845 458 9911.

All other sales enquiries should be directed to Everyman Chess,


Northburgh House, 10 Northburgh Street, London EC1V 0AT
email: info@everymanchess.com; website: www.everymanchess.com

Everyman is the registered trade mark of Random House Inc. and is used in this
work under licence from Random House Inc.

Everyman Chess Series


Chief advisor: Byron Jacobs
Commissioning editor: John Emms
Assistant editor: Richard Palliser

Typeset and edited by First Rank Publishing, Brighton.


Cover design by Horatio Monteverde.
Printed by TJ International Limited, Padstow, Cornwall.
About the Author
Lorin D’Costa is an International Master and a full-time chess teacher. He coaches
some of the top chess schools in the UK, some of England’s very strongest juniors,
and also the England team at World and European Youth Championships.

Also by the Author:


Who Dares Wins!
The Sicilian Scheveningen: Move by Move
The Panov-Botvinnik Attack: Move by Move
Contents
About the Author
Series Foreword

Introduction
1 World Champions and the Queen’s Indian
2 Fianchetto Variation: 4 g3 Ba6 5 Qc2 and Minor Fifth Moves
3 Fianchetto Main Line: 4 g3 Ba6 5 b3 Bb7 6 Bg2 Bb4+
4 Fianchetto Variation: 4 g3 Ba6 5 b3 b5
5 Fianchetto Variation: 4 g3 Bb7 5 Bg2 g6
6 Petrosian Variation: 4 a3 Ba6
7 Petrosian Variation: 4 Nc3 Bb7 5 a3
8 4 Nc3 Bb7: Other Fifth Moves
9 4 e3 and Other Fourth Moves
10 King’s Indian Attack and Réti Lines

Index of Games
Series Foreword
Move by Move is a series of opening books which uses a question-and-answer
format. One of our main aims of the series is to replicate – as much as possible –
lessons between chess teachers and students.
All the way through, readers will be challenged to answer searching questions, to
test their skills in chess openings and indeed in other key aspects of the game. It’s our
firm belief that practising your skills like this is an excellent way to study chess
openings, and to study chess in general.
Many thanks go to all those who have been kind enough to offer inspiration, advice and assistance in the creation of
Move by Move. We’re really excited by this series and hope that readers will share our enthusiasm.

John Emms,
Everyman Chess
Introduction
The Queen’s Indian Defence is characterised by the moves:
1 d4 Nf6 2 c4 e6 3 Nf3 b6

It is important to note that the Queen’s Indian is a sister opening to the Nimzo-Indian
Defence which occurs after 3 Nc3 Bb4, and for details on the Nimzo I refer you to the
excellent book on the subject by GM John Emms in this Move By Move series.
One of the key strategic ideas in Queen’s Indian (and the Nimzo-Indian) is the battle for the crucial e4- and d5-
squares. Much of the early play revolves around these squares and this theme will recur throughout the book. So why
does White play 3 Nf3, rather than 3 Nc3? With 3 Nf3, White plays an important developing move, avoids the 3 ...
Bb4 pin and retains the flexibility of playing Nb1-c3 later. Meanwhile, Black’s move 3 ... b6 prepares the development
of the c8-bishop, either as a fianchetto on the b7-square or sometimes, to the a6-square in order to attack the c4-
pawn. While Black has other 3rd move alternatives, such as 3 ... Bb4+ (the Bogo-Indian Defence), I personally prefer
the Queen’s Indian since pieces are kept on the board and Black, in my view, has greater winning chances. For us
lower down the chess pyramid, we should be trying to play for a win with both colours!
As an introduction to the Queen’s Indian Defence, in Chapter One I have analysed
five games by World Champions playing the opening from the Black side.
The main line over the years has been 4 g3. White intends to fianchetto his light-
squared bishop, which from the g2-square fights for the e4- and d5-squares and goes
head-to-head with its counterpart should Black’s c8-bishop land on b7. I have decided,
after much deliberation, to go with 4 ... Ba6 as my main line recommendation against
the Fianchetto Variation, but am demonstrating two relatively unexplored and much
underrated lines against 4 g3. Chapters Two, Three and Four see our discussion on that
topic. Additionally, I felt that the reader should know something about 4 ... Bb7, so
Chapter Five presents a sideline that you can surprise your well prepared opponent
with.
The Petrosian Variation is introduced by 4 a3 or 4 Nc3 Bb7 5 a3. By playing an
early a2-a3, White prevents ... Bb4, thus keeping the c3-knight on the board to
maximise influence over those important e4- and d5-squares, and aiming to build a big
pawn centre with e2-e4. It was no less a player than the 13th World Champion, Garry
Kasparov, who championed this line from the 1980s onwards. We will be examining
this variation in detail in Chapters Six and Seven of this book.
Chapters Eight and Nine cover a number of less popular but respectable systems that
White can adopt to counter the Queen’s Indian. Finally, Chapter Ten shows how a
Queen’s Indian player can respond to Flank Openings such as the King’s Indian Attack
and Réti.
As a professional coach, I am always trying to extol the virtues of really
understanding what you are doing and the plans available, rather than just learning rote
moves. The Queen’s Indian has many plans available, to both sides, and by and large
these tend to be of a positional nature. This somewhat disappointed a friend of mine,
I’ll call him ‘Dave’ (rated around the 2000 mark) when I told him this, since he wants
to attack at all costs. However, to improve as a player I think it is important to
understand how to play positionally as well as aggressively. In addition, I am not for
one moment stating that you cannot attack in the Queen’s Indian, with either colour – a
number of the games in this book involve both attacking and sacrificial play. It is no
coincidence that some of the world’s most aggressive attacking players have added the
Queen’s Indian to their opening repertoires.
By holding this book you have already taken the first step towards adding the Queen’s Indian to your armoury, and I
hope that by reading the notes and playing through the games you will be able to play this opening confidently in your
own games.

Lorin D’Costa,
London, October 2015
Chapter One
World Champions and the Queen’s
Indian
In this chapter we will be analysing some Queen’s Indian games played by world
champions. Do take a look at these carefully – they already contain plenty of
opening theory and middlegame strategy that will be an indicator of how this
opening is to be handled.

Game 1
B.Van Trotsenburg-M.Euwe
Amsterdam 1920

1 d4 Nf6 2 Nf3 e6 3 c4 b6 4 Nc3 Bb7 5 e3


5 a3 is the subject of Chapter Seven. White commits the sin in this game of simply
playing ‘moves’ rather than playing with a purpose. This allows the future World
Champion Max Euwe time to build up an attack.


Question: Is 5 ... c5 a good move here?

Answer: Black’s ... c5 is a typical pawn thrust to challenge the centre, but here it is not so good. 5 ... c5 6 d5! is
the sort of position Black is trying to avoid. The b7-bishop is out of the game, and White is going to build up a central
space advantage with e3-e4, Bf1-d3 and so on.
5 ... d5!
Even back nearly 100 years ago, Euwe knew the attacking setup with ... d5 and ...
Bd6 that I am recommending.
6 cxd5
After 6 Bd3 Black can proceed with:
a) 6 ... Bd6 7 0-0 0-0 8 b3 Nbd7 9 Bb2 a6, see Chapter Nine for more details.
b) 6 ... dxc4 can transpose to a favourable Queen’s Gambit Accepted setup, for
example 7 Bxc4 a6 8 0-0 b5 9 Be2 Nbd7 10 a3 c5 11 dxc5 Bxc5 which was already
very comfortable for Black in D.Cori Tello-G.Sargissian, Villafranca 2010.
6 ... exd5 7 Bd3 Bd6
This is a much more active square for the bishop than e7.
8 0-0 0-0 9 Qc2 Re8
9 ... a6 prevents 10 Nb5 but allows the central break 10 e4 dxe4 11 Nxe4 Bxe4 12
Bxe4 Nxe4 13 Qxe4 Nd7 14 Bg5 Qe8 which is roughly equal.
10 Bd2
This bishop performs no useful role here. 10 b3 and Bc1-b2 was better.
10 ... Nbd7 11 Rae1 Ne4

12 Bc1 Ndf6
12 ... a6 preventing Nc3-b5 and then getting on with the kingside plan was
preferable.
13 Nb5 Bb4 14 Rd1 c6 15 Nc3 Bd6
Resuming normal operations.
16 h3 Bc7 17 Ne2 Rc8 18 b3 Bb8 19 Bb2


Exercise: What should Black do next?

Answer: There are many possible moves, but I like Euwe’s strategy here
19 ... c5!
A pawn break to release the pieces, namely the c8-rook and the b7-bishop.
20 dxc5 Rxc5
Euwe plays for an isolated queen’s pawn (IQP) as he senses he has greater piece
play and the chance to attack.
20 ... bxc5 controlling d4 and giving rise to hanging pawns, was certainly an option.
21 Qb1 Qc7 22 b4
This looks good, but is met by a clever riposte, 22 Ned4 was more solid.
22 ... Ng5! 23 Nf4
Others:
a) 23 bxc5 Nxf3+ 24 gxf3 Qh2# was the main tactical idea behind 22 ... Ng5.
b) 23 Ned4 Rc4! 24 Bxc4 dxc4 25 Rfe1 Nxf3+ 26 Nxf3 Bxf3 27 gxf3 Qh2+ 28 Kf1
Nh5 and now the threat is ... Nf4 and ... Qh1 mate. Some great attacking ideas there.
23 ... Nxf3+ 24 gxf3 Rc6 25 Bb5
It looks as though Black has just blundered, but Euwe had seen further.
25 ... Nh5! 26 Qf5
Or 26 Bxc6 Nxf4 27 exf4 Qxf4 with mate very soon.
26 ... Nxf4 27 Qxf4 Rg6+ 28 Kh1
28 ... d4!
This is a natural yet very strong move, which blows open the long diagonal leading
to White’s king on h1. It is fitting that the Queen’s Indian bishop on b7 delivers the
denouement.
29 Bc1
Or 29 Qxc7 Bxf3+ 30 Kh2 Bxc7#.
29 ... Bxf3+! 0-1
Mate will follow soon.

Game 2
B.Gelfand-G.Kasparov
Novgorod 1997

1 d4 Nf6 2 c4 e6 3 Nf3 b6 4 a3
Interesting that Gelfand plays the Petrosian system against none other than its main
practitioner Kasparov!
4 ... c5!?
4 ... Ba6 5 Qc2 Bb7 6 Nc3 c5 7 e4 cxd4 8 Nxd4 is the line I am advocating in
Chapter Six.
5 d5 Ba6 6 Qc2 exd5 7 cxd5

7 ... g6
Or 7 ... Bb7?! 8 e4 Qe7 9 Bd3 Nxd5 10 0-0 wins a pawn but at a cost – lack of
development, and uncoordinated pieces. Do not play like this, even in a must win
situation!
Not 7 ... Nxd5?? 8 Qe4+ losing the d5-knight or the a8-rook.
8 Nc3 Bg7 9 g3
9 e4 Bxf1 10 Kxf1 d6 11 g3 0-0 12 Kg2 Nbd7 is also possible.
9 ... 0-0 10 Bg2 d6 11 0-0 Re8 12 Re1 Nbd7

Now we have reached a Benoni type structure as Black, in a Queen’s Indian!


Sometimes in the Benoni, Black does play ... b6 and ... Ba6, in order to play for the ...
b5 queenside advance. It is not clear if a2-a3 and Qd1-c2 have helped White too much.
13 h3
13 e4 Ng4 with the plan of ... Ne5 and a future ... Nd3 would be an active plan. This
is why White’s e-pawn tends to remain on e2 until the right moment.
13 Bf4 is more direct, attacking Black’s d6-pawn straight away. 13 ... Qe7 14 Qa4
Bb7 15 Nb5 and now:
15 ... Ne5 16 Nxe5 dxe5 17 d6 and White’s d6-pawn is dangerous, after 17 ... Qd7
18 Bxb7 Qxb7 19 Bg5 Qc6 20 Qa6 as played in K.Miton-L.Nisipeanu, Skanderborg
2005, looked quite nice for White.
15 ... Ne4!? 16 Nc7 Bxb2 17 Rab1 Bc3 18 Rec1 Bg7 19 Nxe8 Qxe8 and Black was
fine in E.L’Ami-J.Timman, London 2007.
13 ... b5 14 e4 Qc8!


Question: What is the point of 14 ... Qc8?

Answer: To cover the a6-bishop and prepare the ... b4 pawn advance. Since White’s queen on c2 is unprotected,
White is dissuaded from playing axb4, as then ... cxb4 will win material due to the pin down the c-file.
15 Bf4
15 Bf1!? b4 16 Nb5 Qb8 is also unclear, but probably better for White than the game
continuation.
15 ... b4 16 Na4 b3!
Nice active play from Kasparov, who realises the moment to strike is now before
White could consolidate his centre with a move like Ra1-d1.
17 Qxb3
17 Qb1 blocks in the a1-rook but does keep hold of the important central e4-pawn –
a difficult choice for White to make.
17 ... Nxe4
Black has exchanged a side pawn for an important central pawn, clearly to his
advantage.
18 Qc2
18 Nc3? Rb8 19 Qc2 Rxb2! 20 Qxb2 Bxc3 is a neat trick to remember.
18 ... Ndf6 19 g4 Qd7 20 g5
20 Ng5 to remove the annoying knight on e4 might be the last chance to equalise for
White.
20 ... Nh5


Question: Doesn’t this just lose the e4-knight?

Answer: Due to tactics, Black is fine.
21 Bh2
After 21 Rxe4 Rxe4 22 Qxe4 (22 Nxc5 is answered by Rc4 23 Qxc4 Bxc4 24 Nxd7
Nxf4) 22 ... Re8 the f4-bishop cannot be protected so Black regains the piece. Possible
continuations are then:
a) 23 Nxc5 Rxe4 24 Nxd7 Rxf4 and White’s position is very uncoordinated.
b) 23 Ne5 Bxe5 24 Bxe5 Rxe5 25 Qh4 Qf5 gives Black a very nice position. The
h5-knight will come to the excellent square f4, White’s kingside is weak, and the white
pieces in general are all worse than their black counterparts.
21 ... f5! 22 Nc3
22 gxf6 Nhxf6 defends e4.
22 ... Rab8 23 Rab1 Bxc3!
23 ... Rxb2! was likely even stronger, with a ‘computer line’ being 24 Rxb2 Bxc3 25
Rxe4 fxe4 26 Qxc3 exf3 27 Bxf3 Qxh3 28 Bxd6 Be2! and the pin on the f3-bishop will
cost material.
24 bxc3 Rxb1 25 Rxb1 Bc4 26 Nd2 Nxd2 27 Qxd2 f4!
Kasparov shows the positional side to his game. He hems in the h2-bishop and
homes in on White’s pawn weaknesses on d5 and g5.
28 Re1 Re5!? 29 Re4
29 Rxe5 dxe5 30 d6 Kf8! 31 Bf3 Ng7 32 Bg4 Nf5 33 Bxf5 gxf5 is still bad for
White. Look at that bishop on h2!
29 ... Rxe4 30 Bxe4 Qxh3 31 Bg2 Qg4 32 Qe1


Exercise: Can Black win here with 32 ... f3?

Answer: No, as White is threatening a perpetual check after 32 ... f3 33 Qe8+ Kg7 34 Qe7+ Kg8 35 Qe8+ with a
draw. Instead, Black played:
32 ... Ng7! 33 f3 Qxg5
A second pawn goes, and with it White’s position.
34 Qb1 Nf5 35 Qb8+ Kg7 36 Qxa7+ Kh6 37 Qf7 Bf1!
There was still time to go wrong in this sharp position, but Kasparov is clearly up to
the task. 37 ... Qh4 38 Qe8 Bxd5 is also strong and 37 ... Ne3?? 38 Bxf4 even loses!
38 Kxf1
38 Bxf4 Qxf4 39 Qf8+ Kh5 40 Bxf1 Qg3+ 41 Kh1 Ne3 everyone knows the queen
and knight are a fearsome attacking combination.
38 ... Ne3+ 39 Ke1 Qh4+! 40 Ke2 Qxh2 41 Kd3 Nf5! 0-1
Accurate to the end. Kasparov prevents any perpetual check and snares the bishop on g2.
Game 3
R.Ponomariov-V.Kramnik
Sofia 2005

1 d4 Nf6 2 c4 e6 3 Nf3 b6 4 Nc3 Bb7 5 Bg5 h6 6 Bh4

6 ... g5
6 ... Be7 7 Qc2 c5 is my recommendation in Chapter Eight (Games 53-54).
Kramnik’s idea is very interesting and unbalancing, which can of course backfire.
6 ... Bb4 7 e3 g5 8 Bg3 Ne4 9 Qc2 would transpose to another line in Chapter Eight
(Game 58), where I touch on some Nimzo-Indian style play.
7 Bg3 Nh5

Question: Why is Black going after this dark-squared bishop?

Answer: So he can play ... Bg7 and have an unopposed fianchettoed bishop on g7. White will always have to
keep an eye on this bishop as it targets the d4- and b2-squares.
7 ... Bb4 8 Qc2 Ne4 transposing to the Nimzo-Indian is also possible. Such
transpositions occur fairly frequently, but this is not something we should be afraid of
as this means the positions that arise tend to be similar.
8 e3 Nxg3 9 hxg3 Bg7 10 Qc2 Nc6 11 0-0-0 Qe7 12 g4


Question: What is the purpose of the move 12 g4?

Answer: The idea is to keep the h6-pawn tied down and therefore a future target for White. This was a drawback
of Black allowing the h-file to be opened.
12 ... Nb4
12 ... 0-0 looks really risky, although Black never finds a safe haven for his king in
the game in any case.
12 ... 0-0-0 13 a3 Kb8 14 Bd3 was probably the best way to safeguard the king,
away from the semi open h-file.
13 Qd2 c5
This is the typical way to attack White’s centre, and it is consistent with the strategy
of attacking d4 to open up the position for the g7-bishop.
After 13 ... 0-0-0 14 a3 Nc6 15 Bd3 Black is a bit short of space, but this was
certainly the safer option.
14 d5 exd5 15 a3 dxc4?!
Black offers a very risky but interesting piece sacrifice.
15 ... Bxc3 16 bxc3 Nc6 17 cxd5 Ne5 18 Nxe5 Qxe5 19 f4 Qf6 20 g3 was safer, but
Kramnik must have been trying to win at all costs!

16 axb4! cxb4 17 Nb5


17 Nd5 c3 18 bxc3 bxc3 19 Qa2 Bxd5 20 Rxd5 is messy but the extra piece should
probably be preferred, as it is not clear how the lack of pawn cover in front of the
white king can be exploited.
17 ... d5 18 Nbd4 Bf6
After 18 ... Qc5 19 Nf5 Bf6 20 Kb1 Bc6 21 N3d4 Bd7 22 Qe2 c3 23 e4 the position
is still murky, but the Black monarch is not that safe either and he is a piece down.
19 e4!
Counterattacking and putting the question to Black as to where he will run with his
king.
19 ... c3
19 ... dxe4 20 Nf5 c3 21 bxc3 Qc7 might have been the best chance for a
counterattack.
20 bxc3 dxe4
20 ... bxc3 21 Qa2 Bxd4 22 Nxd4 Qxe4 23 Qa3 Qe7 24 Bb5+ and after the queen
exchange on e7, Black’s pawns will drop like flies.
21 Nf5 Qc7
21 ... Bxc3 22 Qa2! Qc7 23 Bb5+ Kf8 24 Rd7 looks crushing.
22 Bb5+
22 Nd6+! Kf8 (or 22 ... Ke7 23 Nd4 bxc3 24 Qa2 Kf8 25 Bc4 –– what a position!)
23 Nxb7 exf3 24 Qd6+ Qxd6 25 Nxd6 Bxc3 26 gxf3 a5 27 Bc4 and the extra piece is
more important than the pawns here.
22 ... Kf8 23 N3d4 Rc8?
23 ... Rd8 or 23 ... a6 would have put up better resistance.
24 Ne2 e3 25 Qxe3

25 ... Bxg2
25 ... Bxc3 26 Nxc3 Bxg2 27 Kb1 bxc3 28 Rd7 Qc5 29 Qxc5+ bxc5 30 Re1 Rb8 31
Kc2! the bishop is immune due to 32 Rd8 mate.
26 Rhg1
26 Rd7! was also strong, 26 ... Qc5 27 Rxh6 Rxh6 28 Nxh6 Be7 29 Nf5 Qxe3+ 30
fxe3 when with the queens off, the white king will sleep easier.
26 ... Bc6 27 Bxc6 Qxc6 28 Rd6 Qa4 29 Rxf6 Qa1+
29 ... Re8 30 Qf3 Qa1+ 31 Kc2 Qa2+ 32 Kd3 Qa6+ 33 Kd4 and amazingly the king
is safe here! After 33 ... Rxe2 34 Qa8+ Re8 35 Qc6 Rd8+ 36 Rd6 Qc8 37 cxb4 and the
checks have run out.
30 Kd2 Qa2+ 31 Kd1 Re8
31 ... Qb1+ 32 Nc1 Rd8+ 33 Nd6 there is no more attack and the two extra pieces
will count here.
32 Qd3 Qa4+ 33 Qc2! Qxc2+
33 ... b3 34 Qb2 Qa2 35 Qxa2 bxa2 36 Kd2 the pawn has been stopped.
34 Kxc2 Rxe2+ 35 Kb3 bxc3 36 Rd1

Technically Black is ahead in material, but the f5-knight is clearly superior to the
black pawns which are not very far advanced.
36 ... Rd2
36 ... c2 37 Rd8+ Re8 38 Rd7 c1N+ (38 ... c1Q 39 Rdxf7+ Kg8 40 Rg7#) 39 Kc2
Rc8+ 40 Kd1 Rh7 and now a lovely move: 41 Rc6! wins the knight on c1 as 41 ... Rxc6
allows mate on d8.
37 Rc1
37 Rxd2 cxd2 38 Rd6 f6 39 Rxd2 was also winning.
37 ... c2 38 Rc6 Kg8 39 Nxh6+ Kh7 40 Nxf7 Rg8 41 R1xc2 Rxc2 42 Kxc2 Rg7 43
Rf6 Kg8 44 Ne5 Kh7 45 Rf5 1-0
So quite a crazy game, but even the world’s top players sometimes look to mix it in
the Queen’s Indian.

Game 4
J.Lautier-A.Karpov
Amber Rapid, Monaco 2000

1 d4 Nf6 2 c4 e6 3 Nf3 b6 4 g3 Ba6 5 Nbd2 d5


Karpov follows my recommendation in Chapter Two.
6 Bg2 Be7 7 0-0 0-0

8 Ne5

Question: Why play this move so early, when
development of the c1-bishop is more important?

Answer: b2-b3 and Bc1-b2 is of course natural, but 8 Ne5 opens up the g2-bishop’s path towards the a8-rook. It
also prevents 8 ... Nbd7, as then 9 Nc6 secures a strong square for the knight. Others:
a) 8 Qc2 Bb7 9 b3 a5 10 Bb2 Na6 11 Rac1 a4, as played in J.Aagaard-V.Gavrikov,
Gothenburg 1998 is similar to the game.
b) 8 b3 Nbd7 9 Bb2 c5 10 Ne5 Rc8 was equal in J.Timman-B.Gelfand, Wijk aan
Zee 1998.
8 ... Bb7
8 ... c5 9 cxd5 exd5 10 Ndf3 Re8 11 b3 Ne4 12 Bb2 Bb7 13 Rc1 Nd7 14 Nd3 Rc8
A.Hoffman-P.Zarnicki, Villa Gesell 1998 with a typical hanging pawns scenario if
White ever plays dxc5.
9 cxd5 exd5
It is clear that White would much rather have the d2-knight on c3, to add extra
pressure on the d5-square. This will be explained further in Chapter Two (Game 7,
Radjabov-Gelfand).
10 b3 a5 11 Qc2 Na6 12 Bb2 c5 13 dxc5 bxc5

Here we encounter a typical and important pawn structure in the centre, the hanging
pawns. They can be regarded as either weak or strong, depending on whether White can
attack them effectively. Here though, they are fine because the d2-knight is not exerting
any pressure on d5 (as it would do from the c3-square) and Black can play ... a5-a4 to
soften up White’s queenside as we see in the game.
14 Rad1
A solid move, but Karpov now demonstrates the potential of Black’s setup.
14 ... a4! 15 bxa4
15 e4 Nb4 16 Qb1 d4 allows use of the c4-square, but Black is better due to the
queenside pressure and passed d-pawn.
15 ... Nb4 16 Qb1 Qc7
16 ... Rxa4 17 a3 Nc6 18 Ndc4 allows a little more pressure than in the game.
Karpov plays it much more accurately.
17 e4

Exercise: What should Black do about the attack on the d5-pawn?

Answer: 17 ... d4!
Keep those central passers! Why not? Any endgame will be excellent for Karpov,
and it means exchanges will be very welcome for him.
18 a3 Nc6 19 Nec4 Rxa4 20 Rfe1
20 Qc2 Rfa8 21 f4 was better, to try and generate some kingside play.
20 ... Na5 21 Nxa5 Rxa5 22 Qc2 Qc6 23 Nc4 Ra7 24 f4 Nd7 25 Qd2
25 e5 Qxg2+ 26 Qxg2 Bxg2 27 Kxg2 Ra4 28 Rc1 Rb8 (threatening ... Rxc4) 29 Re2
Nb6 30 Nxb6 Rxb6 and now that the blockade of c4 is cleared, Karpov could set about
trying to push those pawns.
25 ... Nb6 26 Rc1 Nxc4 27 Rxc4 Rd8 28 Rec1 Qb6 29 a4 Ba6
Karpov’s strategy has come to fruition. Compare this position to the diagram on
move 13 – the hanging pawns are now extremely strong passed pawns, supported by the
rest of Black’s army.
30 a5 Qb3 31 Rxc5
What else?
31 ... Bxc5 32 Rxc5 Rb7 33 Bxd4 Qa4 34 Kf2 Rb4! 0-1
Karpov was never going to fall for 34 ... Qxd4+?? 35 Qxd4 Rxd4 36 Rc8+.

Game 5
J.Piket-V.Anand
Cologne 1995

1 d4 Nf6 2 c4 e6 3 Nf3 b6 4 Nc3 Bb4


Anand transposes to a variation of the Nimzo-Indian, but one that has a lot in
common with Queen’s Indian lines.
4 ... Bb7 5 a3 d5 is the subject of Chapter Six.
5 Bg5 Bb7 6 e3 h6 7 Bh4 c5
Not the only move, but a standard one to know about. Black chips away at White’s
centre and exerts some pressure.
7 ... g5 8 Bg3 Ne4 9 Qc2 transposes to a line examined in Chapter Eight.
8 dxc5
Alternatively:
a) 8 d5 is always a reply to consider, in this case after 8 ... exd5 9 cxd5 g5 10 Bg3
Nxd5 it is an interesting pawn sacrifice, bearing in mind that Black cannot castle either
way safely. We will see Black play this ... g7-g5 move to break the pin, but it can be
construed as weakening. It is up to you to decide!
b) 8 Bd3 is the most common move, and that makes sense as it fights for e4, for
example 8 ... cxd4 9 exd4 0-0 10 0-0 Be7 11 Re1 d6 12 h3 Nbd7 13 Bg3 d5 14 Rc1
dxc4 15 Bxc4 Rc8 16 Bb3 with an unclear IQP position, V.Bologan-V.Iordachescu,
Moscow 2008.
8 ... Bxc5 9 Bd3 Be7 10 0-0 0-0 11 Rc1


Exercise: What should Black do with the b8-knight?

Answer: Almost anywhere is good, in this book you will learn the various methods of development Black has at
his disposal.
11 ... Na6
Or:
a) 11 ... d6 12 Nd4 Nbd7 with a hedgehog structure is also absolutely fine.
b) 11 ... Nc6 is OK but I prefer putting the knight on d7, or executing the ... Na6-c5
manoeuvre, attacking e4, which Anand plays in the game.
12 Bb1 Nc5 13 Nd4 Nfe4
Black frees the position with exchanges, though it was possible to maintain the
tension as well.
14 Bxe7 Qxe7 15 Nxe4 Bxe4 16 Bxe4 Nxe4 17 Qc2 Nf6 18 Rfd1

Question: This position looks deadly dull, how am I
going to checkmate my opponent from this sort of position?

Answer: Interesting, since Anand does end up doing so! Positional play is called for here, and it is vital to add
positional understanding to your game to improve as a chess player. Watch how Anand increases his advantage slowly
over the coming moves.
18 ... Rfc8
The queenside is where he will play, and rook to the semi-open file is a good start.
19 Qa4 a6 20 Qb3 Qc5 21 h3
21 Rc2 prevents the coming idea, but I prefer Black as he can play for ... d5 or ... b5,
whereas White has no real plan here except to shuffle around. Black can then play 21 ...
Rc7 with the idea of doubling on the c-file and preparing the above mentioned pawn
breaks.
21 ... b5!
Anand does not miss such chances.
22 cxb5 Qxc1 23 Rxc1 Rxc1+ 24 Kh2 axb5 25 f3
25 a3 to save the queenside was preferable.
25 ... b4

26 e4
26 Nc2 was a better attempt to hold, though after 26 ... Rb8 White should not play 27
Nxb4? because of 27 ... Nd5 28 a3 Rc5 29 Qd3 Nxb4 30 axb4 Rd5 with the plan to
eventually win both b-pawns.
26 ... Ra1 27 Qxb4 R1xa2 28 h4 R2a4 29 Qc3 Rb8 30 b3 Ra2 31 b4 Rba8 32 b5
Rf2
White has been outplayed in classical style.
33 Nc2
33 b6 Raa2 34 b7 Rxg2+ and now:
a) 35 Kh1 Rgb2 36 Nc2 Rb1+ 37 Kg2 Rab2 wins the dangerous b-pawn.
b) 35 Kh3 Rh2+ 36 Kg3 Nh5+ 37 Kg4 Rag2+ 38 Kxh5 g6+ 39 Kxh6 Rxh4 is mate.
33 ... Ra2 34 Ne1 Rfb2 35 Qc4 d5 36 Qc8+ Kh7 37 e5 Nh5 38 Qc1 Rxb5
Once this pawn goes, so does the game.
39 Kh3 Rbb2 40 Qe3 Re2 41 Qb3 Nf4+ 42 Kg4 Ng6 43 f4
Now there is an excellent tactical finish from Anand.
43 ... Rxg2+! 0-1
Excellent technique from Anand, who showed the dynamic potential of Black’s Queen’s Indian pawn structure with ...
d5 and ... b5 pawn breaks. Hopefully now this will entice you to read further, and delve into the mysteries of the
Queen’s Indian Defence!
Chapter Two
Fianchetto Variation: 4 g3 Ba6 5 Qc2
and Minor Fifth Moves
1 d4 Nf6 2 c4 e6 3 Nf3 b6 4 g3
The 4 g3 variation is the main line against the Queen’s Indian Defence, and with
good reason. Since Black is going to challenge the e4- and d5-squares, White opts
to post his bishop on g2 and fight for these squares too. White delays the
development of the b1-knight, as this knight is waiting to see where the f8-bishop
decides to go.
4 ... Ba6
This is the move I am going to recommend as our main line against the 4 g3 systems.


Question: Why play 4 ... Ba6? The c4-pawn can surely be
defended easily. Didn’t you say the d5- and e4-squares
are more important and therefore 4 ... Bb7 should be played?

Answer: Actually the c4-pawn is a key target for Black in the Queen’s Indian (as well in the Nimzo-Indian), and
defending this pawn is likely to lead to a compromise in piece or pawn placement. Often the bishop returns to the b7-
square at some point, having done its dirty work on a6.
From the diagram position above, 5 b3 is the most popular choice for White, and
will be covered in the next chapter. In this chapter we will look at 5 Qb3 (Game 6), 5
Nbd2 (Game 7), 5 Qa4 (Game 8) and finally 5 Qc2. In the last of these lines, after 5
Qc2 Bb7 6 Bg2 c5, the pawn sacrifice 7 d5!? was very fashionable in top level chess
for a while, so it is important to know how to proceed with Black. This continuation is
covered in detail in Games 9-11.

Game 6
L.Psakhis-E.Lobron
Baden-Baden 1992

1 d4 Nf6 2 Nf3 e6 3 c4 b6 4 g3 Ba6 5 Qb3


Not the main move, but a tricky one nonetheless. It prevents Black’s bishop from
coming to b4 and so in some lines, the knight comes to c3, its best square.

5 ... Nc6!

Question: What is the point of this?

Answer: This odd looking move serves to counter White’s idea mentioned in the previous note. The threat of ...
Na5 almost forces White to now play the b1-knight to d2, which is not its best square. Black’s c-pawn, most
commonly used as a source of counterplay in the centre, is blocked for the moment but not for long!
6 Nbd2
6 Bd2 Bb7 7 d5 (7 Bg2? Nxd4! was the trick set up by Black’s last move) 7 ... Ne7
8 dxe6 fxe6 9 Bc3 Ne4 10 Nbd2 Nxc3 11 Qxc3 Ng6 was more than satisfactory for
Black, with the two bishops and potentially useful f-file, as played in F.Caruana-
S.Karjakin, Russian League 2012.
6 ... Na5 7 Qa4
7 Qc3 is another important line. Black must act immediately as 8 b4 is threatened,
and the queen on c3 also eyes up a quick e4-e5 push. After 7 ... c5 8 dxc5 bxc5 White
has tried:
a) 9 Bg2 Bb7 10 0-0 Be7 11 b3 0-0 12 Bb2 d6 13 Rfd1 Qb6 14 Qc2 h6 when Black
stood more than fine in L.Voloshin-R.Cvek, Czech Championship 2012.
b) 9 e4 Bb7 10 e5 Ne4 11 Nxe4 Bxe4 12 Bg2 Nc6 13 0-0 Rb8 Black’s aim of
getting the knight quickly to d4 is even more important than castling early here, mainly
because the centre is closed. 14 Re1 Bxf3 15 Qxf3 (15 Bxf3 Nd4 was seen in
A.Shirov-B.Gelfand, Bazna 2009 and now 16 Bg2 would maintain equality) 15 ... Nd4
16 Qd3 Be7 17 Rb1 a5 18 Bd2 0-0 19 Be4 g6 with a complicated fight ahead as played
in E.Bareev-A.Grischuk, Moscow (blitz) 2007.

7 ... Bb7

Question: Why not play 7 ... c5 immediately, doesn’t that feel right?

Answer: It is possible, but because Black doesn’t control the e4-square, White has an active idea here: 7 ... c5 8
e4! Bb7 (8 ... cxd4 9 b4 Nc6 10 b5 wins a piece) and after 9 d5 exd5 10 exd5 Be7 11 Bd3 0-0 12 0-0 d6 13 Re1
White was better in R.Wojtaszek-N.Djukic, Istanbul Olympiad 2012 as the knight was stranded out on a5.
8 Bg2 c5
Now this is the right time, as e2-e4 is not a threat in this position.
9 dxc5 bxc5
After 9 ... Bxc5 10 b4 Bc6 11 Qa3 Be7 12 Qc3 Nb7 13 Bb2 White was slightly
better because of Black's awkwardly placed pieces in G.Sargissian-A.Zhigalko,
European Club Cup, Rogaska Slatina 2011.
10 0-0
10 ... Qc7!
10 ... Be7 11 Ne5 Bxg2 12 Kxg2 Qb6 13 Ndf3 Qb4 14 Qc2 Qb7 15 Rd1 Nc6 should
also be playable. Black must be careful not to leave the knight out on a5 for too much of
the game.
11 Re1
11 Nb3 Nxb3 12 Bf4 Bd6 13 Bxd6 Qxd6 14 Qxb3 Qc7 was completely fine for
Black in R.Dautov-Z.Ribli, German League 1991. In the long term ... a5 and ... Rb8,
with pressure down the queenside, would give Black some winning chances.
11 ... Nc6 12 e4 Bd6!
We have spoken about the important d5- and e4-squares in the Queen’s Indian (and
the Nimzo-Indian also) however here the d4- and e5-squares are crucial too.
Instead, after 12 ... e5 13 Nb1 Nd4 14 Nc3 White wrests control of d5, at the
expense of some development.
13 Qc2
13 b3 Ne5 is comfortable for Black, and could get better after 14 Bb2?! Nd3 15
Reb1 Nxb2 16 Rxb2 Nxe4 with a clear pawn advantage.
13 ... Ng4! 14 b3

14 ... h5
Interesting, but there was a more solid alternative:
14 ... 0-0 was the logical idea, and Black still gains control of the key central
squares after 15 Bb2 f6 16 Rad1 Nge5 17 Nxe5 Bxe5 18 Nf3 Bxb2 19 Qxb2 e5 with ...
Nd4 to come.
15 Bb2 f6 16 Rad1 0-0-0 17 Nf1 Nge5 18 Qd2 Nxf3+ 19 Bxf3 Be5 20 Bxe5

20 ... fxe5
20 ... Qxe5 21 Bg2 h4 and 20 ... Nxe5 21 Bg2 g5 might have been safer than the
game, as now White manages some counterplay.
21 Qg5 h4
21 ... Nd4 22 Bxh5 Nc2 23 Re2 Nd4 24 Ree1 Nc2 with an easy draw for Black if he
wants.
22 Qxg7
A brave pawn grab, White decided he might as well have some material to show for
his passive position.
22 ... Nd4
22 ... Rdg8 might have been slightly more accurate, however Black still has
compensation for the pawn minus in any case after 23 Qf7 Rf8 24 Qg7 Nd4 25 Bg2.
23 Bg2 Rdg8 24 Qe7 a5 25 Rd3 d6 26 Qxc7+ Kxc7

Despite the queen exchange, Black still has pressure. The plan to put the knight on d4
has been successful, though White clings on to a draw here.
27 gxh4
27 h3 Bc6 28 g4 a4 29 Nd2 Rb8 with pressure on the queenside now.
27 ... Rxh4 28 Rg3 Rf8 29 Nd2 Rhf4 30 Rg7+ Kb6 31 f3 Bc6 32 Kf2 Nc2 33 Re2
Nd4 34 Re3 Nc2 35 Re2 Nd4 ½-½
The game was agreed drawn, however Black was the one pressing in this game.

Game 7
T.Radjabov-B.Gelfand
Odessa (rapid) 2007

1 d4 Nf6 2 c4 e6 3 Nf3 b6 4 g3 Ba6 5 Nbd2




Question: This obvious move 5 Nbd2 must make the bishop on a6 redundant now?

Answer: As previously stated, the knight does not really want to go to d2 (c3 is a more active square) and Black
can still apply pressure to the central pawns on c4 and d4, as Gelfand demonstrates. Once Black plays ... c5 and ...
d5, the knight is terribly missed on c3 as from there is could inflict some damage to the black centre.

Question: Can we play 5 ... c5 here to attack d4,
since the queen is not defending d4 now?

Answer: Yes it is possible, but from my own experience (see variation below) Black must be very careful not to
allow White to successfully execute the following plan: 5 ... c5 6 e4! cxd4 7 e5 Ng8 (7 ... Ng4 8 h3 Nh6 9 Bg2 Nc6 is
still playable for Black) 8 Bg2 Bb7? (8 ... Nc6 9 0-0 Bb4 was the last chance to get a position) 9 0-0 d6 10 Nxd4! and
Black is under pressure, for example:
a) 10 ... Bxg2 11 Kxg2 dxe5 12 Qf3 Nd7 13 Nb5 Rc8 14 Ne4 with Ne4-d6+ coming
next.
b) 10 ... Qd7 11 Ne4 d5 (11 ... dxe5 is answered by 12 Nb5) 12 cxd5 Bxd5 13 Nd6+
Bxd6 14 Bxd5 Bxe5 15 Nxe6 Nc6 as played in Q.Loiseau-L.D’Costa, Paris 2012. Now
White could have applied maximum pressure with 16 Qf3! Rc8 17 Ng5 and Black is in
all sorts of trouble. So do watch out for this early e4-e5 push.
Instead, Black took control of the e4-square with:
5 ... d5

6 Bg2 Be7 7 0-0 0-0 8 b3 c5
Now it is safe to play this push, and with ... Nc6 coming Black gets an active
position.
9 Bb2 Nc6!

Usually the b8-knight goes to d7, but if it can come to the active c6-square then why
not? Compare this to White’s knight on d2, which would prefer c3 but was forced to
compromise early on due to 4 ... Ba6.
10 dxc5 Bxc5
10 ... bxc5 11 cxd5 exd5 gives a very good hanging pawns structure for Black,
although that is not to everyone’s taste. Gelfand goes for the active option.
11 Rc1 Rc8 12 e3 Qe7 13 a3


Exercise: Obviously White threatens b3-b4 and b4-b5
winning a piece, but isn’t his last move a blunder?

Answer: No, as after 13 ... Bxa3 14 Bxa3 Qxa3 15 Ra1 the a6-bishop is lost.

13 ... dxc4 14 Bxf6
This cannot be what Radjabov intended when entering this line. However, Black’s
activity is highlighted after the routine capture on c4. After 14 Nxc4 Rfd8 15 Qe2 there
are two good options for Black:
a) 15 ... Bxa3! 16 Bxa3 Qxa3 17 Ra1 Bxc4 18 Qxc4 Qd6 with a clear pawn
advantage.
b) 15 ... Na5 is also strong, 16 Nfe5 Bxa3 17 Bxa3 Qxa3 18 Nxa3 Bxe2 again with
an extra pawn for Black.

Exercise: How should Black recapture on f6?

Answer:
14 ... gxf6!
14 ... Qxf6 15 b4 Be7 16 Qa4 Bb7 17 Nxc4 could be nice for White, as he has
regrouped.
15 b4
15 Nxc4 Rfd8 16 Qe2 Bxa3 is similar to the note to White’s 14th move.
15 ... c3 16 Rxc3 Bxf1 17 Bxf1 Bd6
An exchange ahead, Gelfand is not going to miss the opportunity. Notice how despite
the apparently weakened position of Black’s king, White is not really able to get any
attacking chances.
18 Qc2 Qb7 19 Qe4 f5 20 Qh4 f6 21 Bc4 Qe7 22 e4 Ne5 23 exf5 Nxc4 24 Nxc4
exf5
Radjabov has done his best to drum up something, but with precise play it will not
be enough.
25 Rd3 Qe4 26 Rxd6 Rxc4 27 Qxe4 fxe4 28 Nh4 f5 29 Kf1 f4 30 Re6 f3 0-1

Game 8
Wang Yue-E.Bacrot
Nanjing Pearl Spring 2010

1 d4 Nf6 2 c4 e6 3 Nf3 b6 4 g3 Ba6 5 Qa4


Another queen move to defend the c4-pawn, but unlike in the 5 Qb3 variation, the
queen is not so vulnerable on a4. This old line has regained popularity after Kramnik’s
excellent win over Carlsen (given below) but of course Black has resources too.
5 ... Bb7 6 Bg2 c5
This had to be the central pawn push to make since the d7-pawn is pinned, but since
White’s d4-d5 thrust is not on the cards here, this move is extremely adequate.
6 ... Be7 7 0-0 0-0 is possible, to try and play ... c5 or ... d5 at a later stage, however
I see nothing wrong with the quicker ... c5 push.

7 dxc5
7 0-0 is the alternative, though I believe Black is comfortable there. After 7 ... cxd4
8 Nxd4 Bxg2 9 Kxg2 Black can choose between:
a) 9 ... Qc7 10 f3 a6 11 Rd1 Ra7 12 Nc3 Be7 13 e4 0-0 14 Be3 d6 15 Rac1 Rc8 16
b4 Qb7 17 Qb3 Nbd7, with a typical Hedgehog structure that can often arise in the
Queen’s Indian, was V.Korchnoi-Z.Almasi, Paks 2005.
b) 9 ... Qc8 10 Nc3 Be7 11 Bf4 a6 12 Rac1 0-0 13 Rfd1 d6, as in R.Borgula-
W.Moranda, Polish League 2008, is similar. The black queen heads to the excellent b7-
square, where it looks over the b5- and d5-squares whilst also eyeing up the white king
on g2.
7 ... bxc5
7 ... Bxc5 is also playable, and the variation I used to prefer. I will quickly
demonstrate how play can pan out from here, 8 0-0 0-0 9 Nc3 and now:
a) 9 ... Ne4 10 Qc2 Nxc3 11 Qxc3 Be4 12 Be3 Qc7 13 Rfd1 Rc8 14 Bxc5 Qxc5 15
Ne5 Bxg2 16 Kxg2 f6? 17 Qf3 Nc6 18 Nxc6 dxc6 19 Rd7 Rd8 20 Rad1 Rxd7 21 Rxd7
Re8 22 Qg4 g6 23 Qf4 1-0 was a quick win for White in R.Wojtaszek-V.Iordachescu,
European Championship, Yerevan 2014.
b) 9 ... Be7 10 Rd1 Na6 11 Bf4 Nc5 12 Qc2 Qc8 13 Rd4 d5 14 Rc1 with unclear
play. One high-level example went 14 ... dxc4 15 Rxc4 a5 16 Na4 Nfd7 17 Ng5 Bxg5
18 Bxb7 Qxb7 19 Nxc5 Nxc5 20 Bxg5 a4 21 Be3 and Black’s queenside position felt a
bit ropey in S.Mamedyarov-P.Leko, Moscow (blitz) 2008.
8 0-0 Be7 9 Nc3 0-0 10 Rd1 Qb6 11 Bf4


Exercise: Can Black grab the (poisoned?) pawn on b2?

Answer: Not really, as although the queen cannot be trapped immediately, there are long term problems getting
her out safely, for example 11 ... Qxb2 12 Rab1 Qxc3 13 Rxb7 (13 Be5 Bc6 might hold but it still looks dangerous) 13
... Nc6 14 Bd2! and the queen will have to leave the board. Instead, the game continued:
11 ... Rd8 12 Rd2
12 Rab1 was played in the aforementioned Kramnik win over Carlsen. The rook
sortie is designed to allow a potential b4-pawn advance. After 12 ... h6 (12 ... d6 13 b4
was White’s idea, with dangerous play) 13 Qb5 Bc6 14 Qxb6 axb6 15 b3 g5 16 Bxb8
Raxb8 17 Ne5 Bxg2 18 Kxg2 Rbc8 19 e4 d6 20 Nd3 Kf8 when against most players,
Black should be OK here, but Kramnik manages to extract the maximum in a well-
played game. It is well worth playing through to see how to he broke Carlsen’s
resistance. 21 h3 Nd7 (21 ... d5 doesn’t really solve the problems since after 22 e5 Ne4
23 Nb5 Ra8 24 Rb2 d4 25 Rc1 and with Kg2-f3 or f2-f3 next, the e4-knight is a bit
shaky) 22 f4 Nb8 23 Na4 Nd7 24 Nf2 Ra8 25 Rd2 Ra5 26 Nc3 Nb8 27 Rbd1 Nc6 28
Nb5 gxf4 29 gxf4 d5 30 exd5 exd5 31 cxd5 Rxb5 32 dxc6 Rc8 33 Ng4 Rxc6 34 Ne5
Re6 35 Kf3 Ra5 36 Rg2 f6 37 Nc4 Ra7 38 f5 Rc6 39 a4 Rb7 40 Rg6 h5 41 Rdg1 Ke8
42 Ne5 Rc8 43 Rg8+ Bf8 44 Ng6 Rf7 45 Rd1 1-0, V.Kramnik-M.Carlsen, Bilbao 2010.
12 ... d6 13 Rad1 h6

14 Qb5
14 Qb3 used to be the main line in this position but recent results among top players
haven’t been very convincing for White.
14 ... Ne8 15 e4 g5
A risky move, but Bacrot envisages that the queens will come off anyway so his king
will not be in danger. Also, it alleviates the pressure on d6 by forcing the bishop back.
16 Be3 Nd7 17 h4 g4 18 Ne1
18 ... Kg7
18 ... Qxb5 19 cxb5! gives White the chance to use c4 as a useful square for the
knight. Now D.Navara-P.Leko, Khanty-Mansiysk Olympiad 2010 continued 19 ... Bf8
20 Nc2 Ne5 21 Na3 Rdc8 22 Bf4 Bg7 23 Bxe5 Bxe5 24 Nc4 Bd4 25 Ne2 e5 26 a4 Kf8
27 Rc2 Rc7 28 Na5 which was unclear but gave Black a difficult defence over the
board.
19 Bf4?!
‘A complicated position arises after 19 Qa4 Bc6 20 Qc2 where the caveats of 18 ...
Kg7 become apparent. White could keep the queens on the board and Black’s king
shelter would be attacked with even heavier artillery!’ according to GM Moriadiabadi,
and I agree!
19 ... Bc6 20 Qxb6 axb6 21 Nc2 Ne5 22 Ne3
22 ... Nf3+!
22 ... h5 23 Ncd5 looks dangerous, even if the computer says that it is probably
equal. If Black plays 23 ... exd5? then 24 Nf5+ Kf8 25 Bh6+ and White is much better.
23 Bxf3 gxf3
Black is relying on the bishop on c6 to make its presence felt, which Bacrot does in
style.
24 h5 Bf6 25 Ng4 Bxc3 26 bxc3 Bxe4 27 Bxh6+ Kh7 28 Bg5 Rd7 29 Bf4 f6 30
Kh2??
Wang Yue must have been in time trouble, as White loses a piece here. Instead, after
30 Bxd6 Rad8 31 Bf4 Rxd2 32 Rxd2 Rxd2 33 Bxd2 Bd3 34 Ne3 f5 35 Kh2 Nf6 Black
has the chances even though he is temporarily a pawn down.
30 ... e5 31 Nxe5
31 Be3 Rg7 32 Nh6 f5 is no better, with a timely ... f4, to win the knight, on the
cards.
31 ... fxe5 32 Bxe5 Bf5 33 Bxd6 Rad8 34 Bf4 Rxd2 35 Rxd2 Rxd2 36 Bxd2 Nd6
Materially, White is currently equal, but his pawns will all fall like skittles now.
37 Bf4 Nxc4 38 g4 Bxg4 39 h6 Be6 40 Kg3 Bd5 41 Kg4 Nb2 42 a3 Nd1 43 c4
Nxf2+ 0-1
So the 5 Qa4 line is definitely worth checking, as this game demonstrates that White
has some ideas in this variation.

Game 9
Y.Vovk-H.Melkumyan
Lviv 2007
1 d4 Nf6 2 c4 e6 3 Nf3 b6 4 g3 Ba6 5 Qc2

Like many chess opening variations, this line has gone in and out of fashion over
time. The 5 Qc2 variation has certainly hit the heights in recent years with the new idea
of a pawn sacrifice on d5, as we will see in this game. In general, 5 Qc2 feels quite
natural – Black has played ... Ba6 to try and encourage White to compromise on piece
placement to defend c4, but Qd1-c2 is a typical move to try and put pressure on the
important e4-square. Of course, if Black were able to play ... c5 and ... Rc8 quite soon,
the queen might be exposed. It is precisely for this reason that we see White go on the
offensive at an early stage, as a slower game would not benefit him with the queen
placed like this.
5 ... Bb7

Question: Why did we lose time by playing ... Ba6 then back to b7,
especially if queen to c2 is a move White might want to play?

Answer: White may have control over the e4-square, but he has loosened himself on the other important central
square, d5. This will become apparent in a moment.
6 Bg2 c5 7 d5!
In my opinion, the most testing move for Black to face. All other variations give
Black a comfortable game. For example:
a) 7 dxc5 Bxc5 8 0-0 a6 9 a3 Be7 10 Nbd2 Qc7 11 b4 d6 12 Bb2 Nbd7 13 Rac1
Rc8 14 Qb1 0-0, with a completely acceptable Hedgehog position for Black, was
F.Bersoult-L.D’Costa, Dieppe 2009. In these Hedgehog positions, Black waits for a
chance to strike out with a ... b5 or ... d5 pawn break, and Black’s setup is difficult for
White to break down. In my opinion, it is a good line to play against both stronger and
weaker players. The Hedgehog is both solid and gives chances to play for a win, since
most of the pieces remain on the board.
b) 7 0-0 cxd4 8 Nxd4 Bxg2 9 Kxg2 reaching the diagram position below.

Now Black should post the queen on b7, via c7 or c8, for example 9 ... Qc7 10 Qd3
a6 11 Nc3 Be7 12 e4 0-0 13 f3 Rc8 14 Na4 Nc6 15 b3 Ne5 16 Qe2 Qb7 17 Be3 d6
was played in D.Elizarov-A.Lastin, Belorechensk 2010. In general, Black should not be
worried about the trade of the light-squared bishops. He has a good Hedgehog position,
can fight for ... d5 or ... b5, and if the position should ever open up then the white king
on g2 might feel a bit draughty.
c) 7 e3 cxd4 8 exd4 d5 9 0-0 dxc4 10 Qxc4 Be7 11 Nc3 0-0 12 Rd1 Nc6 13 Bf4
Na5 14 Qe2 Rc8 15 Rac1 Nd5 16 Nxd5 Qxd5 gave Black a very nice position in
G.Skurygin-P.Kostenko, Pavlodar 2007.
7 ... exd5 8 cxd5 Nxd5
If Black takes on d5 with the bishop, the power of White’s initiative can be seen in
the following example: 8 ... Bxd5 9 Nc3 Bc6 10 e4 d6 11 Bf4 Nh5 12 0-0-0 Nxf4 13
gxf4 Be7 14 e5 0-0 15 h4 Qc7 16 Ng5 g6 17 Bd5 dxe5 18 Nxh7! Kxh7 19 h5 Kg7 20
hxg6 fxg6 21 Rdg1 Qd6 22 fxe5 1-0 P.Tregubov-A.Greet, European Club Cup, Fuegen
2006.

Question: On what grounds can White justify this pawn sacrifice?

Answer: White gains a lead in development, and there are now always tactics in the air on the h1-a8 diagonal that
Black needs to monitor. The semi-open d-file is also a cause for concern, as White can post a rook on d1 very quickly
and apply some pressure. Even if Black consolidates, it can be difficult to utilise the extra d-pawn, as White often
plays e2-e4 and tries to clamp down on the d5-square. However, a pawn is a pawn, and a central one at that! If Black
does weather the storm, he can certainly play for a win. This is why I am recommending snatching the pawn with 8 ...
Nxd5.
9 0-0 Be7 10 Rd1
At this early stage Black must already tread carefully. Threats are now appearing on
d5 and therefore indirectly on b7 and d7 also.
10 ... Nc6

An important tabiya for this variation. Apart from 11 Qf5, White has 11 a3 and 11
Qa4 which are examined in the following two games.
11 Qf5
11 Rxd5?? Nb4 was the trick justifying Black’s last move.
11 Nc3 is another interesting sideline worth considering. 11 ... Nxc3 12 bxc3 0-0 13
Ne5 Nxe5 14 Bxb7 Rb8 15 Be4 g6 16 Qa4 Qc7 17 Rb1 Bf6 18 Bf4 d6 19 c4 Rfd8 was
very unclear in V.Bologan-A.Ivanov, Moscow 2007. Black retains the pawn advantage
though it is obvious there is some pressure in return. It is hard to know who is better!
Positions such as this are common in this variation.
11 ... Nf6 12 e4
12 ... g6

Question: Why are we playing this weakening move?

Answer: Black must deal with the threat of e4-e5 and the pressure on the d7-pawn. Alternatives are:
a) 12 ... 0-0 is not good enough as the threat of e4-e5 is too strong. After 13 e5 g6 14
Qh3 Nh5 15 Bh6 Ng7 16 Rxd7 (16 Nc3 retaining the idea of taking on d7 is also good)
16 ... Qc8 17 Bxg7 Kxg7 18 e6 fxe6 19 Ne5! and this is the sort of position we should
try to avoid.
b) 12 ... d6 13 e5 Qd7! 14 Qxd7+ Nxd7 15 exd6 Bf6 16 Re1+ Kf8 17 Nc3 Nb4 18
Bg5 Nc2 19 Re7 is extremely unclear. From here play could go:
b1) 19 ... Bxf3 20 Bxf3 Bxg5 21 Rxd7 Rd8 22 Rxd8+ Bxd8 23 Rd1 Nd4 was
M.Carlsen-Y.Pelletier, Biel 2008, which was a bit better for White.
b2) 19 ... Bxe7 20 Bxe7+ Kg8 21 Rd1 and it’s anyone’s guess what is going on here,
even if the computer favours Black.
13 Qf4 0-0 14 e5
Although this move is strong, because White’s queen is no longer on f5 there is less
pressure on d7 and so Black can organise a defence.
14 ... Nh5
15 Qg4
15 Qh6 looks much more threatening, but Black has defensive resources too. 15 ...
Nd4 and now:
a) 16 Ng5 Bxg5 17 Bxg5 Qc7 18 Rxd4 cxd4 19 Bxb7 Qxb7 and White’s attempt to
give checkmate falls just short, for example 20 g4 (or 20 Bf6 Qc6) 20 ... Rae8 21 gxh5
Rxe5 22 Nd2 f6 23 Bh4 Rxh5 24 Qf4 g5) beats off the attack with a material plus.
b) 16 Nxd4 Bxg2 17 Kxg2 cxd4 18 g4 reaches the position in the analysis diagram
below.

Analysis Diagram

Exercise: What should Black do here to defend?

Answer: He has a great defence with 18 ... Rc8!! when White can try:
b1) 19 gxh5 g5 followed by ... Rc6 trapping White’s queen! For example 20 e6 Rc6
21 Rxd4 Rxe6 and Black wins.
b2) 19 Rxd4 Ng7 20 Nc3 f5 and although White managed to hold a draw in
L.Aronian-P.Leko, Moscow 2006, I much prefer Black here.
15 ... Qb8!
Now d7 is momentarily poisoned.
16 Bh6
After both 16 Qxd7 Rd8 and 16 Rxd7 Bc8 material is lost for the first player.
16 ... Re8 17 Nc3

17 ... Nxe5
Snatching a second pawn, which is risky but if the e5-pawn can be removed then it
should be, as it was a focal point for White’s attacking play.
18 Nxe5 Bxg2 19 Nxf7
Alternatively:
a) 19 Nxd7 Qb7 keeps everything defended.
b) 19 Nxg6 hxg6 20 Kxg2 Qb7+ 21 Kg1 Nf6 22 Qc4 Rad8 23 Nd5 Nxd5 24 Rxd5
Bf6 is another example of Black keeping hold of the extra pawn and developing safely.
19 ... Nf6 20 Qh4 Bc6 21 Bf4
21 ... Nh5!
A nice counterattacking move. We have reached the critical moment of the game – if
Black defends he should be successful, if not ...
22 Ng5 Qd8 23 Be3 d5 24 Qg4 Qd6 25 Rac1 Bxg5 26 Bxg5 d4
The initiative has gone. The rest of the game must have been difficult for White to
play.

27 f3 Qe6 28 Nb1 Qxa2 29 b4 Qb3 30 Nd2 Qxb4


Now the pawns fall one by one.
31 Re1 Qa4 32 Bh6 Qa2 33 Rxe8+ Rxe8 34 Nc4 Qe2 35 Nd2 Qe6
Once the queen gets back to defend, it’s all over for White.
36 Qh4 Nf6 37 Bg5 Nd7 38 Rf1 a5 39 g4 a4 40 f4 Qd5 41 Nf3 Re3 42 f5 Rxf3 0-
1
A nice game by the Armenian Grandmaster. He defended carefully and once he
dampened White’s initiative, he was quick to take advantage.

Game 10
A.Getz-M.Adams
World Open, Philadelphia 2011

1 d4 Nf6 2 c4 e6 3 Nf3 b6 4 g3 Ba6 5 Qc2 c5

An interesting move-order.
6 d5
6 Bg2 Nc6!. This is the point – the bishop can remain on a6 and attack c4. ... Rc8
ideas are in the air, and also we see the queen on c2 not defending the d4-pawn (a
drawback of the Qc2 variation). After 7 dxc5 Bxc5 8 a3 Black has a choice:
a) 8 ... Rc8 might be playable, with interesting play arising from 9 b4 Bd4 10 Ra2
Bxc4 11 Qxc4 Na5 12 Qd3 which was played in H.Golda-G.Euler, German League
2002, and now maybe 12 ... Rxc1+ 13 Kd2 Rxh1 14 Bxh1 Bxf2 15 bxa5 0-0 gives
compensation for the piece
b) 8 ... Be7 9 0-0 Rc8 10 Rd1 0-0 11 b3 and now Black should be fine after
something like 11 ... Qc7 (another way to open the c-file is 11 ... Na5 12 Nc3 b5!? 13
cxb5 Nd5?!, H.Kock-Z.Almasi, Balatonbereny 1993, but here White is better after 14
Rxd5! exd5 15 bxa6) 12 Nc3 d5 13 cxd5 Na5 when Black gets excellent play.
6 ... exd5 7 cxd5 Bb7
8 Bg2
8 e4 defends the d5-pawn, but Black can win the pawn in any case with 8 ... Qe7 9
Bd3 Nxd5 10 a3 (to prevent ... Nb4) 10 ... Nc6 11 0-0 Nc7 12 Nc3 Ne6 13 Be3 Qd8 14
Rad1 Be7 15 Bc4 0-0 16 e5 which was seen in S.Mamedyarov-B.Gelfand, Wijk aan
Zee 2006. Play is similar to the main lines, except the bishop is posted on c4 and not on
g2. Clearly that gives Black more attacking chances along the h1-a8 diagonal, but
White’s active c4-bishop means he can think about attacking also.
8 ... Nxd5
Now we have transposed back into the gambit main line.
9 0-0 Be7 10 Rd1 Nc6 11 a3

Question: Why play 11 a3? It looks very slow.

Answer: Yes this does look rather slow, but by preventing ... Nb4 ideas White can continue with the plan of e2-e4
and Nb1-c3 to control the d5-square. An important part of White’s strategy is to prevent a quick ... d5-d4 when the
extra d-pawn’s presence will be felt.
11 ... Nc7 12 Nc3 0-0 13 e4 d6

A very typical position for this gambit. White needs to try and win the d-pawn back
or elicit some kind of activity fairly quickly to prove compensation for the pawn.
14 b4
An attempt to open the position, but more ‘normal’ moves also are not too
threatening for Black:
a) 14 Bf4 Ne6 15 Be3 Qe8 16 Rd2 Ne5 17 Rad1 Ng4, or
b) 14 Be3 Bf6 15 Rd2 Qe7 16 Rad1 Rad8, and in both cases Black has regrouped
and held the extra pawn.
14 ... cxb4 15 axb4
15 Nd5 Rc8! again there are tactics against the queen on c3, for example 16 axb4
Nxd5 17 exd5 Nxb4 18 Qb3 a5 and Black has two extra pawns.
15 ... Nxb4 16 Qb1 a5 17 Be3 Ne6

18 Nd5
18 e5 is another try, but with care Black can repulse White’s play; 18 ... d5 19 Nd4
Rc8 20 Na4 Nc5 21 Nf5 Nxa4 22 Rxa4 Bc5 and he remains firmly in control.
18 ... Bxd5
18 ... Nxd5 is also possible. After 19 exd5 Nc5 20 Bxc5 dxc5 21 Ne5 Bd6 22 Nc6
Qc7 the two pawn advantage should tell soon.
19 exd5 Nc5 20 h4 Re8 21 Ra3 Bf6 22 Ng5
Finally White musters a threat, which is easily parried.
22 ... g6 23 Bd4 Ra7 24 Rf3 Be5 25 Bh3 Kg7 26 Be3 Bf6

Exercise: Can White play 27 Rxf6 here, followed by 28 Bd4?

Answer: After 27 Rxf6 Black has the counter 27 ... Qxf6 28 Bd4 Re1+! 29 Rxe1 Qxd4 retaining all the trumps
(two extra pawns, safe king, activity). Instead, White played:
27 Kg2 Rxe3
An exchange sacrifice to rebuff any Rxf6 and Be3-d4 ideas such as 27 ... h6 28 Rxf6
Qxf6 29 Bd4 Re5 30 Nf3 which would give White some chances.
28 Rxe3 Bxg5 29 hxg5 Qxg5
With three pawns for the exchange, Adams still holds a material advantage whilst
reducing the opponent’s counterplay.
30 Qb2+ Qf6 31 Qe2
31 Qxf6+ Kxf6 and the extra pawns on the queenside should eventually be enough.
31 ... h5 32 Qb5 Nc2 33 Re8 Nd4 34 Qxb6 Qf3+ 35 Kh2 Qxf2+
35 ... Qxd1 36 Qxd6 actually allows White back into it, with some dangerous threats
beginning with 37 Qf8+. Adams chooses the simple option.
36 Bg2 Nf3+ 37 Kh3
37 ... Ne4!
A brilliant finish, the queen on b6 hangs or Black delivers mate. White chooses the
latter.
38 Qxf2 Nxf2# 0-1

Game 11
Wang Yue-P.Leko
FIDE Grand Prix, Elista 2008

1 d4 Nf6 2 c4 e6 3 Nf3 b6 4 g3 Ba6 5 Qc2 Bb7 6 Bg2 c5 7 d5 exd5 8 cxd5 Nxd5 9 0-0 Be7 10 Rd1 Nc6 11
Qa4

This is probably the main line against 10 ... Nc6. Without wasting time with a2-a3,
the queen moves to take the sting out of ... Nb4 tricks and threatens to swing over to the
kingside in some lines. Also d7 is eyed up.
11 ... Nf6
An example of the power of this variation for White is the following miniature: 11 ...
Nc7 12 Nc3 Qc8 13 Qg4 the queen comes over the kingside to attack, and after 13 ... 0-
0 14 Bf4 f5 15 Qh5 Ne6 16 Nd5 Nxf4 17 Nxf4 Bf6 18 Ng5! Black resigned in
V.Cmilyte-N.Kazimova, European Women’s Championship, Plovdiv 2008, since 18 ...
Bxg5 19 Bd5+ Kh8 20 Ng6# is an attractive finish.
12 e4
12 Nh4 is played just as often:

Question: What is the point of moving the knight to h4?

Answer: To play Nh4-f5, exerting pressure on the key squares g7, d6 and e7. It also opens up the g2-bishop along
the long diagonal, towards the important d5-square. There are a few methods to try and deal with this idea:
a) 12 ... g6 13 Bh6 and now:
a1) 13 ... a6 14 Nc3 b5 15 Qf4 Qb8 16 Qf3 Qe5 17 Bg7 Rg8 18 Rd5 Qe6 19 Bxf6
Qxf6 20 Qxf6 Bxf6 21 Ne4 Be7 22 Nxc5 Bxc5 23 Rxc5 was a better endgame for the
first player in D.Ludwig-M.Adams, Chicago 2012.
a2) 13 ... Bf8 is probably the safest way here. Play may proceed 14 Bxf8 Kxf8 15
Nc3 Kg7 with a further split:
a21) 16 Rd6 Rb8 17 Rad1 Qe7 18 e4 a6 19 R1d2 b5 20 Qd1 b4 21 Ne2 Rhd8 when
Black had consolidated but White still had some pressure in B.Adhiban-S.Narayanan,
Indian U-20 Championship, Chennai 2009.
a22) 16 Qf4 Qe7 17 Qg5 Qe6 18 Nd5 Nxd5 19 Rxd5 h6 20 Qd2 Rad8 21 Rd1 was
T.Nyback-R.Wojtaszek, Warsaw 2009, where 21 ... d6! might have given Black winning
chances, as there is the trick 22 Rxd6 Rxd6 23 Qxd6 Rd8! winning for Black.
b) 12 ... Qc8 13 Nf5 Nd4 is a line some strong 2700s have used.

I am not enthusiastic about it, however, since we are trying to play for a win with
Black and this line is drawish at best in my opinion. White can continue with:
b1) 14 Nxg7+ Kf8 15 Bh6 b5 16 Nf5+ Ke8 17 Ng7+ Kf8 18 Nf5+ with a draw by
repetition.
b2) 14 Nxd4 cxd4 15 Qxd4 0-0 16 Nc3 Re8 17 Qd3 Bxg2 18 Kxg2 Bb4 19 Bd2 and
Black did achieve the draw eventually in P.Leko-E.Bacrot, FIDE Grand Prix, Jermuk
2009, however he had to defend this position for a while.
c) 12 ... 0-0 13 Nf5 d5 14 Nc3 Nd4 15 Nxd4 cxd4 16 Rxd4 Bc5 17 Rd3 Qe8 18 Qh4
Ne4 19 Be3 Nxc3 20 Rxc3 d4 21 Bxd4 Bxd4 22 Qxd4 Bxg2 23 Kxg2 Qxe2 24 Re3 and
the players agreed a draw in a sterile position in E.Najer-Z.Almasi, German League
2011.
Returning to the position in the main game after 11 e4:
12 ... 0-0 13 e5 Ne8 14 Nc3 Nc7 15 Be3 Qe8
15 ... f6 to try and gain some counterplay might be risky – probably the best strategy
is to sit tight and hold onto the extra pawn. 16 Rd2 fxe5 17 Nxe5 Nxe5 18 Bxb7 Rb8 19
Be4 d6 20 Bf4 Nf7 21 Qxa7 Bf6 22 Nd5 was played in P.Gnusarev-D.Bocharov,
Tomsk 2009. It is not completely clear but should be better for White with easier piece
play and control of d5.
16 Rd2 Rd8

17 Re1
17 Rad1 is more natural, and might be better. 17 ... f5 (17 ... h6 sitting tight was
advisable, although Black can also follow up with the same idea as in the game, for
example 18 Qg4 f5!?) 18 exf6 Bxf6 19 Bxc5 bxc5 20 Qb3+ Ne6 (20 ... d5 21 Qxb7
Rf7) 21 Qxb7 Ned4 22 Nxd4 Bxd4 23 Qb3+ Kh8 24 Nb5 d5 25 Nxd4 Nxd4 26 Qa3
Qb5 27 Qxa7 Ra8 28 Qe7 Rae8, as played in V.Akopian-M.Palac, European Club Cup,
Kallithea 2008, gave White a pawn advantage but Black had counterplay with a strong
knight on d4.
17 ... Ba8 18 Ne4 Ne6 19 a3 Kh8 20 Qc2 h6 21 g4 Ncd4
It is easy to see why Leko tried to clarify matters and exchange a few pieces.
Instead, 21 ... Rc8 22 Qd1 Na5 23 Qc2 b5 and holding off on simplifications for a few
more moves keeps the extra pawn, but how much pressure can one take?
22 Nxd4 cxd4 23 Bxd4 Nxd4 24 Rxd4

24 ... d6
The move Leko had to see before playing 21 ... Ncd4. The pin down the e-file
towards the rook on e1 just about lets Black get away with this.
25 Nxd6
25 exd6 Bxd6 26 Rdd1 (26 Red1? Bxh2+! 27 Kxh2 Rxd4 28 Rxd4 Qe5+ wins) 26 ...
Be5 is approximately equal.
25 ... Bxd6 26 Bxa8 Bxe5 27 Rxd8 Qxd8 28 Rxe5 Qxa8 29 Qe4 ½-½
Here the players agreed a draw, a fair result after an interesting skirmish.
So Black has defences against the 5 Qc2, 7 d5 pawn sacrifice. In some variations you may have to sit tight and suffer
a little, but the extra pawn often comes in handy as we saw in several lines.
Chapter Three
Fianchetto Main Line: 4 g3 Ba6 5 b3
Bb7 6 Bg2 Bb4+
1 d4 Nf6 2 c4 e6 3 Nf3 b6 4 g3 Ba6 5 b3

This is the absolute main line. In the last chapter we have seen White try and defend
the c4-pawn with pieces, now White decides to play b2-b3 which can be useful to
prepare Bc1-b2 and put the bishop on the long diagonal.
5 ... Bb7 6 Bg2 Bb4+
Now we see why Black tempted the move 5 b3 so early. White is practically forced
to play 7 Bd2 to block this check, as alternatives only bring problems to the first player.
7 Bd2 a5

This is the start of our repertoire main line. The pawn move 7 ... a5 tries to
discourage the exchange Bd2xb4, as ... axb4 will give Black a strong open a-file and
also prevent the b1-knight ever coming to c3. If White doesn’t exchange on the b4-
square, there is the potential for ... a5-a4 to prise open the queenside to Black’s
advantage – another reason why tempting b2-b3 was advantageous. Whilst both sides
jostle for the e4- and d5-squares (and in particular White will try and get in e2-e4 at
some point) Black has pawn breaks of his own. ... c7-c5 is usually the first one to
consider, and also ... d7-d5 to play in classical fashion. Finally ... e6-e5 may be
possible, if preparations can be made for that advance. In general, the Queen’s Indian
player needs to be patient and await developments out of the opening. A rash move can
be costly.
From the diagram position above, after 8 0-0 0-0, Game 12 covers 9 Bg5, while
White’s main move 9 Nc3 is dealt with in Games 13-15. Finally the less common move
8 a3 is dealt with in Game 16.

Game 12
F.Gheorghiu-R.Hübner
Novi Sad Olympiad 1990

1 d4 Nf6 2 c4 e6 3 Nf3 b6 4 g3 Ba6 5 b3 Bb7




Question: Having just said White might want to play b2-b3 and Bc1-b2, did
we not just give White an extra move by moving our light-squared bishop twice?

Answer: Yes we did, but as discussed in the chapter introduction, 5 b3 is a potentially weakening move which
gives Black some extra options too. One of them being the main line I am recommending here.
6 Bg2 Bb4+ 7 Bd2
Not the square the bishop wanted to go to (it had its eye on b2) but there wasn’t
much choice. Not 7 Nbd2 Bc3 8 Rb1 Bxd4 winning a pawn.
7 ... a5
This is the line I will be recommending against 5 b3. It is solid and gives Black
plenty of winning chances.
8 0-0 0-0 9 Bg5


Question: What is the point behind the move 9 Bg5?

Answer: To try and leave the b4-bishop stranded, and to pin the f6-knight so White can fight for the key d5- and
e4-squares. However there is a counter trick here ...
9 ... a4! 10 Nbd2
White’s alternatives are:
a) 10 a3 axb3 11 Qxb3 Be7 and now the a3-pawn will always be a long term target
down the a-file.
b) 10 bxa4 wins a pawn but this is hardly likely to matter in the short term as the a-
pawns are doubled and will be blockaded. Now 10 ... d6 11 Qb3 Ba5 12 Na3 Nbd7 13
Rad1 h6 14 Bc1 Qe7 15 Nh4 Bxg2 16 Kxg2 Rfc8 17 Nc2 Qe8 18 Nf3 Ne4 19 Rd3 c5
20 d5 exd5 21 Rxd5 Ndf6 22 Rd3 d5 gave Black active play in V.Babula-T.Petrik,
Banska Stiavnica 2012.
10 ... h6 11 Bxf6 Qxf6 12 Qc2
12 bxa4 Bc3 13 Rc1 Bxd4 14 Nxd4 Bxg2 15 Kxg2 Qxd4 is clearly better for Black
– just compare the relative weaknesses of the pawns.
12 ... c5

13 Ne4 Qe7 14 dxc5 bxc5 15 Nc3
15 bxa4 d5 16 cxd5 exd5 17 Ned2 Nc6 is excellent for Black, the a4-pawn is hardly
running away and the two bishops combined with Black’s central pawn mass far
outweigh White’s extra (doubled) a-pawn.
15 ... axb3 16 axb3 Rxa1 17 Rxa1 Qf6 18 Rc1 Bxc3
18 ... Nc6 was also good, but the German Grandmaster saw his chance to reach a
favourable ending.
19 Qxc3 Qxc3 20 Rxc3 Nc6


Question: Am I supposed to shake hands on a draw here?

Answer: No! This position is more difficult for White than it first appears. The b3-pawn is a permanent weakness,
especially as it stands on an open file. Black also holds a potentially good central pawn majority, and can play for a
win here with very little risk. I recommend playing through this endgame to learn some endgame technique from
Robert Hübner.
21 Rd3 Rd8 22 Nd2 Kf8
The king will be very useful on e7.
23 Ne4 Ne5 24 Rd1 Bxe4 25 Bxe4 f5 26 Bg2

26 ... Rb8
26 ... Ke7 was also just good, protecting the d7-pawn and allowing the rook to come
to b8.
27 f4 Ng4 28 Rd3
28 Rxd7 Rxb3 29 Bf3 Rb1+ 30 Kg2 Ne3+ 31 Kf2 Nxc4 32 Rc7 might have been the
chance to try and draw.
28 ... Nf6 29 Kf2 Ke7
Returning to a similar position we might have reached after 26 ... Ke7.
30 Bf3 g5 31 h3 Rb6 32 e3 d6 33 Rc3 e5
The advantage is growing. Black has active plans, whereas White must just sit and
wait.
34 g4 fxg4 35 fxg5 hxg5 36 hxg4 e4

37 Be2 Nd7 38 Kg3 Rb8 39 Bd1 Rh8 40 Rc2 Rh1 41 Rd2 Ne5 42 Kg2 Rh4 43
Kg3 Ke6 44 Be2 Rh8 45 Bd1 Ra8 46 Bc2 Ra2 47 Rf2 Nd3 48 Re2
48 Bxd3 Rxf2 49 Kxf2 exd3 is an easily winning pawn ending, for example50 Ke1
Ke5 51 Kd1 Ke4 52 Kd2 Kf3 53 Kxd3 Kxg4 is decisive.
48 ... Nb4 49 Bd1 Ra1 50 Rd2 Ke5 51 Be2 Rb1 52 Bd1 Nd3 53 Kg2 Rc1 54 Be2
Rc3 55 Bd1 Nb4 56 Kf2 d5 57 cxd5 Nxd5 58 Re2 Rc1 59 Rd2 Rc3 60 Re2 Rc1 61
Rd2 Rb1 62 Be2

62 ... Nxe3! 0-1


Having seen enough, White threw in the towel.
Game 13
I.Morovic Fernandez-M.Adams
Istanbul Olympiad 2000

1 d4 Nf6 2 c4 e6 3 Nf3 b6 4 g3 Ba6 5 b3 Bb7 6 Bg2 Bb4+ 7 Bd2 a5 8 0-0 0-0 9 Qc2
White plays a natural move, fighting for the e4-square.

9 ... d6
Others:
a) 9 ... d5 to follow up with ... c5, and potentially creating hanging pawns, was fine
also. Then 10 cxd5 exd5 11 Nc3 Re8 12 Rad1 Ne4 13 Nxe4 dxe4 14 Ne5 f6 15 Bxb4
axb4 16 Nc4 Na6 was played in E.Magerramov-I.Ibragimov, Dubai 2000, which was at
least for Black.
b) 9 ... h6 might be the move for you if you are worried about Bd2-g5 ideas. Play
could go 10 Rd1 Re8 11 Nc3 d6 when a typical position was reached in A.Beliavsky-
J.Timman, Manila Olympiad 1992. Now Beliavsky went for a thematic idea which was
met with a cool riposte. Following 12 d5 exd5 13 Nh4 (13 Nd4 might have been
better), if White could get in cxd5, e2-e4 and Nh4-f5 he would have an excellent
position. Instead, Timman demonstrated Black’s counter-chances with the nice 13 ...
Nc6! 14 Nf5 (14 Nxd5 Nd4 15 Qd3 Nxe2+ 16 Kh1 Nxd5 17 cxd5 Qf6 leaves Black a
clear pawn to the good) 14 ... dxc4 15 bxc4 Bc5 16 Qb2 Re5 17 e4 Ng4 18 Be1 Ne7
19 Nd4 Qd7 20 Nb3 Re8 21 Nd5 Black was still a pawn up though the position was
now getting unclear.
10 Nc3
Instead, White could play 10 Bg5 trying to play like the previous game (Gheorghiu-
Hübner) and leave the b4-bishop out of play:


Exercise: Can you remember the trick Hübner played?

Answer: 10 ... a4! Following 11 Nc3 (11 a3 axb3 12 Qxb3 Ba5 saves the bishop) 11 ... Bxc3 12 Qxc3 Nbd7 13 b4
h6 14 Bc1 Black has 14 ... b5! which is an excellent pawn sacrifice to gain control of the d5-square. V.Anand-
M.Adams, Madrid 1998 then continued 15 cxb5 Nb6 16 Ba3 Rc8 17 Nd2 Bxg2 18 Kxg2 Nfd5 19 Qd3 f5 Black slows
down the e2-e4 push, and goes for kingside attacking play. 20 Rac1 Qd7 21 e3 Rb8 22 f3 f4 23 gxf4 Rxf4 24 e4 Nf6
25 Kh1 Rf8 26 Rg1 and now Black should have played 26 ... d5 27 e5 Nh7! and with ... Ng5 and ... Nc4 to follow, the
f3-pawn might be caving in.
Returning to the main game after 10 Nc3:
10 ... Nbd7 11 Rad1
11 Rfe1 is the subject of the next game.

11 ... Bxc3

Question: Why are we voluntarily giving up the bishop like this?

Answer: Adams’ next move will highlight the strategic thinking behind this capture on c3.
11 ... Qb8!? and delaying taking on c3 was possible. Here the queen eyes up the
queenside and potential ... b5 or ... a4 pawn breaks. Again we see White’s b2-b3 move
early on has deprived him of possibilities. With the pawn on b2, White could play a2-
a3 and question the bishop. Here of course this is not possible.
12 Bxc3 Be4!
Control of the centre in any opening is crucial, but the e4-square is very important in
the Queen’s Indian. The bishop cannot be easily ousted. To do so, White would either
need to move the f3-knight and exchange light-squared bishops, or play Bg2-h3 and then
Nf3-d2 to attack the bishop on e4.
13 Qc1
Now the plans of both sides are clear: White will play in the centre, trying to use his
two bishops. Black will play ... a4 and ... b5, after what he will get one or more open
lines on queenside.
13 ... a4 14 Bh3
This strange looking move has a strong point to it. As mentioned above, White plans
Nf3-d2 without allowing the exchange of bishops. Of course, this is time consuming.
Alternatively, 14 b4 b5 15 cxb5 Qb8 gives Black play on the queenside.
14 ... b5
14 ... Bxf3!? 15 exf3 d5 trying to blunt both bishops is another plan, though is more
solid.

15 Nd2
After 15 cxb5 axb3 16 axb3 Qb8 Black regains the pawn with advantage, as now
there is the b3-weakness to attack.
15 ... axb3 16 axb3 bxc4 17 bxc4
17 Nxe4 Nxe4 18 bxc4 Ra4! 19 Bb2 Nb6 20 c5 Nc4 is quite annoying, though White
can try to hold. Not really what he was expecting with the white pieces within 20
moves!
17 ... Ra2 18 f3?
18 Nxe4 Nxe4 19 Bg2 Nxc3 20 Qxc3 Rxe2 wins a pawn, but the computer thinks
Black will be hard pressed to convert this. Still, it is not a bad situation to be in from
Black’s point of view.
18 ... Bc2
Possibly White underestimated this move, which gains time for Black to strike in the
centre with our typical pawn breaks.
19 Rde1


Exercise: Which pawn advance should Black
go for here: a) 19 ... e5, b) 19 ... d5 or c) 19 ... c5?

Answer:
19 ... c5!
I think attacking the d4-pawn is the way to go, so in fact 19 ... e5 is also good.
Others:
a) 19 ... e5 20 Rf2 Qe7 with the ideas of attacking the e2-pawn or playing the f8-
rook to the queenside.
b) 19 ... d5 20 Rf2 c5 21 cxd5 Nxd5 22 Bb2 Ne3 is also playable, with nice activity
for Black. So maybe the answer is that all three pawn moves are profitable, but 19 ...
c5 is the most natural for a Nimzo/Queen’s Indian player.
20 d5
After 20 e3 Qc7 it is not clear what White can do to improve his position.
20 ... exd5 21 Bxd7 Nxd7
21 ... Qxd7 22 Bxf6 gxf6 23 cxd5 is not really the human way to play as the open
kingside can offer counter-chances.
22 cxd5 Qa8!

23 Bb2
23 e4 Bd3 24 Rf2 Rc2 25 Qa1 Qxa1 26 Bxa1 f5! 27 e5 dxe5 28 Bxe5 Nxe5 29 Rxe5
c4 and the dangerously advanced passed c-pawn, combined with his more active
forces, gives Black the better chances here.
23 ... Qa4
23 ... Rb8 bringing the remaining rook into the action was also playable, for
example:
a) 24 Nc4 Qa4 25 Qg5 Bg6 and now the c4-knight and b2-bishop cannot be
defended well enough.
b) 24 Qxc2 Rbxb2 25 Qf5 Nb6 26 Ne4 Qxd5 27 Qxd5 Nxd5 28 Nxd6 Ne3 29 Rf2 f5
30 f4 Rd2 31 Nb5 c4 and it is clear that the c-pawn far outweighs the e2-pawn.
24 Bc3 Nb6 25 e4 Bd3 26 Rf2 Rc2 27 Qa1 Qxa1 28 Bxa1 Ra8 29 Nb3?
29 Nf1 Rxf2 30 Kxf2 Nc4 with only an advantage to Black. The game continuation is
a mistake, but this is understandable since he has been under pressure all game.
29 ... Nc4
Surprisingly it is very difficult for the first player to hold this position. His minor
pieces are in a bit of a jam defending each other.
30 e5
30 Nc1 Bxe4 31 Bxg7 Rxf2 32 Kxf2 Bxd5 was maybe the last chance to defend, but
it is still unpleasant.
30 ... Rxf2 31 Kxf2 Bc2
Now it is game over, the b3-knight suffers a quick demise.
32 Nd2 Nxd2 33 exd6 Nc4 34 d7 Ba4
Preventing the last trick of 35 Re8+.
35 Rc1 Bb5 0-1

Game 14
L.Aronian-S.Agdestein
Norway Chess, Stavanger 2014

1 d4 Nf6 2 c4 e6 3 Nf3 b6 4 g3 Ba6 5 b3 Bb7 6 Bg2 Bb4+ 7 Bd2 a5 8 Nc3 0-0 9 0-0 d6 10 Qc2 Nbd7 11
Rfe1

The previous game saw White play 11 Rad1, while here there is a threat of the
immediate 12 e4.

Exercise: How can Black counter White’s plan?

Answer: By now (hopefully!) the following two-move plan should be clear.
11 ... Bxc3 12 Bxc3 Be4!
I cannot stress enough the importance of occupying the e4-square like this when
required.
13 Qb2
In this position Black has a number of plans, so while the main game continues with
13 ... c6, it is worthwhile analysing the alternative ideas.
13 ... c6

Question: What is the point behind this? You’ve been
mentioning the ... c5 pawn break in most games.

Answer: Since the queen on b2 and the c3-bishop are pointing towards g7, Black can change tack and play for ...
c6 and ... b5 to threaten ... b4, forcing the bishop back. If Black refrains from ... c5 or ... e5 over the next few moves,
White’s queen and bishop are not doing anything useful. Alternatives for Black on move 13 are as follows:
a) 13 ... Re8 14 Bh3 Bb7 15 Nd2 e5 16 Rad1 h6 17 f3 exd4 18 Bxd4 Ne5 with some
active play, was played in T.Radjabov-V.Ivanchuk, Sofia 2008.
b) 13 ... Qb8 14 Bf1 c5 15 Rad1 Rd8 16 Bh3 a4 17 Nd2 axb3 18 axb3 Bb7 19 d5
was seen in L.Van Wely-A.Karpov, Tilburg 1996, and now instead of Karpov’s 19 ... e5
allowing White a big space advantage, he should have played 19 ... exd5 20 Bxd7 Rxd7
21 Bxf6 gxf6 22 cxd5 Bxd5 23 Qxf6 Qd8 24 Qf4 b5 to maintain equality.
c) 13 ... a4 14 Bf1 c5 15 Nd2 Bb7 16 b4 a3 17 Qb3 cxd4 18 Bxd4 e5 19 Be3 Qc7
20 Rac1 Qc6 21 f3 Qa4 22 Qb1 d5 gave Black a nice initiative with active central play
in V.Topalov-M.Adams, Frankfurt (rapid) 2000.
d) 13 ... c5 is probably OK, for example 14 Rad1 Qc7 15 Rd2 h6 16 Red1 d5 though
I can’t help but feel that opening up the position will end up suiting White’s two
bishops.

14 Rac1
Grandmaster Zoltan Almasi has played the position after 14 Bf1 b5 a few times,
with success:
a) 15 d5 cxd5 16 Bxf6 Nxf6 17 cxb5 a4 18 bxa4 Rxa4 19 Rec1 Qa5 20 a3 Rb8 with
queenside pressure was played in M.Sjoberg-Z.Almasi, Malmo 1994.
b) 15 Ng5 Bg6 16 Bg2 d5 17 c5 h6 18 Nf3 Ne4 19 Bd2 Nxd2 20 Qxd2 with a
further split:
b1) 20 ... Qf6 21 Rac1 a4 22 b4 e5 23 Nxe5 Nxe5 24 dxe5 Qxe5 25 a3 and the
players agreed to a draw in J.Pinter-Z.Almasi, Hungarian Championship, Budapest
1997.
b2) 20 ... a4 21 Qc3 Re8 22 b4 a3 23 Nd2 Qc7 24 e4 dxe4 25 Nxe4 Bxe4 26 Bxe4
Nf6 with balanced chances in P.Kiss-Z.Almasi, Hungarian League 1998.
14 ... h6 15 Bf1 b5
We have seen that this move is consistent with the earlier ... c6.

16 Bd2
16 Nd2 is thematic, to try and play e2-e4 at some point but it allows 16 ... Bg6 (16
... b4 17 Nxe4 Nxe4 18 Bd2 f5 19 Bg2 is also playable, though I don’t like giving up
the e4-bishop so easily) 17 a3 Qb6 18 b4 axb4 19 axb4 d5 20 c5 Qb7 with an
acceptable position.
16 ... bxc4 17 bxc4 Re8 18 Red1 Qc7
Notice that over the past few moves Black has not committed to anything, so it is
hard for White to come up with a plan of action. Which pawn break is Black going for?
White is kept guessing.
19 Qa3 c5 20 dxc5 dxc5
The former Norwegian International footballer decides to prevent Nd4-b5 and plays
the solid option against his esteemed opponent. 20 ... Nxc5 21 Nd4 Qb6 22 Be3 Red8
23 Nb5 Rac8 should be fine in any case for Black, but it is understandable Agdestein
didn’t want to allow too much play.
21 Ne1 Bb7 22 f3 e5 23 e4


Exercise: What is the key positional idea for both players here?

Answer: Well done if you spotted the outposts on the d5-square (for White) and on the d4-square (for Black).
Knights like these squares, so both sides head immediately for them.
23 ... Nf8 24 Ng2 Ne6 25 Ne3 Nd4 26 Nd5 Qd6 27 Kg2 Nd7 28 Rb1 Reb8 29 Kf2
Bc6 30 Bc3
Allowing a thematic exchange sacrifice which gives Black plenty of play. Instead,
30 Rxb8+ was better, but after 30 ... Rxb8 White should not grab the a-pawn with 31
Bxa5, since after 31 ... Ra8 32 Qc3 f5 Black takes over the initiative.
30 ... Rb4! 31 Bxb4 axb4 32 Qb2 Bxd5 33 cxd5 Nb6
It is easy to look just at the queenside for play, but in fact our silicon friend kindly
points out that Black can attack from the other direction beginning with 33 ... f5 34 Bd3
Ra3 and it is difficult to see a defence for White here.
34 Rbc1 Na4 35 Qd2 Nc3
36 Rxc3
Aronian reasons that he has to give up an exchange anyway, and he does it in the best
way. 36 Ra1 Qf6! is crushing as ... Qxf3 and ... Nxe4 are both on the cards, for example
37 Re1 Nxe4+ 38 Rxe4 Qxf3+ 39 Kg1 Qxe4 and Black wins.
36 ... bxc3 37 Qxc3 Rxa2+ 38 Rd2 Rxd2+ 39 Qxd2 Qb6 40 Bc4 g5 41 Kg2 ½-½
A very good draw from the Black perspective, as world number two Levon Aronian
was not able to prove an advantage against our line beginning with 13 ... c6.

Game 15
D.Palo-S.Brynell
Aarhus 2014

1 d4 e6 2 c4 Nf6 3 Nf3 b6 4 g3 Ba6 5 b3 Bb7 6 Bg2 Bb4+ 7 Bd2 a5 8 0-0 0-0 9 Nc3
d6 10 d5
The standard d4-d5 thrust can surprisingly even be played in a position like this,
where the d2-bishop hinders the queen’s influence on d5. This move has rarely been
played but it has a lot of positives behind it, as this game shows.
10 ... e5
Perhaps Black should explore the alternatives here:
a) 10 ... exd5 11 Nxd5 Bxd2 12 Qxd2 Nc6 13 Nd4 Nxd4 14 Qxd4 Rb8 15 Rac1 Re8
16 e3 with a solid position for Black, was played in A.Livner-S.Brynell, Norrkoeping
2008.
b) 10 ... Na6 11 Nd4 Nc5!? is an interesting pawn sacrifice that has not yet been
tried in practice. After 12 dxe6 Bxg2 13 exf7+ Rxf7 14 Kxg2 d5 there is some
compensation for the pawn, for example 15 cxd5 Nxd5 16 Rc1 Bxc3 17 Bxc3 Ne4! 18
Be1 (18 Bb2 Rxf2+ wins) 18 ... Qf6 and Black has attacking play down the f-file.
c) 10 ... Bc5 and 10 ... Re8, keeping the pawn structure flexible, are worth
investigating.
11 Qc2
11 Ne1 is quite promising as well. 11 ... Nbd7 (11 ... c6!? may be worth a try) 12
Nd3 Bxc3 13 Bxc3 Qe7 14 e4 a4 15 b4 c6 16 b5 cxd5 17 cxd5 Rfc8 18 Bb4 Nc5 19
Nxc5 bxc5 20 Bc3 Rcb8 21 Rb1 Bc8 22 Re1 Nd7 23 f4 f6 24 Bf1 was S.Safin-
E.Pileckis, Moscow 2008, where Black is solid but the two bishops give White hopes
for an edge. This line resembles a Bogo-Indian (3 Nf3 Bb4+) rather than the Queen’s
Indian (3 ... b6).
11 ... Nbd7
11 ... c6, aiming for active play, may be better than agreeing to the fixed pawn
structure that occurs in the game. Black eventually plays ... c6 in the game (see move
17), but by that stage White’s forces are better developed.
12 Nb5
Alternatively, White could kick the b4-bishop with 12 a3, but after 12 ... Bxc3 13
Bxc3 a4 14 bxa4 (14 b4 b5! 15 cxb5 Bxd5 16 Rac1 Qb8 gives Black good play) 14 ...
Nc5 15 a5 bxa5 16 Rab1 Nfe4 17 Be1 f5 Black is fine.
12 ... Bxd2 13 Nxd2 Re8 14 a3 Nf8 15 b4 Qd7 16 Nc3 axb4?!
Playing something non-committal like 16 ... h6 rather than opening the a-file
immediately was preferable.
17 axb4 c6 18 Nde4 Nxe4 19 dxc6 Bxc6 20 Bxe4 h6 21 b5 Bxe4 22 Qxe4
After allowing the position to be opened up, Black is in much more danger than it
first appears. The d5- and e4-squares are firmly under White’s control, and there is a
decision to be made about the a-file since the a8-rook is being attacked. Finally, there
are a few weak pawns that can be targeted (on b6 and d6).
22 ... Rxa1 23 Rxa1 Nh7


Exercise: What should White play here: a) 24 Ra6, b) 24 Qc6 or c) 24 Nd5?

Answer: All are good and give an advantage, but the queen invasion poses the most problems.
24 Qc6
After the alternatives 24 Nd5 Nf6 25 Nxf6+ gxf6 26 Ra6 Rb8 and 24 Ra6 Nf6 25
Qc6 Rc8 26 Qxd7 Nxd7, Black still has a grim position in both cases, but he can try to
defend.
24 ... Qxc6 25 bxc6 Nf6
25 ... Rc8 26 Nd5 Kf8 27 Ra7 Nf6 28 Ne7 Re8 29 c7 and White queens the c-pawn.
26 Nd5! 1-0
It looks a bit premature to resign, but after 26 ... Nxd5 27 cxd5 White has a
technically winning position. He can bring the king to b5 and win the b6-pawn, or
infiltrate down the a-file immediately and win either the b6- or d6-pawns. Black
decided to call it a day rather than face long arduous torture here.

Game 16
A.Oliveira-I.Ibragimov
Lisbon 2000

1 Nf3 Nf6 2 d4 e6 3 c4 b6 4 g3 Ba6 5 b3 Bb7 6 Bg2 Bb4+ 7 Bd2 a5 8 a3



Here White decides to immediately resolve the situation with the b4-bishop, at the
cost of a tempo.
8 ... Be7
This is my recommendation, keep the pieces on in order to maintain winning
chances.
8 ... Bxd2+ 9 Qxd2 0-0 10 Nc3 Ne4 11 Nxe4 Bxe4 looks thematic, though Black
should be careful as he has played this ... Ne4 idea quite early. After 12 Qf4 f5 13 0-0
d6 14 Ng5 Bxg2 15 Kxg2 Qf6 the position looks fine for Black, but I was impressed by
Dimitry Andreikin’s handling of White’s side in the following game. Following 16 d5
e5 17 Qh4 h6 18 Ne6 Qxh4 19 gxh4 Rf7 20 f4 exf4 (20 ... e4 21 b4 keeps the initiative)
21 Rxf4 Na6 22 Raf1 Nc5 23 e4 g6 24 h5 Nxe6 25 dxe6 Rg7 26 Kh3 it is now tough
for Black to defend as the kingside pawns are falling. D.Andreikin-E.Gasanov, Minsk
2006 concluded 26 ... Re8 27 hxg6 fxe4 28 Rf7 Kh8 29 R1f6 Reg8 30 e7 e3 31 Re6
Re8 32 Rf8+ Rg8 33 Rxe8 Rxe8 34 Kg3 Kg7 35 Kf3 h5 36 h4 1-0.
9 Nc3 0-0 10 0-0 d5 11 Ne5
11 ... Na6

Question: Why play this, rather than challenge the e5-knight with ... Nbd7?

Answer: There is nothing wrong with that, as the following variation demonstrates. 11 ... Na6 looks odd, but has
some plus points: it allows for a ... c5 pawn break, prevents b3-b4 and once ... c5 is played, the knight can return to
the centre with ... Na6-c7. From there, it defends the d5-pawn and can come to the nice e6-square.
After 11 ... Nbd7 play can continue 12 Bf4 c5 13 cxd5 Nxd5 14 Nxd5 exd5 15 dxc5
Nxe5 16 Bxe5 bxc5 17 e3 Qd7 18 a4 Rfd8 19 Rc1 Qe6 20 Ba1 Bf8 21 Qc2 Rd7 22
Rfd1 Rad8 23 Rd2 Ba6 24 Bc3 Qb6 25 Rcd1 and all of Black’s moves are geared up to
the following pawn break: 25 ... d4! 26 exd4 cxd4 27 Qf5 Bc5 28 Qf4 Rd6 29 Ba1
Qxb3 winning a pawn and eventually the game in J.Gonzales-Zhang Zhong, Singapore
2007.
12 cxd5 exd5 13 Bg5 c5 14 e3 Nc7 15 Qd2 Re8 16 Rfd1
Play has been quite cagey from both sides.
16 ... cxd4 17 exd4
Instead 17 Qxd4 Black can play 17 ... Ne6, the manoeuvre I mentioned in the notes to
11 ... Na6 above. Then if 18 Qh4 Qc7! attacks both of White’s knights.
17 ... Ne4 18 Nxe4 dxe4 19 Be3
19 Bxe7 Qxe7 20 Rac1 Rad8 gives good play against the isolated d4-pawn, and the
d5-square in front of it is firmly under Black’s control.
19 ... Bf8 20 Bf1 f6 21 Nc4
Or 21 Bc4+ Nd5 22 Ng4 h5 traps the knight.
21 ... Nd5 22 Qb2 b5 23 Nd2 Bc6
23 ... Qd7 is also good. Watch how the Grandmaster slowly outplays his opponent
from here.
24 Rdc1 Qd7 25 Nb1 b4
Black prevents Nb1-c3.
26 Bc4 Kh8 27 Bxd5 Bxd5 28 a4 Bf7 29 Nd2 Bh5 30 Rc4 Be2 31 Rcc1 Qh3 32
Kh1 Rad8 0-1
White can only sit and wait for ideas of ... Rd5-h5 or ... Bf3+ with a mating attack.
Overall, I feel this variation with 6 ... Bb4+ 7 Bd2 a5 has a solid foundation but provides plenty of winning chances.
Perfect I believe to add to your Queen’s Indian repertoire!
Chapter Four
Fianchetto Variation: 4 g3 Ba6 5 b3 b5
1 d4 Nf6 2 c4 e6 3 Nf3 b6 4 g3 Ba6 5 b3 b5


Question: This move 5 ... b5 seems a little odd! You haven’t
mentioned this move before as a way to attack the centre.

Answer: This move unbalances the position early and is a good way to play for a win. If you want something
more solid, Chapter Three is the way to go. However, such an approach may not always work against an opponent
who is willing to sit tight. Here, Black immediately offers to exchange the b-pawn for the more central c-pawn. The
argument is that the loss of time is not so crucial given that the centre is quite closed. If the c- and b-pawns are
exchanged, as they are in some lines, then Black can even go for an early ... a5-a4 (as also sometimes happened in
Chapter Three) to instigate early play on the queenside. Care is need here though, in case the centre gets opened up
and Black’s lack of development ever gets highlighted.
The material is structured as follows. The natural 6 cxb5 Bxb5 7 Bg2 is covered in
Games 17-20, when Black’s main replies are 7 ... Bc6 (Games 17-18), 7 ... d5 (Game
19) and 7 ... Bb4+ (Game 20). While the games with 7 ... Bc6 illustrate some thematic
ideas, White’s 9 a3 idea in Game 18 currently looks quite dangerous. Meanwhile 7 ...
d5 is a solid choice and 7 ... Bb4+ leads to more adventurous positions if White takes
up the challenge. The somewhat surprising 6 Bg2, allowing 6 ... bxc4 is dealt with in
Games 21-23. Finally, 6 Nbd2 is the subject of Games 24-25, and 6 Qc2 is seen in
Game 26.
Game 17
M.Grabarczyk-Y.Yakovich
Bad Wiessee 1999

1 d4 Nf6 2 c4 e6 3 Nf3 b6 4 g3 Ba6 5 b3 b5 6 cxb5


Not the only move of course, but the obvious one.
6 ... Bxb5 7 Bg2

7 ... Bc6
This natural move is the subject of this and the following game. Alternatives that
will be examined later in this chapter are 7 ... d5 (Game 19) and 7 ... Bb4+ (Game 20).
8 0-0 a5

Black instigates queenside play with the intention to play ... a4 quickly. Black’s
structure is compact, and by exchanging the black a-pawn for the white a-pawn (or b-
pawn) will give a future target to aim at. Note the important e4- and d5-squares are in
Black’s control for now (and when are they not important?) so White is in hardly any
position to counter quickly.
9 Nc3
9 a3 to prevent 9 ... Bb4 will be examined in the next game.
9 ... Bb4 10 Bb2 0-0 11 Ne5
11 Qc2 Qc8 12 Ne5 Bxg2 13 Kxg2 Qb7+ 14 Kg1 d6 15 Nf3 Bxc3 16 Bxc3 was
played in J.Horvath-M.Nemeth, Hungarian League 2002 and now maybe 16 ... Nbd7 17
Rfc1 Rfc8 18 Nd2 c5 19 dxc5 Rxc5 gives Black something to work with on the
queenside.
11 ... Bxg2 12 Kxg2 Bxc3
12 ... Qc8 13 Na4 might have been the reason for the early exchange on c3. 13 ...
Qb7+ 14 f3 Be7 15 Rc1 d6 16 Nd3 Nbd7 17 e4 Rfc8 18 Rf2 Nb6 though I might prefer
White as he has e4 in and it is not easy to get in the ... c5 counter break.
13 Bxc3 Qc8


Question: What is Black’s idea here?

Answer: to bring the queen to the nice b7- or a6-square, where it will eye up White’s queenside as well as the
king on g2.
14 f3 Qa6 15 Be1
15 e4 was far more to the point. 15 ... d5 16 Qc2 Nbd7 looks fine, but could go
wrong quickly if Black is not careful. After, for example 17 Nxd7 Nxd7 18 exd5 exd5
19 Rfe1 Black should play on the queenside with 19 ... a4, as swapping rooks with 19
... Rfe8 would give White the better endgame after 20 Rxe8+ Rxe8 21 Re1 Rxe1 22
Bxe1 c6 23 a4! intending to target the a5-pawn as soon as possible.
15 ... Nd5 16 Bf2 f5 17 Qd2
17 e4 fxe4 18 fxe4 Nf6 19 Qc2 d6 20 Nc4 Nbd7 21 Rae1 Qc6 and compared to 15
e4 in the note above, this is a far rosier position from the Black perspective.
17 ... d6 18 Nd3 Qb7 19 Rfc1 Nd7 20 Rc2 N7f6 21 Rac1 a4 22 b4


22 ... a3!
Preventing a2-a3 and therefore giving rise to a queenside target – the b4-pawn.
23 Rc6 Ra6 24 R6c4 Rb6 25 Kg1 h6 26 Qe1 Rb8 27 Qd2 Qa6 28 Kg2
It is clear White has no plan and is just shuffling around. He cannot get in e2-e4
easily, so must sit and wait.
28 ... Qa4 29 Be1 Qe8 30 Bf2

30 ... g5
This looks like one of those ‘going for it’ type moves, but it gains space and keeps
White guessing on whether he might play ... f4 or ... g4 to attack.
31 Rb1 Rc6 32 Qc2 Rxc4 33 Qxc4 Qa4 34 Qc1 Kg7 35 Rb3 Ra8 36 Kg1 Qa6 37
Kg2 Ra7 38 Qc2 Qb5 39 Qc1 Qa6 40 Qc2 Qb7 41 Qc1 Qa8 42 Kg1 Ra6 43 Kg2
Kg6 44 h3 Kg7 45 Bg1 Kg6 46 Bf2 Nh5
No draw!
47 b5 Ra4 48 Qb1 Nhf6 49 Qc1 Qb7 50 Rxa3?! Qxb5 51 Rxa4 Qxa4 52 Qb2
Qd1
After the queen infiltration, surprisingly this position is very tricky for White. He
still cannot play e2–e4 and he cannot get the a-pawn moving fast either.

53 Qc1?
53 Bg1? doesn’t solve much. Black can continue 53 ... Nh5 54 Bf2 f4 55 g4 Ng3 56
Bxg3 fxg3 and the threat of ... Ne3+ is deadly. 53 Ne1 was more tenacious, though
Black is still better.
53 ... Qxe2 54 Nb2 g4 55 hxg4 fxg4 56 f4 Ne4 57 Qe1 Qf3+ 58 Kg1 Ndc3 0-1

Game 18
R.Markus-E.Postny
European Championship, Dresden 2007

1 d4 Nf6 2 c4 e6 3 Nf3 b6 4 g3 Ba6 5 b3 b5 6 cxb5 Bxb5 7 Bg2 Bc6 8 0-0 a5 9 a3



Question: How is this 9 a3 move useful – doesn’t this just
weaken the b3-pawn, which is what Black was aiming for?

Answer: Yes it does, but only a little bit. In the meantime ... Bb4 is prevented and so the b1-knight can come to its
ideal square c3, where it controls the squares e4 and d5. If White can get in e2-e4 and d4-d5 fairly rapidly, the
apparent weakness of the b3-pawn won’t be felt so much.
9 ... Be7
9 ... Qc8 10 Nc3 should lead to the game continuation, but White can also play 10
Nbd2 Be7 11 Qc2 Qb7 12 Re1 when Black has:
a) 12 ... 0-0 13 e4 challenging Black’s strategy of controlling the e4-square, is
exactly what White wants.
b) 12 ... Ne4 is the typical way to prevent e2-e4, however with the knight on d2 and
not c3 there are extra options such as 13 Ne5 Nxd2 14 Nxc6 Qxb3 15 Qxd2 Nxc6 16
d5! with excellent compensation for the pawn. The bishop will come to b2 and rake
across the board.
c) 12 ... Bxf3 13 Bxf3 d5 14 e4 c6 15 Bb2 0-0 16 Rac1 Ra6 17 h4 h6 18 Qd3 Rd8
19 Bd1 dxe4 20 Nxe4 Nbd7 21 Nd2 c5 22 Bf3 Nd5? 23 Bxd5 1-0 was the abrupt end
to the game Z.Varga-F.Jenni, Mitropa Cup, Charleville 2000.
10 Nc3 Qc8 11 Qc2 Qb7
We already saw this plan in the previous game – from here b3 is monitored, as well
as e4.
12 Bg5
White has also tried 12 Re1 Ne4 13 Ne5 Nxc3 14 Nxc6 and now:
a) 14 ... Nxc6 15 Qxc3 Rb8 16 Rb1 0-0 and despite Black’s pressure down the b-
file, White is a little better due to his two bishops and pressure on the c6-knight.
b) 14 ... Nxe2+ 15 Qxe2 dxc6 16 Qc2 0-0 17 Be4 h6 18 Bf4 Bf6 19 Be5 Bxe5 20
dxe5 Ra6 21 Rac1 Rb6 22 Re3 was played in M.Vasic-M.Urosevic, Serbian Women’s
Championship, Budva 2004, which saw Black pretty tied up despite her pawn
advantage.
12 ... h6 13 Bxf6 Bxf6 14 e4 0-0
15 Rab1

Question: Isn’t this just passive? Rooks tend to belong in the centre after all.

Answer: Here White’s plan is to play b4–b5 rather than Ra1-d1 and then push the d-pawn. Both are effective
plans to increase the pressure.
15 Rad1 was an alternative, to try and force through d4-d5 more quickly. After 15 ...
Ra6 16 Rfe1 Rb6 17 d5 Bxc3 18 Qxc3 exd5 19 Nd4 White has definite compensation.
Nd4-f5 to attack g7 is also on the cards. If now 19 ... dxe4 20 Nf5 f6 21 Qc4+ Kh7 22
Bxe4 and White has the initiative.
15 ... Ra6?!
This plays into White’s hands. 15 ... d6 was preferable, when White is only slightly
better.
16 b4 axb4 17 axb4 Ra3
17 ... Rb6 is met by the tactic 18 b5 Bxb5 19 Nxb5 Rxb5 20 e5 as 20 ... Be7 21 Ng5!
which is another useful motif to know.
18 b5 Rxc3
Forced otherwise the c6-bishop drops.
19 Qxc3 Bxe4 20 Ne1!
Exchanging off the strong bishop on e4.
20 ... Rc8 21 Bxe4 Qxe4 22 Nf3 c5


Exercise: Should White capture on c6 en passant?

Answer: It is better to restrict the b8-knight’s movement by leaving the pawn on b5.
23 Qe3
23 bxc6 Nxc6 24 Qe3 Qd5 allows Black to coordinate well.
23 ... Qf5 24 dxc5 Rxc5 25 Qf4
It is not clear why White didn’t go for the obvious looking 25 Rfc1 Be7 26 b6
Rxc1+ 27 Rxc1 Nc6 28 Nd4 when the b-pawn will make the difference.
25 ... Qxf4 26 gxf4 Kf8 27 Rfc1 Be7 28 Ne5 Ke8 29 Nd3
Exchanging the active black pieces off, leaving him with the passive b8-knight.
29 ... Rxc1+
29 ... Rf5 30 Rc8+ wins immediately.
30 Rxc1 Kd8 31 Kg2
No rush – the b8-knight is still dominated by the b5-pawn.
31 ... f6 32 Kf3 Ba3 33 Rc3 Bd6 34 h4 h5 35 Nc5 1-0
The threat of Nc5-b7+ and Rc3-c8 cannot be easily stopped. Once White got in 14
e4 his position looked quite dangerous. The b3-pawn was never in that much danger, so
maybe this is the way forward for White in this line. Alternatives for Black are
discussed in the next two games.

Game 19
A.Beliavsky-V.Topalov
Madrid 1997

1 d4 Nf6 2 c4 e6 3 Nf3 b6 4 g3 Ba6 5 b3 b5 6 cxb5 Bxb5 7 Bg2 d5


7 ... d5 is quite a reliable way to meet White’s threats along the h1-a8 diagonal,
although the potential weakness of the c5-square must also come into consideration.
8 0-0 Nbd7 9 Nc3 Ba6
White has quite a wide choice here.
10 Ne5
Probably the most aggressive move in the position. Alternatives are generally fine
for the second player, for example:
a) 10 Bb2 Rc8 11 Re1 Bb7 12 Ne5 c6 13 Nd3 c5 14 e4 dxe4 15 dxc5 Nxc5 16 Nxc5
Bxc5 17 Nxe4 Bxe4 18 Bxe4 Qb6 19 Qf3 Bd4 20 Bxd4 Qxd4 21 Rad1 Qb6 22 Qe3
Qxe3 23 Rxe3 Ke7 was seen in K.Kulaots-L.Le Quang, Khanty-Mansiysk Olympiad
2010 and Black was no worse here.
b) 10 Na4 Rc8 11 Bd2 Be7 12 Ba5 0-0 13 Rc1 Ne4 14 Nd2 Nd6 15 Nc5 Nxc5 16
dxc5 Nb7 17 Bb4 Qe8 18 a4 Rb8 19 Ba3 Na5 20 b4 Nc4 21 Nxc4 Bxc4, as played in
S.Agdestein-M.Carlsen, Tromso 2007, and again Black was fine here.
c) 10 Re1 Bb4 11 Bd2 with a further split:
c1) 11 ... c5 12 dxc5 Bxc5 13 b4 Bxb4 14 Qa4 Qa5 15 Qxa5 Bxa5 16 Nxd5 Bxd2 17
Nxf6+ Nxf6 18 Nxd2 Rc8 19 Nb3 0-0 and the game was soon drawn in V.Anand-
A.Karpov, Amber blindfold, Monaco 1997.
c2) 11 ... 0-0 keeps more tension in the position. After 12 a3 Be7 13 b4 Bc4 14 Na4
Ne4 15 Nb2 Ba6 16 Rc1 Bd6 17 Bf4 Bb7 18 Nd3 g5 19 Bxd6 cxd6 20 Nd2 Nb6 21
Nxe4 dxe4 22 Nb2 f5 Black had some play on the kingside in M.Le Huec-E.Postny,
Rethymnon 2012.

Question: I don’t much like that knight on e5. Can’t I just chop it off?

10 ... Bd6

Answer: Black’s lack of development will tell if the position opens up, and the following line is thematic.
10 ... Nxe5?! 11 dxe5 Nd7 12 Nxd5! is an excellent sacrifice. After 12 ... exd5 13
Qxd5 Nb6 (if 13 ... Rb8 then 14 e6 Nf6 15 exf7+ Ke7 16 Ba3+ wins) 14 Qc6+ Qd7 15
Rd1 Qxc6 16 Bxc6+ Ke7 17 a4! Bxe2 18 Rd2 Bg4 19 a5 Bd7 20 Bf3 Rc8 21 axb6
axb6 22 Ra7 Ke8 23 Bb7 Rd8 24 Ba6 L.Cernousek-I.Priborsky, Czech League 2007
was just clearly better for White.
10 ... c5 is another option, but after 11 Nxd7 Qxd7 12 Ba3 Rc8 13 Rc1 Be7 14 dxc5
Bxc5 15 Bxc5 Rxc5 16 Qd4 Rc6 17 Rfd1 0-0 18 e4 White had some advantage in
A.Delchev-V.Iordachescu, French League 2012.

11 Nc6
11 Bf4 is a serious alternative, for example 11 ... 0-0 12 Rc1 Bxe5 13 dxe5 and now
two moves have been tried:
a) 13 ... Ng4?! is met by the now familiar thematic sacrifice 14 Nxd5! exd5 15 Qxd5
Nb6 16 Qa5 Bxe2 17 Rfe1 Bd3, A.Karpov-M.Carlsen, Blitz World Championship,
Moscow 2007 and now 18 h3! Nxf2 (18 ... Nh6 19 Bxh6 gxh6 20 Rcd1) 19 Kxf2 Qd4+
20 Be3 Qb2+ 21 Kg1 Rad8 22 Rxc7 is very good for White.
b) 13 ... Nh5 is a better attempt, and after 14 Na4 Nxf4 15 gxf4 Bb5 16 Nc5 Nxc5 17
Rxc5 c6 18 Qd2 Qb6 19 Rfc1 a5 20 e4 a4 was just unclear in L.Aronian-M.Carlsen,
Moscow (rapid) 2011.
11 ... Qc8 12 a3 0-0 13 Bg5 Qe8
Instead, after 13 ... Qb7 14 Na5 Qb6 15 b4 h6 16 Be3 Ng4 17 Bc1 c6 18 h3 Ngf6 19
Be3 Rac8 20 Rc1 Black is OK but his c6-pawn will require constant attention.
14 Qd2 Nb6
15 Na5
15 Bxf6 gxf6 16 Nb4 Bb7 17 Nd3 was another way to keep control of the position.
By preventing the standard ... c7-c5 break, it is not easy to see where Black’s
counterplay will come from.
15 ... c5 16 dxc5 Bxc5 17 Bxf6 gxf6
Despite the bad kingside structure, Black has gained some space with the ... c5
break.
18 Rfc1 Rc8 19 e4 d4 20 Nd1 d3?!
Possibly a bit too ambitious. 20 ... Qb5 21 b4 Bd6 22 Nb2 Qe2 was simpler, to try
and exchange queens and ease the pressure.
21 e5 Bd4 22 Rxc8 Qxc8 23 Rc1 Qd7 24 Bc6 Qe7 25 Be4 fxe5 26 Nc6 Qf6 27
Bxd3 Bb7
It is hard to say what is going on here! Both players have active and inactive pieces,
so the assessment is not clear.
28 Be4 Bxc6 29 Rxc6 Rd8 30 Nc3 Nc8 31 Bg2 Kg7 32 Ne4 Qg6 33 Qa5 Rd5 34
Qb4 Nb6 35 Qe7 Bb2?!
35 ... Rd7 leaves the position roughly balanced.
36 h4 Rd1+ 37 Kh2 Nd5 38 Qe8 Nf6 39 Nxf6 Qxf6 40 Rc8
Now both kings are under fire, but it is probably Black’s king which is the weaker.
40 ... Kh6
40 ... Qxf2?? 41 Qg8+ Kh6 42 Qg5#.
41 Qf8+ Qg7

42 Qc5
When attacking, do not queen swap. This would alleviate the pressure on the Black
king.
42 ... Qg6 43 Bf3 Rd7 44 g4 f6 45 Qf8+ Qg7 46 Qe8 Qf7 47 Qh8 e4 48 Bxe4
Qg7 49 Qxg7+
By exchanging now Beliavsky emerges a pawn ahead, however Topalov holds firm
for the draw.
49 ... Kxg7 50 Rc6 Bxa3 51 Rxe6 Re7 52 Ra6 Bc5 53 f3 Bb6 54 Ra2 Bc7+ 55
Kh3 Bf4 56 Ra5 h6 57 Rf5 Bd2 58 Kg2 Re5 ½-½
Once the rooks come off, opposite colours bishops tend to be a draw. This position
is no exception.

Game 20
C.Vernay-E.Postny
French League 2010

1 d4 Nf6 2 c4 e6 3 Nf3 b6 4 g3 Ba6 5 b3 b5 6 cxb5 Bxb5 7 Bg2 Bb4+ 8 Bd2 a5


Black initiates early play on the queenside, and in fact we saw a similar ... a5 idea
in the previous chapter. There is always the idea of ... a5-a4 to try and give White a
weak pawn on the queenside, which can be targeted at a later date. Notice Black does
not fear a discovered attack on the a8-rook by the f3-knight, since the knight currently
has no decent attacking square to go to. Of course, this possibility must always be
considered!

9 Bxb4

Question: This looks anti positional doesn’t it? After Black recaptures he will
have an open a-file, and the b4-pawn will paralyse White’s queenside pawns.

Answer: As is often the case in modern day chess, the ‘ugly’ moves can have a tactical reason behind them.
Here we see an unusual tactic that already wins White a pawn. Alternatives for White are less dangerous:
a) 9 0-0 d5 10 Nc3 Ba6 11 a3 Be7 12 Ne5 0-0 13 Re1 Nfd7 14 Nf3 Nf6 15 Ne5
Nfd7 16 Nxd7 Nxd7 17 e4 dxe4 18 Bxe4 Rb8 19 Na4 Nf6 20 Bf3 Qxd4 21 Bxa5 Qxd1
22 Bxd1 c5 was roughly equal in E.Bacrot-V.Anand, Wijk aan Zee 2006.
b) 9 a3 Bxd2+ 10 Qxd2 Bc6, and here the difference made by the inclusion of a2-a3
and ... a7-a5 over the normal lines is hard to quantify, but the b3-pawn could be loose
in some variations. 11 0-0 Qc8 12 Qc2 Qb7 13 Nbd2 0-0 14 Rfc1 Ra7 15 Ra2 Bd5 16
Rb2 d6 17 Ne1 Bxg2 18 Nxg2 Nbd7 19 Qc6 was played in A.Karpov-J.Timman, 4th
matchgame, Groningen 2013 and now after 19 ... e5 20 Qxb7 Rxb7 21 Nc4 exd4 22
Nxa5 Ra7 23 b4 c5 Black would have an acceptable game.
9 Ne5 d5 10 Nc3 Ba6 also brings about little harm for Black.
9 ... axb4

10 Qd2!
Attacking the b4-pawn, but also threatening Qd2-g5 with a double attack on the g7-
pawn and b5-bishop.
10 ... Qe7
After 10 ... Nd5 11 e4 Nb6 12 Qxb4 Ba6 13 Nbd2 Nc6 14 Qc5 Ne7 15 Bf1 Bb7 16
Bg2 Ba6 17 Bf1 Bb7 18 Bd3 0-0 19 0-0 f5 20 Qc2 fxe4 21 Bxe4 Bxe4 22 Qxe4 Nbd5
23 a4, in E.L’Ami-J.Timman, Amsterdam 2006, Black had control of the d5-square but
possibly this did not compensate for the pawn deficit.
11 Qg5

11 ... c5
Given the course of the main game, maybe Black should look elsewhere in this
position for potential winning chances.
a) 11 ... Ra5 has never been played but my computer suggests this. Interesting play
arises from 12 Qxg7 Rg8 13 Qh6 Nc6 14 Qc1 (14 Qd2 Qd6 15 0-0 Ne4 16 Qe3 f5 is
similar) 14 ... Qd6 15 Qb2 Ne4 16 0-0 f5 17 Re1 f4 with an attack on the white king.
Note that the b1-knight still cannot come out as 17 Nbd2 is well met by 17 ... Nc3!
b) 11 ... d5 12 Qxg7 Rg8 13 Qh6 with two interesting lines from Black’s point of
view:
b1) 13 ... c5 14 dxc5 Qxc5 15 0-0 Ke7 and now the e2-pawn is hanging and it is not
easy to defend. 16 Rc1 (16 Qd2 Ne4 17 Qb2 Nc6 with play similar to 11 ... Ra5 above
– the b1 knight still cannot move) 16 ... Qxf2+ 17 Kxf2 Ng4+ 18 Ke1 Nxh6 19 a4 bxa3
20 Nxa3 Bd7 21 Ne5 Ng4 22 Nxg4 Rxg4 23 Nc2 Rxa1 24 Rxa1 Nc6 25 Kd2 f5 gave
Black some grinding chances which the Grandmaster managed to convert in K.Stokke-
V.Malakhatko, Gjovik 2009.
b2) 13 ... Nc6 is quite interesting, playing for a central break. 14 Qc1 e5 15 dxe5
Nxe5 16 0-0? and after this the tables turned. T.Sachdev-G.Papp, Paks 2007 concluded
16 ... Bxe2 17 Nxe5 Qxe5 18 Re1 Qxa1 19 Qc6+ Kf8 20 Qxa8+ Kg7 21 Qa5 Bd3 22
Qxc7 Re8 23 Rd1 Qxa2 24 Nd2 Be2 25 Qf4 Bxd1 26 Qg5+ Kf8 27 Qxf6 Qxd2 28
Qh8+ Ke7 29 Qe5+ Kd7 30 Bh3+ Re6 0-1.
So further tests are required, but there is some potential activity here for the second
player to get their teeth into.
12 Qxg7
12 Qxc5 Qxc5 13 dxc5 Ra5 14 Nd4 Bc6 15 Nxc6 Nxc6 16 Bxc6 dxc6 17 Nd2 Rxc5
18 Nc4 Ke7 brings about a fairly even queenless middlegame.
12 ... Rg8 13 Qh6 Nc6
13 ... c4?! 14 bxc4 Bxc4 15 Qd2 Bd5 16 Qb2! Nc6 17 0-0 Kf8 18 Nbd2 Ne4 19
Nxe4 Bxe4 20 Rfc1 Qd6 21 e3 and it looked like Black had very little compensation
for the pawn in Z.Ilincic-B.Knezevic, Serbian Championship 1998.
14 dxc5 Qxc5 15 0-0 Ke7 16 Nbd2


Exercise: Should Black take the e2-pawn?

Answer: No. 16 ... Bxe2 is met by the nice tactic 17 Qxf6+ Kxf6 18 Ne4+ Ke7 19 Nxc5 Bxf1 20 Bxf1 when the
two pieces should outweigh the rook.
16 ... Qb6 17 Nc4
Black appears to have compensation here for the pawn. If instead 17 Qh4 then 17 ...
Rg4 18 Qh6 Rg6 with a draw by repetition.
17 ... Bxc4 18 bxc4 Ra3 19 e3 Rg6 20 Qh3 Qa6
Now the a2- or c4-pawn will fall.
21 Rab1 Qxc4 22 Nd2 Qc2 23 Bxc6 Qxd2
23 ... dxc6! was the way to go. 24 Nb3 Rxa2 25 Nd4 Qe4 26 Rbc1 Kf8 27 Rxc6 Kg7
and it is hard for White here to even draw.
24 Rfd1 Qc3 25 Rbc1 Qb2 26 Rb1 Qxa2 27 Qh4!
Well played. By going out too hard for the win, Black left himself wide open. This
excellent move turns the tables back in White’s favour.

27 ... Qxb1
27 ... dxc6 28 Qxb4+ Ke8 29 Qb8+ Ke7 30 Qd8# was the tactical reasoning behind
27 Qh4!
28 Rxb1 Rg4 29 Qh3 dxc6 30 Qf1 Nd5 31 Qd1 Rc4 32 Qe2 Rac3 33 e4 Nf6 34
e5 Nd5
Yet again Postny defends well and comes back into the game. Those two outside
passed pawns could cause trouble if they roll down the board.
35 Qd2 Kd7 36 Ra1 c5 37 Ra6 Ra3??
37 ... b3! and the advanced b-pawn gives Black sufficient counterplay.
38 Rd6+ Kc7 39 Qg5
Unfortunately the queen steps in and it’s all over. Her Majesty is notoriously good at
chomping on weak opposition pawns.
39 ... Ra8 40 Qg7 Kb7 41 Qxf7+ Nc7 42 Qd7 b3 43 Qc6+ Kc8 44 Rd7 Rc1+ 45
Kg2 Ra7 46 Rxh7 Rd1 47 Qb6! 1-0
A very entertaining game. The 7 ... Bb4+ and 8 ... a5 line remains an interesting
choice, as Black can sacrifice the g7-pawn in most lines and obtain decent play.

Game 21
A.Indjic-Yu Yangyi
World Junior Championship, Athens 2012

1 d4 Nf6 2 c4 e6 3 Nf3 b6 4 g3 Ba6 5 b3 b5 6 Bg2


It appears as though White missed the previous move of the opponent and played on
autopilot, as the c4-pawn is now only defended once! However, White has a tactical
trick based on the a8-rook, as we shall soon see.
6 ... bxc4 7 Ne5 Bb4+
Other moves are just worse:
a) 7 ... c6 8 bxc4 Qc7 9 Qa4 Bb7 10 Nc3 looks very pleasant for White, especially
given Black cannot strike out successfully with 10 ... c5 as after 11 Nb5 Qb6 12 d5!
exd5 13 cxd5 the d-pawn is poisoned, since 13 ... Bxd5 14 Bxd5 Nxd5 15 Qc4 Qe6 16
Qxd5! wins material.
b) 7 ... d5 8 bxc4 c6 9 0-0 Be7 10 Qa4 0-0 11 Rd1 Nfd7 12 Nxd7 Qxd7 13 Nd2 Qc8
14 Ba3 Bxa3 15 Qxa3 Nd7 16 Rac1 Nb6 17 cxd5 exd5 18 e4 was always better for
White, although the second player held the draw in I.Zaja-D.Campora, Porto San
Giorgio 2002.
8 Bd2

8 ... cxb3!
Two can play at that game! After just eight moves of a Queen’s Indian, we have two
pieces en prise! Instead 8 ... Bxd2+ 9 Nxd2 d5 10 bxc4 0-0 11 Rc1 looks better for
White, the early ... b5 sortie has not really gained anything from Black’s perspective.
9 Bxb4!?
Others:
a) 9 axb3 is covered in the next game.
b) 9 Bxa8 is answered by 9 ... b2, in a similar way to the game.
9 ... b2
The key idea.
10 Nc3 bxa1Q 11 Qxa1 d5

Question: Why would White go in for this?
He is already an exchange and a pawn down.

Answer: As usual in chess, it is never so ‘black and white’! The black king will be stranded in the centre for a fair
amount of time, and you could argue that the h8-rook is also not participating. It all depends if Black can unravel from
here.
12 Qc1
12 Qd1 is the other move that has been played in this position. 12 ... c6 13 Qa4 Nfd7
14 Nxc6 Nb6 15 Nxd8+ Nxa4 16 Nxe6 fxe6 17 Nxa4 Nc6 18 Bc3 Kf7 19 Nc5 Bc4 20
a3 Rab8 21 Kd2 Rhe8, A.Ebrahimi-V.Gaprindashvili, Urumia 2008, saw Black keep a
small advantage and eventually convert.
12 ... c5
A pawn sacrifice to gain time for development.
13 Bxc5 Nbd7 14 Bb4 Qb6 15 a3 Qxd4
Continuing to play riskily, but the d4-pawn was an important part of White’s setup in
the centre.
16 Nxd7 Nxd7
16 ... Kxd7 17 0-0 Rhc8 18 Rd1 Qe5 19 e4 is still complicated despite what the
computer says here. The king on d7 is still feeling the heat.
17 0-0 Rc8 18 Rd1 Qe5 19 f4 Qc7 20 Qa1 Nf6
It certainly feels as though the exchange sacrifice hasn’t really worked out. Of course
it took accurate play from the Chinese GM to prove this.
21 e4 Qb6+ 22 Kh1 Ng4 23 h3
There wasn’t really anything else.
23 ... Nf2+ 24 Kh2 Nxd1 25 Nxd5 exd5 26 Qxg7 Ne3
Calm as you like. It’s as if both players were trying to give up their rooks in this
game!
27 Qxh8+ Kd7 28 Qxh7 Rc2 29 Qxf7+ Kc8 30 Bd2 Rxd2 31 Qf8+ Kb7 32 Qe7+
Kb8 0-1
No more checks, and so its game over. Another entertaining tussle.

Game 22
S.Brynell-Hou Yifan
Wijk aan Zee 2007

1 d4 Nf6 2 c4 e6 3 Nf3 b6 4 g3 Ba6 5 b3 b5 6 Bg2 bxc4 7 Ne5 Bb4+ 8 Bd2 cxb3 9


axb3

White’s 9th move is the first deviation from the previous game. Brynell decides not
to go in for the complications we saw there, and goes for a pure pawn sacrifice.

Question: What does White gain from this gambit?

Answer: Open lines. The a8-rook is still under threat, and now the a1-rook has joined the party. White has a lead
in development and chances to occupy the centre also with e2-e4 at the right moment. All of that adds up to pretty
decent compensation. I did mention this line was risky for Black, but she has snatched a pawn and hopes to hang onto
it until the endgame. Both players accept the challenge and we see an interesting fight.
9 ... Bxd2+ 10 Qxd2 d5
In practical play, Black has scored well with this move.
10 ... c6, on the other hand, has not scored as well, for example after 11 0-0 0-0 12
Rc1 Nd5 13 e4 Ne7 14 d5 Ng6 15 Nxg6 hxg6 White has 16 d6! which is an excellent
idea, borrowed from Bronstein who used this theme way back. Black can hardly move
after this, and S.Ernst-B.Michiels, Groningen 2003 concluded 16 ... Bb7 17 e5 Qb6 18
Nc3 Na6 19 Ne4 Rfb8 20 Ra4 Qb5 (20 ... Qxb3 21 Qf4! and Ne4-g5 and Qf4-h4 will
come with devastating effect) 21 Bf1 Qxe5 22 Bxa6 Qd5 (22 ... Bxa6 23 Rc5 highlights
the defects of Black’s position) 23 Rd4 Qh5 24 Be2 Qh3 25 Nc5 Bc8 26 Qg5 1-0. An
excellent example of how to use a space advantage by Dutch Grandmaster Sipke Ernst.
11 0-0 0-0 12 Nc3

12 ... Qd6
This move is a little slow. Instead the immediate 12 ... Nfd7 looks better, for
example 13 Nxd7 Qxd7 14 Rfc1 c6 15 Ra5 Qb7 16 Qa2 was played in I.Arakelov-
B.Sambuev, Voronezh 2003 and now after 16 ... Qb6 Black would be fine.
13 Rfc1 Nfd7?!
Instead, after 13 ... c6 14 e4 Qb4, White has compensation for the pawn, but Black is
solid.


Question: What should White do about the threat to e5?

Answer: Nothing! White should counter using his lead in development.
14 e4! c6
14 ... Nxe5 15 dxe5 Qxe5 16 exd5 is already almost close to losing for the second
player.


Question: How should White proceed? The d5-pawn appears to be well defended.

Answer: This is where White should be searching for breakthrough tactics. Black’s position looks to be built on
sand.
15 Nxc6! Qxc6
15 ... Nxc6 16 Rxa6 regains the piece with advantage.
16 exd5 Qd6
After 16 ... exd5 17 Nxd5 Qd6 18 Qb4! Qxb4 19 Nxb4 and the a8-rook is lost.
17 Ne4 Qb6
18 Nc5
18 dxe6! would have been the culmination of White’s strategy in this game. 18 ...
Bb7 (18 ... fxe6 19 Nc5 and everything is hanging again) 19 exd7 Nxd7 20 Nc5 and
White is a clear pawn up with a near winning position.
18 ... Nxc5 19 dxc5 Qb5 20 dxe6?!
20 c6 exd5 21 Bxd5 is crushing.
20 ... Bb7
Hou Yifan didn’t become Women’s World Champion for no reason, and she begins
her fight back.
21 exf7+ Kh8
21 ... Rxf7 22 Qd8+ Rf8 23 Bd5+ Bxd5 24 Qxd5+ Kh8 25 Qxa8 hoovers up all the
pieces.
22 Ra5 Qxb3 23 Qd6 Nc6 24 Bxc6 Qxf7 25 Bxb7 Qxf2+
The best practical chance.
25 ... Qxb7 26 c6 is very uncomfortable, as the extra c-pawn will be a menace.
26 Kh1 Rad8
Suddenly there is counterplay, and with possible time trouble it is not so clear now.
27 Raa1
27 Qc6 Rd2 28 Qg2 Qd4 29 Qg1 Qb2 ends in a perpetual after 30 Rxa7 Rxh2+ 31
Qxh2 Qxc1+ 32 Qg1 Qh6+ 33 Kg2 Qd2+ 34 Kh3 Qh6+ with a draw.
27 ... Rxd6 28 cxd6 Qb6 29 Bc6 Rc8 30 Bg2 Rd8 31 Rc6 Qd4 32 Rca6 Rd7 33 h4
Rxd6 34 Rxa7 h5 35 Ra8+ Kh7 36 R1a2 Qe3 37 R8a3 Qe5 38 Kh2 g5?
Black had outplayed her opponent and was better, but this must have been a time
trouble move, allowing White to reach a drawn position.
39 Ra5 Qe6 40 hxg5 Kg6 41 Rf2 Qe3 42 Raf5 Rd2 43 Rf6+ Kg7 44 Rf7+ Kg6 45
R7f6+ ½-½
Another wild game in this line! Objectively, Black should be OK after the pawn
sacrifice, but needs to pay attention to the details around moves 10-13.

Game 23
R.Osterman-A.Grosar
Slovenian Championship 1993

1 d4 Nf6 2 c4 e6 3 Nf3 b6 4 g3 Ba6 5 b3 b5 6 Bg2 bxc4 7 Ne5 Bb4+ 8 Kf1


Yet another surprise in this variation! White values the loss of castling rights as less
important than the gain in the centre (winning the c4-pawn back quickly) and the
queenside (trying to take advantage of the black pieces such as the a6- and b4-bishops).
8 ... d5
8 ... c6 is playable but a bit dry, for example 9 bxc4 0-0 10 Qa4 Qa5 11 Qxa5 Bxa5
12 Bd2 Bc7 13 Bc3 d5 14 Nd2 Nfd7 15 Nd3 Nb6 16 Nb2 dxc4 17 Rc1 Rd8 18 Bf3
Bb5 19 Kg2 N8d7 20 Ba5 Rac8 21 Ndxc4 Nxc4 22 Nxc4 Bxc4 23 Bxc7 Rxc7 24 Rxc4
Nb6 25 Rxc6 Rxc6 26 Bxc6 Rxd4 27 Rc1 was equal in Y.Bayram-H.Banikas, Turkish
League 2008.
9 bxc4 0-0 10 c5

10 ... Nfd7

Question: Why does Black often play like this?

Answer: Well 10 ... Nbd7 loses immediately to 11 Nc6, but generally it is crucial to oust the beast on e5 as it
controls a lot of squares.
11 Nxd7
11 Nd3 actually loses material. 11 ... Bxd3 12 Qxd3 and now:
a) 12 ... Nxc5 13 dxc5? Qf6 14 a3 Bxc5 0-1 P.Vezzosi-L.Kritz, Conegliano 2008
must be in with a shout for ‘quickest Black win’ using the Queen’s Indian Defence.
b) 12 ... Qf6 13 Bb2 Bxc5 is the other way to do it: 14 Nd2 Nc6 15 Nb3 Bb6 16 Bf3
Rad8 17 Kg2 e5 18 dxe5 Ndxe5, with an extra pawn for Black, was played in
D.Kovaljov-A.Shishkov, Estonian Championship, Tallinn 2008.
11 ... Qxd7
11 ... Nxd7 appears to lose material after 12 Qa4 but again Black has resources, in
this case 12 ... Rb8! 13 Qxa6 Nxc5 14 dxc5 (or 14 Qc6 Rb6 traps the queen) 14 ... Qf6
winning the a1-rook.This could be another useful trap to bank away.
12 a3 Ba5 13 Bf3 Nc6 14 Kg2 Rab8 15 e3 Bc4 16 Bd2

16 ... Rb7

Question: What is the purpose of this move?

Answer: To double rooks on the open b-file. Black is better developed, and has control of the only open file.
17 Qc1 Rfb8 18 Bxa5 Nxa5 19 Nd2 Nb3 20 Nxb3 Rxb3 21 Qd2 Qb5 22 Rhc1
Rb2 23 Qe1 f5
Black prevents the e3-e4 pawn break.

24 h4 Bd3 25 Kg1 Be4 26 Bd1 Qd3 27 f3


Did Black blunder a piece? His position is so good he can continue an attack.
27 ... Rd2 28 fxe4 Rbb2 29 Bf3
29 exd5 Rg2+ 30 Kh1 Rh2+ 31 Kg1 Rbg2#.
29 ... dxe4 30 Bh1 Re2 31 Qc3 Qxe3+ 32 Qxe3 Rxe3
Amazingly, even though the queens have come off, Black is still winning. The h1-
bishop is merely a bystander here.
33 Rab1 Rxg3+ 34 Kf1 Rh2! 0-1
White has some work to do to make the 8 Kf1 line work. Personally I feel the
alternatives are better.

Game 24
G.Sargissian-H.Odeev
Dubai 2006

1 d4 Nf6 2 c4 e6 3 Nf3 b6 4 g3 Ba6 5 b3 b5 6 Nbd2

6 Nbd2 is another natural reaction to 5 ... b5, defending the c4-pawn. The problem
with this is that the early knight sortie to d2 shows White’s hand, and now the knight
cannot go to the more active c3-square.
6 ... c5
This is the move I am recommending. Since the d4-pawn is not protected by the
queen, we should target it. Other moves are:
a) 6 ... Bb4 is the most popular move, for example 7 Bg2 bxc4 8 bxc4 Bxc4 9 Rb1
and now Black has tried:
a1) 9 ... Nc6 10 Rxb4 Bxe2 11 Qxe2 Nxb4 12 Nc4 0-0 13 0-0 Rb8 was
I.Cheparinov-M.Carlsen, FIDE World Cup, Khanty-Mansiysk 2005. I am not convinced
this is the way to go from Black’s point of view.
a2) 9 ... Ba5 10 Qa4 Bxd2+ 11 Nxd2 Bd5 12 e4 Bc6 13 Qc2 0-0 14 0-0 a5 15 Re1
Na6 16 a3! Rb8 17 Bb2 Bb5 18 d5 as played in Y.Pelletier-R.Markus, Turin Olympiad
2006, and again I prefer White. The two bishops and space advantage certainly
compensate for the pawn minus. The a6-knight is also poorly placed.
b) 6 ... bxc4 7 bxc4 c5 8 Bg2 and here the options are:
b1) 8 ... Bb7 9 Rb1 Qc8 10 e4! cxd4 11 e5 Ng8 12 0-0 was the continuation of
M.Brodsky-L.D’Costa, Le Touquet 2007. I can vouch that this line looked pretty scary
at the time!
b2) 8 ... Nc6 is definitely safer. After 9 Ne5 Bb7 10 Rb1 Qc7 11 Qa4 Rc8 12 Nxc6
Bxc6 13 Bxc6 Qxc6 14 Qxc6 dxc6 15 Bb2 Nd7 16 e3 Be7 Black was solid but White
held a slight advantage in F.Vallejo Pons-M.Rivas Pastor, Spanish Championship, Leon
2006.
7 Bg2
The more aggressive 7 d5 is the subject of the next game.
7 ... Nc6

8 Ne5 Rc8
Black parries the pressure on the h1-a8 diagonal, and now White has to agree to
simplifying exchanges.
9 Nxc6 dxc6 10 dxc5 Bxc5 11 0-0 0-0 12 Qc2 Qb6
White’s pieces are a little more active, with Black’s a6-bishop restricted at the
moment.
13 Bb2 Rfd8 14 Ne4 Nxe4 15 Bxe4 h6 16 Bh7+ Kh8 17 Bd3 bxc4 18 bxc4
After 18 Bxc4 Bxc4 19 Qxc4 Bd4 Black has activity to offset the potential weakness
on c6.
18 ... Bd4 19 Rab1 Qa5 20 Rfc1 c5
Black is probably fine here now.
21 Bc3 Bxc3 22 Qxc3

22 ... Qc7
22 ... Qxc3 23 Rxc3 Rd6 should also be equal.
23 Qa3 Rd6 24 Be4 Rcd8 25 Kg2 Rb6?!
Trading a pair of rooks gives White’s pieces a little more scope, so Black should
just maintain his powerful doubled rooks on the d-file.
26 Rxb6 Qxb6 27 Rb1 Qd6 28 Bf3 Bxc4 29 Qxa7 e5?!
If Black goes for counterattack with, for example 29 ... Qe5, he should still be able
to hold.
30 Rc1 Qe6 31 Qxc5 Bxa2 32 Qa5
Now it definitely tricky for Black as the a2-bishop is running short of squares.
32 ... Re8 33 Rc6 Qb3 34 Ra6 e4
34 ... Bb1 35 Rb6 Qc2 36 Qb5 wins.
35 Bg4 Bb1 36 Rb6 Qd1 37 Qb5 1-0
Despite the result, I do not think the opening caused too many problems for Black.

Game 25
B.Gelfand-V.Ivanchuk
FIDE Grand Prix, London 2012

1 d4 Nf6 2 c4 e6 3 Nf3 b6 4 g3 Ba6 5 b3 b5 6 Nbd2 c5 7 d5



We know all about this d4-d5 idea already. Does it work here also?
7 ... exd5 8 cxd5 Nxd5


Question: What is the point of White’s pawn sacrifice?

Answer: Lines are opened for the two bishops and it is not easy for Black to get in ... d7-d5.
9 Bb2
9 Bg2 Qf6! is actually quite tricky for White at this early stage.


Exercise: How should Black develop here, and
what happens if he does try to go for an early ... d5?

Answer:
9 ... Nc7
Black comes up with a plan to regroup his minor pieces and eventually castle
kingside, based on ... Nd5-c7-e6. At the same time, if Black plays ... d5 too early it can
backfire. For example, if Black plays 9 ... Nf6 10 Bg2 (10 e4 Be7 11 a4 is also
possible) and now:
a) 10 ... d5 11 0-0 Be7 12 Nh4! with ideas of coming to the nice f5-square.
b) 10 ... Nc6 11 0-0 d5 12 Nh4 Be7 13 Nf5 0-0 14 e4 and here Black faces great
pressure on all sides of the board, for example 14 ... d4 15 e5! Nd5 16 Qf3 winning a
piece.
10 Bg2 Bb7
Again if 10 ... d5 11 0-0 Nc6 then White again has the key move 12 Nh4, releasing
the g2-bishop along the h1-a8 diagonal. It also dissuades the ... d4 push which Black
would like to play to block out the b2-bishop.
11 0-0 Ne6
11 ... Be7 loses the g7-pawn, so Ivanchuk takes time out to defend g7.
12 a4

12 ... b4
After 12 ... a6 13 Nh4 Bxg2 14 Kxg2 Nc6 15 Ndf3 Be7 16 Nf5 White still has plenty
of pressure.
13 e4 Be7 14 Re1 Nc6 15 Nc4
The consequence of 12 ... b4 is that the d2-knight has this nice square too.
15 ... 0-0 16 e5 Re8 17 Nfd2 Qc7 18 Ne4
The idea of bringing a knight to the d6-square has taken a lot of time, and Black can
hold his own here.
18 ... Ncd4 19 Ncd6 Bxd6 20 exd6
20 Nxd6? Bxg2 21 Kxg2 Qc6+ 22 f3 Rf8 would be good for Black.
20 ... Qb6 21 a5 Qb5 22 Bf1 Qc6 23 Bg2 Qb5 24 Bf1 Qc6 25 Bg2 ½-½
Neither side can realistically avoid the repetition.
While the d4-d5 sacrifice always needs careful analysis, I think 6 Nbd2 c5 is very
much playable for Black.

Game 26
J.Gomez Luesia-D.Adla
Zaragoza 2003

1 d4 Nf6 2 Nf3 e6 3 c4 b6 4 g3 Ba6 5 b3 b5 6 Qc2




6 ... bxc4 7 bxc4 c5 8 Bg2?!
8 d5 is the only decent move here. As the main game demonstrates, autopilot play
lands White in hot water. After 8 d5 exd5 9 cxd5 Black has played:
a) 9 ... d6 10 e4 Bxf1 11 Kxf1 Nbd7 12 Kg2 Be7 13 Rd1 0-0 14 Nc3 Qa5 15 Bf4
Qa6 16 a4 Nh5 17 Be3 g6 18 Nb5 which was pleasant for White in R.Aloma Vidal-
F.Remoli Sargues, Spanish U-18 Championship, Linares 2007.
b) 9 ... Bb7 10 Bg2 (10 e4 Qe7 11 Nc3 Nxd5 wins the pawn) with a further choice:
b1) 10 ... Nxd5 11 0-0 Be7 12 Bb2 0-0 13 Rd1 Na6 with unclear play.
b2) 10 ... Bxd5 11 Nc3 Bc6 12 0-0 Be7 13 e4 0-0 14 e5 Ne8 15 Rd1 Nc7 16 Be3
Ne6 17 Ne4 Bxe4 18 Qxe4 Nc6 19 Nd2 f6 A.Gamundi Salamanca-M.Rivas Pastor,
Ponferrada 1992 with compensation for the pawn, but maybe not more.
The position resembles the 5 Qc2 line from Chapter Two, except that the b-pawns
have also been exchanged off. Whom that favours is not clear, but in general White
tends to get some compensation with active play in these positions.
8 ... Nc6
9 0-0
9 Ne5 Nxd4 10 Qa4 Rb8 11 e3 Rb4 12 Qd1 Qb8 13 exd4 Rxb1 14 Rxb1 Qxb1 15 0-
0 Qb6 A.Kelbling-A.Litwak, Unna 2008 probably isn’t quite enough compensation for
the pawn, even though White has a big lead in development. Black is just very solid
here.
9 ... cxd4 10 Bb2 Bc5 11 Rd1 Rc8 12 Nxd4 Nxd4 13 Bxd4 Bxd4 14 Rxd4 Qb6 15
e3 e5 16 Rd1
16 Rh4 0-0 17 Nd2 h6 and the rook is just out of play on h4.
16 ... Rxc4 17 Qf5 Qe6 18 Qf3 0-0
A pawn to the good, Black makes no mistake converting.
19 Nd2 Rc2 20 a4 Rfc8 21 Bf1 Bxf1 22 Nxf1 d5 23 Nd2 h6 24 Rab1 e4 25 Qg2
d4
Not strictly necessary, but attractive nonetheless.
26 exd4 e3 27 fxe3 Qxe3+ 28 Qf2
28 Kh1 Ng4 is simply horrible.
28 ... Qxf2+ 29 Kxf2 Ne4+ 30 Ke3 Nc3
Black wins the exchange.
31 Ra1 Nxd1+ 32 Rxd1 R2c3+ 33 Ke4 Re8+ 34 Kd5 Rd8+ 35 Ke4 f5+ 0-1
White resigned as 36 Kxf5 Rd3! pins and wins.
In summary, the 4 ... Ba6 5 b3 b5 variation is one of the more risky attempts to play for a win and unbalance the
position from an early stage.
Chapter Five
Fianchetto Variation: 4 g3 Bb7 5 Bg2 g6
1 d4 Nf6 2 c4 e6 3 Nf3 b6 4 g3 Bb7
Here we investigate the classical way to play with 4 ... Bb7. Instead of bothering
with the nuances of getting White to play b2-b3 as we saw in previous chapters, simple
development will be the priority.
5 Bg2 g6

Again I am going for something offbeat and a path less trodden. The idea of posting
the bishop on the g7-square (instead of the usual e7-square), is that in many Queen’s
Indian lines Black plays ... Ne4 and eventually ... Bf8-e7-f6 to attack White’s centre. By
immediately playing the kingside fianchetto, it could be argued that this saves time over
that plan. With the typical Queen’s Indian ‘cat and mouse’ being played between the b1-
knight and f8-bishop, Black decides that since White doesn’t threaten e2-e4 any time
soon, he can put the bishop on a safer square. Another plan for White is to advance d4-
d5 early and seize space. Whilst that attempts to block out the b7-bishop, it opens up
the h8-a1 diagonal for its partner in crime on g7. Consequently there are an abundance
of tactics based on that theme which we will encounter. It should however be noted that
the worst case scenario is that White plays d4-d5, Black obtains no counterplay and
gets squashed when e4-e5 arrives afterwards. Try not to let that happen!
From the position in the diagram above, White has various ways to fight for the
centre. The immediate push 6 Nc3 Bg7 7 d5 is the subject of Games 27-28, while in
Game 29 Black inserts 6 ... Ne4 to counter this plan. In fact, an early ... Ne4 is a
recurring theme for Black in this line, he often plays this as soon as White develops his
b1-knight with Nb1-c3.
If White castles on move 6, then after 6 0-0 Bg7 we look at 7 Nc3 in Games 30-31, 7
b3 in Game 32 and 7 Re1 in Game 33. Finally the unusual 6 Qd3 is the subject of Game
34.

Game 27
B.Morchiashvili-B.Savchenko
European Championship, Plovdiv 2008

1 d4 Nf6 2 c4 e6 3 Nf3 b6 4 g3 Bb7 5 Bg2 g6 6 Nc3 Bg7 7 d5


Question: OK so you mentioned this in the chapter introduction. Should I be worried?

Answer: You will see later on in this book (Game 46, Gupta-Dao Thien Hai) another example of this space
gaining idea. 7 d5 breaks the influence of the b7-bishop towards the centre, especially on e4. It is an ambitious move
that could of course backfire. Both sides must believe in their position and simply go for it!
7 ... 0-0 8 0-0 d6
Black keeps the position closed for now. 8 ... exd5 is covered in the next game.
9 e4 Nbd7 10 Re1 Re8 11 h3 a6
Playing in true ‘Hedgehog’ Style. The pawn breaks ... b5, ... c6 and ... exd5 all come
into consideration, keeping White on his toes. The drawback is that if Black does not
do something constructive soon, he could get pushed off the board. Black is walking a
fine line, but by taking some risks is able to play for a win.
12 Qc2 Nc5 13 Rb1
Getting the rook off the a1-h8 diagonal. White now threatens 14 b4 to push the c5-
knight away.
Note that the immediate 13 b4 is answered by Ncxe4 14 Nxe4 Nxe4 which wins a
clear pawn as the a1-rook is hanging.


Exercise: What should Black do about White’s threat?

Answer: Borrow from the King’s Indian! Black secures the c5-square for his knight with:
13 ... a5 14 b3
14 a3 a4! maintains the knight on c5.
14 ... exd5 15 cxd5
After 15 exd5 Rxe1+ 16 Nxe1 Bc8! 17 Qd1 Bf5 18 Nc2 Black has 18 ... h5
preventing White from playing g3-g4 which would push the f5-bishop back. Black can
hardly complain about his position here, as he is very active.
15 ... Ba6 16 Bf1
16 Nd4 Bd3 was the threat. It turns out White’s rook is not ideally placed on b1.
16 ... Bxf1 17 Kxf1


Question: I still don’t really like Black here. If Nf3-d4 comes,
the c6-square beckons as an excellent square for White’s knight.

Answer: Well let us counter immediately then!
17 ... c6 18 dxc6 Qc8 19 Bg5
19 Kg2 Qxc6 with severe pressure on the e4-pawn.
19 ... Qxc6 20 Bxf6 Bxf6 21 Nd5 Bg7
White’s position should be fine but perhaps he is lulled into a false sense of security.
He must just have believed nothing could go wrong here ...
22 b4 axb4 23 Rxb4 b5 24 Nd4?!

Exercise: After this natural move, Black can get the upper hand. How?

Answer: With the unusual manoeuvre:
24 ... Qc8!
Hitting the h3-pawn and unpinning the c5-knight.
25 Kg2?
25 Nxb5 Qxh3+ 26 Kg1 Re5 27 Nf4 Qd7 gives Black an edge, but is better than the
game continuation.
25 ... Na6! 26 Qxc8 Raxc8
Suddenly out of nowhere, both the d4-knight and b4-rook are hanging.
27 Nxb5 Nxb4 28 Nxb4 Rc5 29 a4
29 Nxd6 Re6 30 Rd1 Bf8 31 Nb7 Rb5 wins one of the errant knights.
29 ... Ra8 30 Nd5 Rxa4 31 Nxd6 Be5 32 Ne8 Rc2
White just cannot coordinate himself well enough to defend here.
33 Nef6+ Kg7 34 Ng4 Bd4 35 Nge3 Rd2 36 Kf3 Raa2 37 Rf1 Bc5 38 Nc3 Ra6 39
Ncd5 Re6 40 Rc1 Bxe3 41 Nxe3 Rf6+ 0-1
The f2-pawn falls and with it the game. Risky stuff, but it brought home the point!

Game 28
A.Yusupov-M.Manolache
European Championship, Dresden 2007

1 d4 Nf6 2 c4 e6 3 Nf3 b6 4 g3 Bb7 5 Bg2 g6 6 Nc3 Bg7 7 d5 0-0 8 0-0 exd5


The actual game move order was 1 d4 Nf6 2 Nf3 g6 3 c4 Bg7 4 g3 b6 5 Nc3 Bb7 6
d5 0-0 7 Bg2 e6 8 0-0 exd5 transposing into our line of the Queen’s Indian. Here Black
deviates from 8 ... d6 which we saw in the last game.
9 cxd5 Re8
Here Black tries to slow down the e2-e4 advance, as opposed to the previous game
where he allowed White to build a big centre. Delaying the advance of the e-pawn
allows Black the extra option of ... c6 or even ... c5 to attack the centre.
10 Bf4
There were other possibilities here:
a) 10 Re1 was chosen in the most high profile game in this line, which continued 10
... c6 11 e4 d6 12 Bf4 cxd5 13 exd5 Rxe1+ 14 Qxe1 Na6 15 Nd4 Qd7 16 Rd1 Nc7 17
Nc2 Rc8 18 Qe2 which looked OK for Black. Play got very interesting after 18 ... Nh5
19 Bg5 Bxc3 20 bxc3 Nb5 21 Re1 Nxc3 22 Qd2 when White has compensation for the
pawn in an unclear position, A.Giri-M.Rodshtein, European Club Cup, Eilat 2012.
b) 10 Nd4 looks most obvious. The idea is to gain space with e2-e4 and maybe f2-f4
if permitted.
I will give some variations to demonstrate the type of play that can arise from this
position:
b1) 10 ... Na6 11 e4 Nc5 and now:
b11) 12 Re1?! allows the thematic tactic 12 ... Nxd5 13 Nxd5 Bxd5 with:
b111) 14 exd5 Rxe1+ 15 Qxe1 Bxd4 winning a pawn.
b112) 14 Nf5 gxf5 15 exd5 Rxe1+ 16 Qxe1 Qe8 17 Qd1 Qe5 is White’s best choice,
but Black is still better due to his extra pawn and greater activity.
b12) 12 f3 with a few lines:
b121) 12 ... c6 immediately attacking the centre is probably best. 13 b4 Na6 (instead
13 ... Ba6 14 bxc5 bxc5 15 Nde2 Nxd5 16 exd5 Bxe2 17 Nxe2 Bxa1 is very messy) 14
b5 Nc5 15 bxc6 dxc6 and if 16 dxc6 then 16 ... Nfxe4! and again the g7-bishop comes
to the fore. If now 17 cxb7 Qxd4+ 18 Qxd4 Bxd4+ 19 Kh1 Nxb7 20 fxe4 Bxc3 and
Black wins a clear pawn.
b122) 12 ... Ba6 13 Rf2 Nd3 14 Rd2 was played in S.Kustar-L.Barczay, Zalakaros
1994 and now 14 ... c5 15 dxc6 dxc6 16 Nxc6 Qc7 was complicated but by no means
bad for Black, for example 17 Nd4 Nxe4!.
b2) 10 ... c5 and now:

b21) 11 dxc6 is pleasant for White, for example 11 ... dxc6 12 Bf4 Nh5 13 Be3 Qd7
(defending the c6-pawn so that the b8-knight can move) and after 14 Qa4 b5 15 Qd1
White has provoked the weakening ... b5 and the black pieces are still not so well
coordinated. B.Jansson-J.Timman, Stockholm 1971 continued with 15 ... Rd8 when the
Dutch great was fortunate his opponent missed the following tricky line 16 Ndxb5! Qe7
(or 16 ... cxb5 17 Bxb7 and White wins) 17 Nd4 c5 18 Bxb7 cxd4 (if 18 ... Qxb7 then
19 Qb3) 19 Bxa8 dxc3 20 Qa4 cxb2 21 Rab1 and Black probably doesn’t have enough
for the exchange.
b22) 11 Ndb5 d6 12 Bf4 Bf8 normally this retreat means that things have gone wrong
in the Benoni pawn structure, though White’s b5-knight can be ousted quite quickly.
Following 13 a4 Nh5 14 Bd2 Nd7 15 f4 Nhf6 16 Na3 a6 17 e4 b5!? Black was trying
for counterplay but this is risky. 18 axb5 axb5 19 Naxb5 Qb6 20 Rxa8 Rxa8 21 Re1
Ba6 22 Bf1 was the continuation of M.Sorokin-R.Casafus, San Martin 1994 where
careful play brought home the point for the first player.
So 10 Nd4 looks quite attractive, maybe best is 10 ... Na6 as 10 ... c5 appears to be
not ideal in this position.
Now returning to the main game after 10 Bf4:
10 ... d6
10 ... Na6 is playable too. 11 Nd4 Nc5 12 Ndb5 d6 13 Rc1 with play fairly similar
to the main game.
11 Qd2 Nbd7 12 Bh6

12 ... Bxh6

Question: What is the need for Black to take this bishop,
and voluntarily bring the white queen near the black king?

Answer: To distract the queen from defending the d5-pawn. Some players are adverse to this, but it happens,
even in openings like the Sicilian Dragon (for example), if there are tactical reasons for doing so.
When looking at the alternative 12 ... Qe7 13 Bxg7 Kxg7 14 Nd4 Ne5 15 b3 maybe
Black sensed there wasn’t a decent plan here.
13 Qxh6 b5 14 a3
14 Nxb5 Rxe2 and Black exchanges a wing pawn for a more valuable central pawn.
14 ... a5 15 b4
15 Ng5 can be answered by 15 ... b4 16 Nce4 Nf8 using the maxim ‘with a knight on
f8 it cannot be mate’. Then after 17 axb4 axb4 18 Rxa8 Bxa8 the d5-pawn could drop
and there is no obvious breakthrough to the black king.
15 ... axb4 16 axb4 Rxa1 17 Rxa1 Nxd5
17 ... Bxd5 18 Nxd5 Nxd5 19 Ng5 N5f6 20 Bd5 looks uncomfortable for Black, but
maybe it is defendable after the queen sacrifice 20 ... Rf8 21 Ra8 Qxa8 22 Bxa8 Rxa8.
18 Nxd5 Bxd5 19 Ng5

No doubt both players had the forced variation calculated out, but this does look
pretty scary for Black!
19 ... Qf6!
Active defence was the only way here.
20 Bxd5 Qxa1+ 21 Kg2 Re7 22 Qxh7+ Kf8 23 Bxf7 Qg7 24 Ne6+ Rxe6 25
Qxg7+ Kxg7 26 Bxe6 Nb6

Despite the pawn advantage, White was unable to convert this endgame to a win.
Those queenside pawns are a nuisance and Black manages to survive.
27 Kf3 c5 28 Ke4 Kf6 29 bxc5 dxc5 30 Ba2 Na4 31 Kd3 g5 32 h4 gxh4 33 gxh4
Ke5 34 Bg8 c4+ 35 Kd2 Kf4 36 f3 Nc5 37 Kc3 Nd7 38 Kd4 Nf6 39 Bf7 c3 40 Bb3 b4
41 Bd1 Nh5 42 e4 Ng3 43 Kd3 Nh5 44 Kc4 Ng3 45 Bc2 Kxf3 46 e5 Kg4 47 Kxb4
Kxh4 48 Kxc3 Kg5 49 Kd4 Nh5 50 Kc5
This is still tricky but Manolache sees that the king should head to the d4-square for
the draw, in order to monitor the e5-pawn from behind.
50 ... Ng7 51 Kd6 Kf4 52 Bg6 Ke3 53 Ke7 Kd4 54 Kf6 Kd5 55 Bf7+ Kd4 56 Kxg7
Kxe5 ½-½
Both sides sailed close to the edge, so a draw was the fair result.

Game 29
F.Urkedal-S.Agdestein
Norwegian League 2014

1 d4 Nf6 2 c4 e6 3 Nf3 b6 4 g3 Bb7 5 Bg2 g6 6 Nc3 Ne4


Black deviates from 6 ... Bg7 which we saw in the previous two games.

7 d5
The b7-bishop is blocked out and White fights for control of the crucial e4-square
early on. Instead, the routine exchange of knights on the e4-square allows Black’s light-
squared bishop to take up a strong central outpost, which is an important theme in the
Queen’s Indian. After 7 Nxe4 Bxe4 8 0-0 Bg7 9 b3 0-0 10 Ba3 d6 11 Rc1 Nc6 12 d5
Ne7 13 dxe6 fxe6 14 Bh3 Nf5 15 Nd2 Bb7 16 e3 Qg5 which was played in
N.Korniushin-M.Matlakov, Vladivostok 2014, we reach the sort of position Black
wants from this line – to avoid theory and obtain chances for an attack.
7 ... Nxc3 8 Qd4
This was the big idea.
8 bxc3 Bg7 9 0-0 0-0 10 Bg5 Qc8 11 Qd2 Na6 is a bit cramped but playable for
Black, but it easy to see the appeal of the game move 8 Qd4 over this position.
8 ... f6
The odd-looking 8 ... Rg8 saw a practical test in J.Hammer-S.Ivanov, Stockholm
2014, which continued 9 Qxc3 Bg7 10 Qd2 Na6 11 Nd4 Nc5 12 0–0 e5 13 Nb5 a6 14
Nc3 a5 15 e4, and although Black eventually drew the game, what his king is doing on
e8 in this position is anyone’s guess.
9 Qxc3 Bg7
Snatching the d5-pawn is very risky, for example 9 ... exd5 10 cxd5 Bxd5 11 0-0
Bg7 12 Rd1 and now:
a) 12 ... c6 13 Rxd5! cxd5 14 Nd4 Nc6 15 Nxc6 dxc6 16 Qxc6+ Kf8 17 b3 when the
white bishops cut through the position brilliantly.
b) 12 ... Bc6 13 Nd4 Bxg2 14 Ne6! Qc8 15 Nxc7+ Kd8 16 Bf4, with a huge attack
for the sacrificed material was played in G.Sibashvili-H.Aleksanyan, European
Championship, Yerevan 2014.
10 0-0
10 Qe3 0-0 11 Nd4 e5 12 Nc2 f5 13 0-0 d6 14 b4 Nd7 15 Bd2, A.Moussa-
T.Gareev, Los Angeles 2014, and now 15 ... f4 16 Qd3 a5 17 bxa5 Nc5 18 Qa3 bxa5 is
nice for Black as he has found decent coordination here.
10 ... 0-0 11 e4
11 ... f5?!
Logical, but it is met by a strong riposte.
11 ... Na6 12 Nd4 Qe7 maintaining the central tension might have been better.
12 e5 exd5 13 cxd5 Bxd5? 14 Ng5 Bxg2
Presumably Black missed the following queen sortie, which was hard to see when
playing 11 ... f5.
15 Qc4+ Kh8 16 Qh4! Qxg5
16 ... h6 17 Nf7+ Rxf7 18 Qxd8+ Rf8 19 Qxc7 Bxf1 20 Kxf1 is similar to the game,
where the queen will dominate.
17 Bxg5 Bxf1 18 Kxf1 Nc6 19 Bf6 Bxf6
19 ... Nxe5 20 Bxg7+ Kxg7 21 Qd4 Rae8 22 f4 c5 23 Qc3 wins the knight.
20 exf6 Nd8 21 Rd1
Materially Black is equal here, but his position is lost. He has no coordination and
he has no match for the f6-pawn, combined with the active white queen and rook.
21 ... Rf7 22 Qc4 c6 23 Rd6 Rc8 24 Qd3 Rc7 25 Qe3
Spotting a nice tactic which cannot be prevented.
25 ... Kg8 26 Rxd7! 1-0
Black resigned as 26 ... Rcxd7 27 Qe8+ Rf8 28 Qxd7 is easily winning for White.

So maybe after 6 Nc3 Black should stick to 6 ... Bg7 as in the first two games of the
chapter, as 6 ... Ne4 7 d5! looks dangerous if White is well prepared.

Game 30
N.Maisuradze-E.L’Ami
Maastricht 2014

1 d4 Nf6 2 c4 e6 3 Nf3 b6 4 g3 Bb7 5 Bg2 g6 6 0-0


A natural move, though I feel more problems can be posed with the 6 Nc3 and 7 d5
line from the previous games. Most players will just castle and play sensibly, and in
this game we get to see Black play the Hedgehog setup.
6 ... Bg7 7 Nc3 0-0
7 ... Ne4 will be examined in the next game. In this game Black allows 8 d5 which
would transpose to the first two games of this chapter (game 27-28), although here
White continues with quiet development.
8 Qc2

8 ... c5
This is logical as White’s d4-d5 thrust is not now supported by the white queen,
though Black’s move is a little risky – have I used that description too many times for a
Queen’s Indian Defence book?
9 Rd1
Or:
a) 9 d5 exd5 10 Nd2 was seen in A.Bird-J.Ikeda, Australian Championship 2008,
and now 10 ... Nc6 11 cxd5 Nd4 12 Qd3 Ng4 is a computer variation to try and
embarrass White’s queen. Black is certainly active here.
b) 9 dxc5 bxc5 10 Bf4 might be best, honing in on the d6-square. After 10 ... d5 11
Rfd1 Nbd7 White can choose between:
b1) 12 Ne5 Nb6 13 Na4 with pressure on the c5- and d5-pawns.
b2) 12 cxd5 exd5 13 Ne5 Nb6 14 a4 a5 which is also very unclear.
9 ... cxd4 10 Nxd4 Bxg2 11 Kxg2 Qc8

Question: What is the point of 11 ... Qc8?

Answer: We have seen this idea before. The queen eyes up the c4-pawn and wishes to find a launch pad on the
b7-square, where it keeps tabs on the important squares d5, d7 and helps a potential ... b5 push. It also looks down the
long diagonal at the white king.
12 b3 a6 13 Ba3 Qb7+ 14 Kg1 Rc8 15 Qd3 Nc6
Both sides complete their development in logical fashion.
16 Nxc6 Rxc6 17 Rac1 Rac8 18 h3 h5 19 h4
Around here there was now a little bit of cat and mouse, where both sides probe
each other, waiting to strike. Black would like to push ... d5 or ... b5, which are the
standard Hedgehog pawn breaks, but they must be timed perfectly.
19 ... Kh7 20 e3 Kg8 21 Kf1? b5! 22 Ne2
If the pawn is snatched, Black gets plenty of open lines and with a potential ... Qh1+
at some point, we now see the drawbacks of 21 Kf1. For example, after 22 cxb5 axb5
23 Qxb5 Qa8 24 Bb2 Ng4 White has too many threats to deal with on the c3-, f2- and
h1-squares.
22 ... Ng4 23 Nd4 bxc4 24 bxc4 Rb6
25 c5?
25 Kg1 e5 26 Nb3 Qf3 27 Rd2 was more resistant than the game.
25 ... Qh1+ 26 Ke2 Qg2 27 Rf1 Rbb8 28 Qd2 Ne5 29 Rg1 Qd5 30 Rgd1 Bf8 31
Qc2 a5 32 e4 Qa8
It feels as though the attack is beaten off, but now there is the weak c5-pawn to
contend with.
33 f4

33 ... Ng4?!
A natural human move to put the knight on the outpost, though our silicon friend
immediately sees a strong continuation, namely 33 ... Bxc5! 34 Bxc5 Rxc5 35 Qxc5
Qxe4+ 36 Kf2 Ng4+ 37 Kg1 Qe3+ 38 Kh1 Nf2+ 39 Kh2 Ne4! with the idea of ...
Qxg3+ and ... Nf2 with a mating attack.
34 Qd3 Re8 35 Nf3 e5 36 Rc4 exf4 37 gxf4 Qa6 38 e5 Be7 39 Rd4 Qe6 40 Rd2
Nh6 41 Rxd7 Rbc8
Suddenly White has won a pawn though the position still has lots of play. King safety
is always important and in time trouble situations, I would much rather have my king on
g8 than in the middle of the board here.
42 Rd6?
Now Black is winning again.
42 ... Qg4 43 Ke3 Nf5+ 44 Ke4 Bxh4 45 Nxh4 Qxh4 46 Qd5 Qg3 47 Re2 Nxd6+ 48 cxd6 Rc3 49 Qd2
Qf3+ 0-1

Game 31
R.Akesson-J.Speelman
European Team Championship, Pula 1997

1 d4
1 Nf3 b6 2 g3 Bb7 3 Bg2 g6 4 0-0 Bg7 5 d4 Nf6 6 c4 e6 7 Nc3 Ne4 was how the players actually reached our line.
1 ... Nf6 2 c4 e6 3 Nf3 b6 4 g3 Bb7 5 Bg2 g6 6 0-0 Bg7 7 Nc3 Ne4
This is a more solid alternative to 7 ... 0-0 that we saw in the previous game. Black
is aiming to get the b7-bishop onto the e4-square as soon as possible.


Question: Remind me we why we aren’t losing
something on h1-a8 diagonal after, say, 8 Ng5 here?

Answer: Because Black can move the e4-knight with tempo after 8 Ng5 Nxc3 9 bxc3 Bxg2 10 Kxg2 0-0 after
which he is fine.
8 Nxe4 Bxe4 9 Bg5 Qc8 10 Qd2 h6 11 Be3 d6
Jon Speelman improves on his own game where he held Vladimir Kramnik in a
rapid game which continued 11 ... Qb7 12 Rfd1 d5 13 Rac1 Nd7 14 cxd5 Bxd5 15 b3
Rc8 16 Ne1 Bxg2 17 Nxg2 Nf6 18 Bf4 Nd5 19 Be5 f6 20 Bf4 g5 21 Be3 0-0 and which
was dynamically equal in V.Kramnik-J.Speelman, PCA Grand Prix (rapid), Geneva
1996.

12 d5
The alternative 12 Rac1 Nd7 13 b4 Nf6 14 Qb2 0-0 15 Rfd1 Qb7 16 Bd2 b5! was
played in K.Ozturk-D.Kuljasevic, Skopje 2014 with the idea 17 cxb5 Qxb5 18 Rxc7
Nd5 19 Rcc1 Rab8 20 a3 a5 regaining the pawn.
12 ... Nd7 13 Nd4 Bxg2 14 Kxg2 Qb7 15 Nc6 Ne5 16 Nxe5 dxe5
16 ... Bxe5 17 Bxh6 0-0-0 gives Black some compensation for the pawn, but
Speelman judged this as unnecessary.
17 Rfd1 0-0-0 18 f3 Rd7 19 Bf2 exd5 20 cxd5

Question: What active plan should Black undertake here?

Answer: Attack through the centre with:
20 ... e4! 21 fxe4 Re8 22 Qc2
22 Bd4 Bxd4 23 Qxd4 Rde7 will lose at least the e4-pawn.
22 ... f5!
Thematically undermining White’s central structure.

23 exf5 Rxd5 24 Kg1 gxf5 25 Rac1 Rxd1+ 26 Rxd1 Qe4 27 Rd2 Qxc2 28 Rxc2
Re4
The ending is a little better for Black as his queenside pawns are more mobile.
29 Kg2 Kd7 30 Kf3 c5 31 b3 Kc6 32 h3 h5
Preventing the undermining move g3-g4.
33 Rc4 Kd5 34 Rxe4 fxe4+ 35 Kf4 b5
Here we see the mobile queenside pawns rolling down the board.
36 Kg5 c4 37 bxc4+ bxc4 38 Kxh5 Bd4 39 Be1 c3 40 Kg6 Kc4 41 Kf5 e3
The threat of ... c2 forces White’s next move.
42 Bxc3 Kxc3 43 Ke4 Kd2 44 Kf3 Ba1 45 h4 Bf6! 0-1
White will be in zugzwang very soon. He cannot move his king as the e2-pawn
drops, and he only has a finite number of pawn moves. It is clear to see why Jon
Speelman is renowned as a world-class endgame player.

Game 32
T.Andersson-P.Cramling
Swedish League 2014

1 d4 Nf6 2 c4 e6 3 Nf3 b6 4 g3 Bb7 5 Bg2 g6 6 0-0 Bg7 7 b3

White is developing sensibly, rather than going for any opening advantage.
7 ... 0-0 8 Bb2 d5
In this game we examine structures with ... d5, while 8 ... d6 9 Qc2 c5 is the
Hedgehog way to play. After 10 Nc3 Nc6 11 Rad1 Qe7 12 a3 Rac8 13 e3 cxd4 14 exd4
Na5 15 a4 was A.Ghasi-S.Tiviakov, British League 2011, and now Black could play 15
... d5 16 Nb5 dxc4 17 Ba3 Qe8 18 Bxf8 cxb3 19 Qd2 Qxf8 with a great position.
9 Nbd2 Nbd7 10 Rc1 c5
So now we have a sort of Queen’s Gambit Declined (QGD) or Catalan setup with
Black’s dark-squared bishop on g7 rather than e7. On the other hand, White does not
normally play b2-b3 and Nb1-d2 in those openings, so it could be argued Black has
reached a favourable version of this structure.

11 e3 Rc8 12 Qe2 Qe7 13 Rfd1 Rfd8 14 h3


Both sides develop before getting on with active plans.
14 ... cxd4
I like this move, as it makes White choose the pawn structure immediately.
15 exd4
15 Nxd4 Nc5 16 cxd5 Nxd5 17 Nc4 e5 18 Nb5 e4 attempting to play Nd3 at some
point.
15 ... Bh6 16 cxd5 Bxd5
In my opinion, Black should always be content with these positions. He has very
little chance of losing and a clear target to aim for (the d4-pawn). This is not an IQP
structure in the classic sense, as White hardly has active play and kingside attacking
chances as they normally would have.
17 Rxc8 Rxc8 18 Qd3 Qd6 19 Ne1 Bxg2 20 Nxg2 Nd5
All exchanges are welcome, as the d4-pawn is still weak.
21 a3 N7f6 22 Rc1 Rxc1+ 23 Bxc1 Qc6 24 Bb2 b5 25 f4 Nb6 26 Ne1 Bg7 27 Qc2
Qd7
Exchanging queens was also good, but maybe Cramling was willing to keep the
queens on to attack the weaknesses on d4, g3 and h3.
28 Nef3 Qc8 29 Qd3 a6 30 Ne5 Nbd5 31 Kf2 Qc7 32 Ke1 h6 33 Ne4 Nxe4 34
Qxe4 h5 35 h4 a5 36 Qe2 Qb7 37 Kd2 Ne7 38 Qf2 Nf5
A nice rerouting of the knight to a better square. From the f5-square it attacks two of
White’s weak pawns.
39 Nf3 Qd5 40 Kc2 a4 41 bxa4 bxa4 42 Kb1 Qe4+ 43 Ka1 Qd3
Black has definitely made progress. What a depressing position for White – he has
had no play all game, and just been outplayed.
44 Ka2 Bf8 45 d5
Desperation.
45 ... Qxd5+ 46 Ka1 Qd1+ 47 Ka2 Qb3+ 48 Ka1 Bxa3 49 Be5 Bb4 50 Ng5 Bc3+
51 Bxc3 Qxc3+ 52 Ka2 Qb3+ 53 Ka1 Qc3+ 54 Ka2 Nd4 55 Qb2 Qxb2+
Simplifying is always a good idea when ahead on material.
56 Kxb2 Ne2 57 Ne4 Kf8 58 Ka3 f5 59 Ng5 Ke7 60 Kxa4 Nxg3
No further comment required, this is an easily winning endgame.
61 Kb4 Ne2 62 Nh3 Kd6 63 Kc4 e5 64 fxe5+ Kxe5 65 Ng5 Nf4 0-1

Game 33
P.Svidler-A.Ipatov
Blitz World Championship, Dubai 2014

1 Nf3 Nf6 2 c4 b6 3 g3 Bb7 4 Bg2 g6 5 d4 Bg7 6 0-0 0-0 7 Re1


Another alternative to the 6 Nc3 and 7 Nc3 lines we saw earlier in the chapter.
7 ... e6 8 Nc3

Exercise: What is the best way to prevent the e2-e4 pawn advance?

Answer:
8 ... Ne4!
The thematic Queen’s Indian move.
9 Bd2 d6
Instead 9 ... c5 10 d5 Nxc3 11 Bxc3 Bxc3 12 bxc3 was played in F.Elsness-
J.Hammer, Norwegian Championship 2008. Unlike a typical Nimzo-Indian where
Black can go after the c4-pawn with ... Ba6, ... Nc6 and ... Na5, here the doubled c-
pawns are not as weak as they look. However in the meantime White can move forward
with e2-e4-e5 etc.
10 Rc1 Nd7 11 d5 Nxc3
11 ... Nxd2 bags the dark-squared bishop, but leaves White with the effective c3-
knight. After 12 Qxd2 Qe7 13 e4 a6 there is plenty of tension in the position.
12 Bxc3 Bxc3 13 Rxc3 e5
The pawn structure has been clarified.

14 Qd2 Kg7 15 e4 a5 16 h4 Nc5 17 h5 Bc8 18 Nh2 g5 19 f4 h6 20 Rf1 Qe7 21
Qe2 f6 22 Rf2 Bd7 23 Nf1 Rae8
Ipatov has employed the strategy of putting his pawns on squares of the opposite
colour to his bishop. He stands solidly in this roughly equal position.
24 Re3 c6
24 ... gxf4 25 gxf4 Rg8 was possible, trying to gain play down the g-file. Both kings
are potentially weak here.
25 dxc6 Bxc6 26 fxg5 fxg5 27 Rxf8 Rxf8 28 Qd1 Qf6 29 Rf3 Qe7 30 Rxf8 Qxf8
31 Qc2
Still this position should be OK for Black, though steps should be taken against Ne3-
f5.
31 ... Qa8
31 ... Ne6 32 Ne3 Nd4 33 Qf2 Qa8 was at least equal for Black. With the weakness
of the e4-pawn there is no reason to claim an advantage for White.
32 Qd2 Bxe4 33 Qxd6 Bxg2 34 Qg6+ Kf8 35 Qxh6+ Ke7 36 Ne3 Be4 37 Qxg5+
Kd6 38 Qf6+
38 ... Ne6?
38 ... Kd7 39 Ng4 Nd3 should defend, though it was a blitz game so time would have
been short by this point.
39 c5+ bxc5 40 Nc4+ Kc7 41 Qxe5+ 1-0
Despite the loss, Black more than held his own against an elite level opponent which
shows this variation is quite robust.

Game 34
M.Roiz-B.Savchenko
European Championship, Plovdiv 2008

1 d4 Nf6 2 Nf3 b6 3 c4 e6 4 g3 Bb7 5 Bg2 g6 6 Qd3


An unusual approach to prepare for the e2-e4 push.
6 ... Bg7 7 Nc3 d5
7 ... 0-0 was simpler and safer, for example 8 e4 c5 9 d5 (or 9 0-0 cxd4 10 Nxd4
Nc6) 9 ... exd5 10 cxd5 d6 and in both lines it is not clear that White’s queen is so well
placed on d3. This is accentuated after 11 0-0? Ba6 12 Nb5 Nxe4 winning a clear
pawn.
8 cxd5 Nxd5
8 ... exd5 9 Bg5 0-0 10 0-0 Nbd7 11 Ne5 with some pressure for White.
9 Bg5 Qc8 10 Nxd5 Bxd5
Instead if 10 ... exd5 then White has 11 h4 trying to get kingside play, given there are
hardly any black pieces in that sector of the board.
11 0-0 Qb7 12 Rfe1

12 ... f5
12 ... Be4 13 Qa3 Nc6 14 Rac1 and even if the threat of 15 Rxc6 and 16 Qe7# is
avoided, it would take a brave soul to want to choose Black in this position.
13 Qa3 Nc6 14 Rac1 h6?
14 ... Bf8 was the only move to defend. After 15 Qa4 b5 16 Qd1 Bb4 17 Bd2, if
Black now plays 17 ... Bd6 then the thematic break 18 e4! is strong, for example 18 ...
Bxe4 19 Rxe4 fxe4 20 Ng5 zoning in on those weak e4- and e6-pawns. The g2-bishop
will also be particularly strong here.


Exercise: Black’s position looks extremely rickety. Can you
see how the Israeli Grandmaster broke through here?

Answer:
15 Rxc6!
Now White is in charge.
16 ... Bf8
15 ... hxg5 was better but still lands Black in trouble after 16 Ne5! and if 16 ... Bxe5
17 Bxd5 exd5 18 Re6+ Kd7 19 Qe7+ Kc8 20 dxe5 and White is winning.
16 Rxe6+ Bxe6 17 Qe3

17 ... Qd5
Everything loses here anyway: 17 ... Kf7 18 Ne5+ and 17 ... Qc8 18 Bf6 Rh7 19 d5
are both horrible for Black.
18 Ne5 1-0
As we know, a small slip right out of the opening in these riskier variations can lead to catastrophe.
Chapter Six
Petrosian Variation: 4 a3 Ba6
1 d4 Nf6 2 c4 e6 3 Nf3 b6 4 a3 Ba6

It was Tigran Petrosian, the ninth World Champion, who first recommended the
move 4 a3 as a way to combat the Queen’s Indian Defence. However, it was only when
Garry Kasparov started playing it in the early 1980s that the variation was really put on
the map. Not only because one of the best players in the world was reinvigorating the
line with plenty of new ideas, but also the results. Kasparov built up a huge record with
the line, and was winning games in style to boot. Subsequently, players of the Queen’s
Indian had to find antidotes to the aggressive lines Kasparov was playing. I hope this
chapter will demonstrate some of those to allay your fears that this is some kind of
‘winning variation’!
What is the idea behind this little pawn move? It prevents the f8-bishop coming to
the b4-square, which means the b1-knight can come to c3 with impunity. Since we know
this knight covers the key e4- and d5-squares, it can therefore be argued that 4 a3 helps
control the centre. The drawback is that such a pawn move at this early stage of the
game does nothing to aid development and can be considered rather extravagant. Chess
surely cannot be so simple that the so called advantages listed above about 4 a3 mean
White gets a winning game? Like the winds of fashion in chess in general, new lines
come and go. Whilst this line is still played by top players, the dangerous ideas that
Kasparov introduced haven’t had the same effect in recent years. Study this line
carefully, and you should be fine!
In this chapter we cover my repertoire recommendation 4 ... Ba6. 4 ... Bb7 is also a
reputable main line, and in fact will usually transpose to the line covered in the next
chapter after 5 Nc3. Before getting into the modern theory of the Petrosian Variation,
we start this chapter with Game 35 which is a classic Kasparov game in this line.
From the above diagram position following 4 a3 Ba6, White’s main move is 5 Qc2
which is covered in Games 39-45. Before that we cover White’s 5th move alternatives
5 e3 (Game 36), 5 Qb3 (Game 37) and 5 Qa4 (Game 38).
After 5 Qc2 Bb7 6 Nc3 c5, White can try 7 dxc5 (Game 39), 7 d5 (Game 40) or the
most popular try 7 e4 whereby White initiates a Maroczy Bind structure. In the latter
continuation, following 4 a3 Ba6 5 Qc2 Bb7 6 Nc3 c5 7 e4 cxd4 8 Nxd4 the material is
further divided after 8 ... Bc5 9 Nb3 (Games 41-43) and 8 ... Nc6 9 Nxc6 which is
dealt with in Games 44-45.

Game 35
G.Kasparov-J.Van der Wiel
Amsterdam 1988

Let’s start by taking a look at one of Kasparov’s wins that helped enhance the
popularity of this line.
1 d4 Nf6 2 c4 e6 3 Nf3 b6 4 a3 Ba6 5 Qc2 Bb7 6 Nc3 c5 7 e4 cxd4 8 Nxd4 Bc5 9
Nb3 Nc6

10 Bg5
We will discuss this in further detail in the game Khenkin-Adams later in this
chapter.
10 ... a6
You will see that I will be recommending 10 ... h6 11 Bh4 Nd4 which I believe is a
safer way to play for Black.
11 0-0-0

Never one to shy away from a complicated game, Kasparov does not fear putting his
king on the c-file and posts the a1-rook actively, on the d-file aiming towards d7.
11 ... Qc7 12 Kb1 0-0-0
12 ... Be7 13 f4 Ng4 14 Bxe7 Nxe7 15 Qd2 Nf6 16 Bd3 d6 17 Rhe1 0-0-0 was
played in I.Novikov-A.Chernin, USSR Championship, Lviv 1987 which was at least
more of an attempt at play than the main game.
13 Qd2
13 f4 h6 14 Bxf6 gxf6 15 g3 Kb8 16 Bg2 was another way to play. Black, as in the
main game, is still quite cramped.
13 ... d6 14 f3
Rather than going for f2-f4, Kasparov prefers the apparently more modest setup with
f2-f3. However it is not modest in the respect that he will still attack, but in a different
manner – a pawn storm with g2-g4 and h2-h4 is on the cards.
14 ... h6 15 Bf4

15 ... Ne5
After 15 ... e5 16 Be3 and the d5-square is probably weaker for Black than the d4-
square is for White. Here ... Nd4 won’t be playable as long as White does not capture
on c5, which would allow ... bxc5 controlling the d4-square.
16 h4 Kb8 17 h5 Rd7 18 Rc1 Rc8 19 Be2 Ka7 20 Rhd1 Rdd8 21 g4 Ng8
Black is doomed to passivity and can only wait while White comes up with an
effective plan.
22 Bg3 Ne7 23 f4 N5c6 24 Bf3 Rb8 25 Bh4 Rd7 26 Nb5+
26 Nxc5 bxc5 27 Bf2 was also playable, but Kasparov always goes for the sharpest
continuation.
26 ... axb5 27 cxb5 Na5
27 ... Nd8 is well met by 28 Nxc5 bxc5 29 Rxc5! dxc5 (or 29 ... Qxc5 30 Qa5+ Ba6
31 Qxa6#) 30 Qxd7 Qxd7 31 Rxd7 and White regains the piece with a winning
position.
28 Nxa5 bxa5

Exercise: What is the winning tactic for White here?

Answer:
29 Rxc5!
29 b4 is also good, but the text is far cleaner.
29 ... dxc5
29 ... Qxc5 30 Qxa5+ with imminent mate.
30 Qxd7 Qxf4 31 Rd6! 1-0
And Black resigned as 32 Ra6 mate is threatened, and any attempt to escape is
rebuffed. For example 31 ... Rc8 32 Ra6+ Kb8 33 Bxe7 Bxe4+ 34 Ka2! with 35 Bd6+
and 35 Qa7# as deadly threats. It is understandable that with games like this, the 4 a3
variation was highly regarded.

Game 36
Z.Kozul-Z.Efimenko
European Club Cup, Kemer 2007

1 d4 Nf6 2 c4 e6 3 Nf3 b6 4 a3
4 ... Ba6
This is my repertoire recommendation against 4 a3.

Question: What is the point of this, since in
some lines the bishop will drop back to b7?

Answer: Just like in the 4 g3 Ba6 variation, this bishop move places unwelcome pressure on the c4-pawn because
most moves to defend this pawn will be some sort of compromise.
5 e3
This shows one difference compared to 4 g3, as the f1-bishop can defend the c4-
pawn like this. However, because White is ‘slowplaying’ the central e2-e4 push, Black
is able to get an acceptable game.
5 Nbd2 is a sideline which shows another way to protect the c4-pawn. After 5 ... d5
6 e3 c5 transposes to the main game, although of course there are other options.
5 ... d5
5 ... c5 is certainly playable, though with the queen still on d1 there are always d4-
d5 pushes to be concerned about. After 6 Nc3 Be7 White has tried:
a) 7 Bd3 cxd4 8 exd4 d5 9 cxd5 Bxd3 10 Qxd3 Nxd5 11 Qb5+ Qd7 12 Nxd5 exd5
13 Qxd7+ Nxd7 14 Bf4 Rc8 15 0-0 f6 16 Rfc1 Kf7 which was completely equal in
M.Hebden-C.Ward, British Championship 1996.
b) 7 d5! exd5 8 cxd5 Bxf1 9 Kxf1 d6 10 g3 0-0 11 Kg2 Nbd7 12 Re1 a6 13 a4 Ne8
14 e4 Bf6 15 Bf4 and White looked to a have a much better Benoni position, which got
worse for Black after 15 ... Ne5 16 Nxe5 dxe5 17 Be3 Nd6 18 a5! Nc4 19 axb6 Nxe3+
20 Rxe3 Qxb6 21 Na4 V.Kramnik-L.Ljubojevic, Amber blindfold, Monaco 2002. This
is why I have gone with the main game continuation of 5 ... d5 instead.
6 Nbd2
After 6 cxd5 it is important to play 6 ... Bxf1 first (not 6 ... exd5? since 7 Bxa6 Nxa6
8 Qa4+ wins a piece) 7 Kxf1 Qxd5 8 Nc3 Qb7 9 b4 Nbd7 with a standard position
except for the king on f1, as seen in P.Vernersson-J.Hellsten, Swedish League 2001.

6 ... c5
Not the only move, but if we can get away with this then why not? Others:
a) 6 ... Be7 is actually more common, to delay the ... c5 advance until later. Then 7
b4 0-0 8 Bb2 Nbd7 9 cxd5 Bxf1 10 Nxf1 exd5 11 Ng3 Re8 12 Ne5 Bd6 13 f4 c5 14 0-
0 c4 with interesting play for both sides, was M.Krasenkow-A.Giri, Spanish League
2011.
b) 6 ... c6 7 Be2 Be7 8 0-0 0-0 9 b3 Nbd7 10 Bb2 Rc8 11 b4 dxc4 12 Nxc4 c5
appeared adequate for Black, as after 13 Qa4 Bxc4 14 Bxc4 cxb4 15 Ba6 bxa3 16
Bxa3 Rc7 he had even managed to win a pawn in R.Kasimdzhanov-V.Anand, Wijk aan
Zee (blitz) 1999.
7 dxc5
7 b3 cxd4 8 exd4 g6 with the idea of fianchettoing the dark-squared bishop in order
to target the d4-pawn. From here 9 Bd3 Bg7 10 0-0 0-0 11 Bb2 Nc6 12 Qe2 Rc8 13
Rfd1 dxc4 14 bxc4 Qc7 15 Rac1 Rfd8 16 Nb3 Qf4 was played in S.Swapnil-E.Postny,
New Delhi 2011 and Black’s chances were no worse.
7 ... Bxc5 8 Qa4+
Alternatively 8 b4 Be7 9 Bb2 0-0 10 Bd3 Nbd7 11 0-0 Bb7 12 Qb1 h6 13 Rd1 a5
14 cxd5 Bxd5 15 e4 Bb7 16 Bb5 Qc7 17 Rc1 Qd8 18 Rd1 Qc7 19 Rc1 Qd8 20 bxa5
Rxa5 21 a4 Nc5 22 Rd1 Qa8 looked quite nice for Black in B.Grachev-A.Saric, Rijeka
2008.


Question: What is the best way to deal with this check?

Answer: The best is to offer a queen swap, as alternatives leave Black worse.
8 ... Qd7
The alternatives are:
a) 8 ... Nfd7 9 cxd5 Bxf1 10 Kxf1 exd5 11 b4 Be7 12 Qb5! and the d5-pawn falls.
b) 8 ... Kf8 can hardly be taken seriously as there are no positives to offset the lack
of ability to castle.
9 Qxd7+ Nbxd7 10 cxd5
10 b4 Be7 11 Bb2 0-0 12 Rc1 Rac8 Is also hardly a threat.
10 ... Bxf1 11 Kxf1
11 dxe6 Bxg2 12 exd7+ Kxd7 13 Rg1 Bxf3 14 Nxf3 g6 15 b4 Bd6 16 Bb2 Ke7 17
Nd4 Rhc8 might even be better for Black, as the h2-pawn is hanging in some variations
and Black has the open c-file already.
11 ... Nxd5


Question: Every opening repertoire book from Black’s viewpoint tells me
that White can make a draw in some variations. Is this one of them?

Answer: Not necessarily. To play for a draw (as I believe White is here) especially with the white pieces is very
negative, and can lead to your position drifting. From Black’s point of view, we have nothing to fear, and on a positive
note, everything to gain by pushing on!
12 Ke2 Be7
I will label this ‘the sort of Grandmaster move’ that club players tend to not
understand. Why are we going backwards? Well, it is prophylaxis. White would like to
activate his bishop with b2-b4 and Bc1-b2, but this is not good now after 12 ... Be7, so
White needs to seek other avenues to develop the c1-bishop.
13 Nb3
After 13 b4 Bf6 14 Nd4 (or 14 Rb1 Nc3+) 14 ... Bxd4 15 exd4 Rc8 Black has a
clear target on d4 to aim at, the d5-knight is excellently placed, and the open c-file is
again in Black’s possession. So Black is better here for these reasons.
13 ... Rc8 14 Bd2 f5 15 Nbd4 Kf7 16 Rac1 a6 17 h4 h6 18 h5 Bf6 19 b4 b5
Black pursues a light-squared strategy. Capablanca was the first one to explain that
you should put your pawns on squares of the opposite colour to your own bishop, and
this ends up being the crucial element in this game.

20 Rc6 Rxc6 21 Nxc6 Rc8 22 Rc1 N5b6 23 Rc2 Nc4 24 Na5 Ndb6 25 Nxc4 Rxc4
26 Rxc4 Nxc4 27 Bc1
It is worth playing through the full game to see how Black proceeds to win this
endgame, as Nimzo and Queen’s Indian players could get these kinds of pawn structures
in their games.
27 ... e5 28 Nd2 Nxd2 29 Kxd2 e4
Pawns on light squares again!
30 Kc2 Bh4 31 g3 Bf6 32 Bd2 g6
32 ... Ke6 bringing the king in is also good, for example 33 Bc3 Bxc3 34 Kxc3 Ke5
35 Kb3 Kf6 36 Kc3 Kg5 37 Kd4 Kxh5 38 Kc5 g5 39 Kb6 Kg4 40 Kxa6 h5 41 Kxb5 h4
and Black queens first.
33 hxg6+ Kxg6 34 Bc3 Bd8 35 Be5 Kg5 36 Kd2 h5 37 Ke2 Kg6!
The king notices the route f7-e6-d5-c4 as entry into the position. White is quite
helpless to prevent this.
38 Bd4 Kf7 39 f4 exf3+ 40 Kxf3 Bc7 41 Bc3 Ke6 42 Be1 Kd5 43 Bf2 Kc4 44 e4
fxe4+ 45 Kxe4 Kb3 46 Kd5 Kxa3 47 Kc6 Bxg3! 48 Bxg3 Kxb4 49 Kd5 Kb3 0-1
White cannot deal with all the pawns on both sides. So if White plays insipidly and
aims for a draw, it does not mean he gets it.

Game 37
A.Istratescu-J.Werle
La Roche sur Yon 2006

1 d4 Nf6 2 c4 e6 3 Nf3 b6 4 a3 Ba6


4 ... c5 5 d5 Ba6 6 Qb3 was how the game actually went.
5 Qb3
Another way to defend the c4-pawn.
5 ... c5
Let’s take a look at Black’s alternatives at move 5:
a) 5 ... Nc6 (trying to replicate the 4 g3 Ba6 5 Qb3 Nc6 idea from Chapter Two)
doesn’t look good here in my opinion, since White has not committed to g2-g3 yet and
so the f1-bishop can defend the c4-pawn. After 6 Qc2 Na5 White has the aggressive
option 7 e4, defending the c4-pawn and advancing quickly in the centre. Following 7 ...
Bb7 8 Nc3 Black has tried:
a1) 8 ... d5 9 cxd5 exd5 10 e5 Ne4 11 Bb5+ c6 12 Bd3 Nxc3 13 bxc3 g6 14 h4
which looked horrible to defend for Black in B.Maksimovic-Z.Zelic, Yugoslav League
1990.
a2) 8 ... c5 9 d5! is always the move White would like to get in; it provides a space
advantage and leaves the a5-knight stranded out of play. K.Sasikiran-Yu Yangyi, Asian
Nations Cup, Tabriz 2014, continued 9 ... exd5 10 exd5 Be7 11 Bd3 Ba6 12 b3 0-0 13
0-0 Nb7 14 Re1 Nd6 15 Bf4 Re8 16 h3 Bf8 17 Be5 g6 18 Re3 Nh5 19 Rae1 Ng7 and
now the rest of the game is extremely instructive by Indian Grandmaster Krishnan
Sasikiran: 20 g4! Bb7 21 Qd2 a6 22 a4 f5 23 Bxd6 Rxe3 24 Qxe3 Bxd6 25 Qh6 and
suddenly Ng5 and threats of mate are in the air. The game finished 25 ... Be7 26 gxf5
Nxf5 27 Bxf5 gxf5 28 Kh1! Bf6 29 Rg1+ Kh8 30 Nh4 1-0.
b) 5 ... Be7 6 Nc3 d5 is a solid line for Black if you don’t like this Benoni type
structure.
Analysis Diagram

We have seen ... Be7 and ... d5 against Nb1-d2 lines, and Black argues that in this
case the queen on b3 is also not well placed. After 7 cxd5 and now either recapture is
playable:
b1) 7 ... Nxd5 8 e4 Bxf1 9 Kxf1 Nxc3 10 Qxc3 0-0 11 Bf4 c5 and a draw was
agreed soon after in A.Dreev-M.Adams, FIDE World Championship, Las Vegas 1999.
b2) 7 ... exd5 8 Bg5 c6 9 Bxf6 Bxf6 10 e4 Bxf1 11 Rxf1 dxe4 12 Nxe4 0-0 13 0-0-0
Nd7 14 Rfe1 Qc7 was equal in D.Jojua-H.Banikas, European Championship, Plovdiv
2008.
6 d5 exd5 7 cxd5
7 ... g6
7 ... d6 8 Nc3 g6 might be a better move order, since 7 ... g6 allows some tricks
(which Black can deal with, I should point out). White continues 9 Bf4 Bg7 and now:
a) 10 e4 Bxf1 11 Kxf1 0-0 12 h3 Re8 13 Re1 a6 14 a4 Ra7 15 g4 Rb7 16 Kg2 h5 17
g5 Nh7 18 Qc4 Qd7 19 e5 b5 was hugely unclear in M.Mchedlishvili-Z.Efimenko,
Dresden Olympiad 2008, although no more unclear than the main game continuation.
b) 10 Qa4+ is one reason why ... d6 was delayed in the main game, as this check can
be awkward to meet. 10 ... Nbd7 is ruled out as the a6-bishop would be hanging.
Instead Black can try one of the following lines:
c) 10 ... b5 11 Nxb5 0-0 when Black gets a lead in development as compensation for
the pawn. 12 Nc3 Qb6 13 Qc2 Nbd7 14 e4 Rfe8 15 Nd2 Nxe4 16 Ncxe4 f5 17 Bc4 Nf6
18 0-0 Nxe4 19 Rfe1 Nxd2 20 Bxa6 Nf3+ 21 gxf3 Qxa6 22 Qd2 Bd4 23 Rxe8+ Rxe8
24 Re1 Qc8 was equal in B.Khotenashvili-J.Werle, Rethymnon 2010.
d) 10 ... Qd7 11 Bxd6 Qxa4 12 Nxa4 Nxd5 13 0-0-0 Ne7 14 e4 Bxf1 15 Rhxf1 Nbc6
16 Nc3 was played in G.Kasparov-S.Gligoric, Bugojno 1982, and now eliminating the
c3-knight to prevent Nc3-b5 ideas with 16 ... Bxc3 17 bxc3 Rd8 looks OK for Black.
8 Bf4 d6 9 Qe3+
9 Nc3 Bg7 10 e4 would transpose to 7 ... d6 above.
9 ... Be7 10 Nc3 0-0 11 Qd2 Re8 12 g3 Bf8 13 Bg2

Exercise: What should Black do here: a) 13 ... b5, b) 13 ... Ne4 or c) 13 ... Nbd7?

Answer: They are all playable, but Black does best to try and seize the initiative with:
13 ... Ne4
In these Benoni type pawn structures, it is often a good idea to get in ... Ne4 and
exchange knights if possible. The alternatives are:
a) 13 ... Nbd7 14 0-0 and now ... Ne4 or even ... c4 and ... Nc5 plans to invade the
b3-square (or maybe the d3-square) are possible. For example, 14 ... c4 15 Nd4 Nc5
16 Qc2 Qd7. White is dissuaded from playing e2-e4 as the d3-square becomes juicy for
the knight to get onto.
b) 13 ... b5 trying to get the queenside play going, for example 14 b4 Nbd7 15 0-0
Rc8 16 Rac1 Nb6 does not look to bad either.
14 Nxe4 Rxe4
Now the e2-pawn is threatened, a by-product of exchanging off the c3-knight.
15 Be3 Qe7 16 h4 Nd7 17 h5 Re8 18 hxg6 hxg6 19 Ng5

Question: I don’t like the look of this for Black.
Open h-file, the e4-rook is short of squares ...

Answer: As Benoni players will know, there are dynamic features to counter these other potential ‘issues’. The
open h-file is not enough in itself to give checkmate, and as for the rook on e4, he has his own ideas ...
19 ... Rxe3!
An excellent exchange sacrifice. At least a pawn or two is regained as well, and the
d7-knight soon finds a great outpost on the e5-square since the white f-pawn will
disappear.
20 Qxe3 Qxe3 21 fxe3 Rxe3 22 0-0
22 Bf3 doesn’t really help protect e2-pawn as Black can play 22 ... Ne5.
22 ... Ne5 23 Ne4 f5
Going after the g-pawn rather than the e-pawn with 23 ... Rxe2 24 Rf2 Re3, which
was also good for Black.
24 Nc3 Rxg3 25 a4 Bc4 26 Ra3 Be7 27 Rfa1 a5 28 Nd1 Rxa3 29 Rxa3 Bxe2
Now there are three pawns for the piece, though technical difficulties remain.

30 Nc3 Bc4 31 Bf1 Kf7 32 b3 Bxf1 33 Kxf1 g5
Those extra pawns will come in handy, and they begin to roll down the board.
34 Ra2 g4 35 Rh2 Kg6 36 Rh8 f4 37 Re8 Kf7 38 Rb8 f3 39 Rxb6
Finally the rook got active, but it is too late.
39 ... Kg6 40 Ne4 Kf5 41 Nxd6+ Bxd6 42 Rxd6 g3 43 Rd8 Ng4 44 Rf8+ Ke5! 0-1
The f3-pawn is immune due to a ... Nh2+ trick, and the d5-pawn is falling.
So whilst we are studying the Queen’s Indian, transposition to other types of pawn
structures (such as the Benoni structure in this game) are useful to know and can occur
across a number of opening setups.

Game 38
A.Vragoteris-A.Sokolov
Greek League 2003

1 d4 Nf6 2 c4 e6 3 Nf3 b6 4 a3 Ba6 5 Qa4



As we saw in chapters on 4 g3 Ba6, White has multiple ways to defend the c4-pawn.
5 ... Bb7 6 Nc3 c5
Black’s options were limited as the d-pawn is pinned, but we are generally happy to
get in the ... c5 pawn push if White cannot successfully counter with d4-d5.
7 dxc5
Another drawback of the queen missing from d1 is that the pressure on the d4-pawn
needs to be resolved. An alternative is 7 e3 Be7 8 Be2 0-0 9 0-0 cxd4 10 Nxd4 a6 11
Rd1 Qc7 12 Qc2 d6 13 e4 Nbd7 14 b3 Rac8 15 Bb2 Qb8 which was played in
A.Franco Alonso-R.Pogorelov, Spain 1997. Here Black has a nice Hedgehog structure
and will look to play ... b5 or ... d5 at the opportune time.
7 ... bxc5
7 ... Bxc5 is completely fine too, and the choice between the two options is a matter
of taste.
8 Bg5 Be7 9 e3 0-0
Both sides now quietly complete their development.
10 Be2 d6 11 Qc2 Nbd7 12 0-0 Qb6
Andrei Sokolov is very well versed in the arts of the Queen’s Indian structures, and
with simple play he gets a very nice position. At the right moment, he will play ... d5
and end up with a favourable hanging pawns structure after c4xd5 and ... e6xd5. More
on that later in the game ...
13 b4 h6
13 ... cxb4 14 axb4 Qxb4? 15 Rab1, winning the b7-bishop, was the trick.
14 Bh4 cxb4 15 axb4 a5! 16 b5
16 bxa5 Rxa5 17 Rxa5 Qxa5 18 Rb1 Nc5 19 Nd4 Ra8 is always a touch better for
the second player. How weak is the c4-pawn? Time will tell, but for the whole of the
game it is a weakness White will have to worry about.
16 ... Rfc8 17 Rfc1

Exercise: Does Black have any potential c-file play here?
Find a good move and follow up for Black.

Answer: Waiting moves are acceptable, but if there is an active plan then this should be played.
17 ... d5 18 cxd5 Bb4!
If 18 ... Nxd5 then 19 Nxd5! attacking the queen on b6. 19 ... exd5 20 Qb2 and we
didn’t want an isolated pawn on d5 blocking in our b7-bishop.
19 Qb2 Nxd5 20 Na4 Qa7 21 Nd4 N7b6
Objectively this position is still close to equal, but as we have already seen in the
games earlier in this book, that does not mean a draw.
22 Nxe6?
This must have come as a shock, but possibly White was getting slightly bored and
decided to go for mate!
Instead after 22 Nxb6 Qxb6 23 Bc4 Bd6 24 Bg3 a4 25 Bxd6 Qxd6 26 Ra3 Nb6 and
Black is upping the ante.
22 ... Rxc1+ 23 Rxc1 fxe6 24 Qe5
I can only assume White went for this as he saw the black major pieces languishing
on the queenside, away from the king.
24 ... Nxa4
24 ... Re8 defends the e6-pawn and appears to do the job.
25 Qxe6+ Kh8 26 Qg6
26 ... Nc5
It is vital to counter 27 Bd3.
27 Bd3 Nxd3 28 Rc7 Nxc7 29 Bf6
This is the chess equivalent of ‘throwing the kitchen sink’ at your opponent.
29 ... Bf8
29 ... Ne6 30 Qxh6+ Kg8 and White has run out of pieces!
30 b6 Qxb6
30 ... Qa6 was even simpler, but Sokolov is so many pieces ahead he can even
sacrifice his queen.
31 Bxg7+ Bxg7 32 Qxb6 a4 33 Qxc7 Rc8 0-1

Game 39
A.Ledger-E.Alekseev
European Club Cup, Kallithea 2008

1 d4 Nf6 2 Nf3 e6 3 c4 b6 4 a3 Ba6 5 Qc2


Now we will look at the main lines of the Petrosian variation, which begin with 5
Qc2. This is unsurprising given that this move was also a key line in Chapter Two with
4 g3 Ba6 5 Qc2.
5 ... Bb7

Question: Do remind me why we ‘lost’ this tempo
again, with 4 ... Ba6 and then back with 5 ... Bb7?

Answer: As you’ll see, once again it is all about the e4- and d5-squares. 5 Qc2 controls the e4-square but loosens
White’s grip on the d5-square.
6 Nc3 c5
Now if White’s queen was on d1, then d4-d5! would give White a nice space
advantage.
7 dxc5 bxc5 8 Bf4
White’s positional idea is to home in on the d6-square, which he believes is a
weakness. This is why I like Black’s next two moves, which appear to deal with this
plan.
8 ... d6 9 Rd1 Qb6 10 e3
10 Nb5 is slightly annoying, but it can be dealt with by 10 ... e5 11 Bg5 Nbd7 12 e3
h6 13 Bh4 Be7 and although there appears to be a gaping hole for a white knight on the
d5-square, this is not easily to exploit, especially given the c3-knight moved away to
b5.
10 ... Be7 11 Be2 0-0 12 0-0 Rd8
Both sides focus on the battle for the d6-square.
13 Rd2 Nbd7 14 Rfd1 Nf8
Black is in time to complete the ... Nb8-d7-f8 knight manoeuvre while keeping the
d6-pawn defended.
15 h3 h6 16 e4!?

Exercise: What plan should Black undertake here?

Answer: White’s last move has weakened the d4-square, and now the f8-knight can join the game in style.
16 ... e5! 17 Be3 Ne6
Black’s e6-knight is now ready to jump to the d4-square.
18 Nd5 Nxd5 19 exd5 Nd4 20 Bxd4 cxd4 21 Qe4 Qc7 22 Bd3 g6 23 Qg4 Kg7 24
Qg3 a5
Trying to prevent the b2-b4 advance. Black starts to outplay his opponent over the
next few moves.
25 h4 Bf6 26 Re1 Bc8 27 h5 g5 28 Rc2 Rb8 29 Rec1 Bd7 30 c5 Ba4 31 cxd6
Qxd6
A sort of ‘win or bust’ position for White. The d5-pawn or b2-pawn will drop, but
there are some chances for a kingside attack for White here.
32 Nh4 Bd7 33 Rc5?
Black is already slightly better, but this allows a winning tactic.
33 ... Rb3!
Pinning the d3-bishop to the white queen.
34 Rxa5 Kf8!
Calmly played. Now there is no pin on the g-file, and the h4-knight is trapped!
35 Qf3 gxh4 36 Ra6 Qxa6
36 ... Rb6 37 Rc6 Rxc6 38 dxc6 Bc8 retains the extra piece as well, but the game
move is even stronger.
37 Rc6
37 Bxa6 Rxf3 38 gxf3 Ke7 and the extra piece should tell eventually.
37 ... Qxd3 38 Qxf6 Qb1+ 39 Kh2 Qf5 40 Qxd8+ Kg7 41 Rc7 Qf4+ 0-1

Game 40
F.Bruno-A.Kovalyov
Sort 2008

1 Nf3 Nf6 2 d4 e6 3 c4 b6 4 a3 Ba6 5 Qc2 Bb7 6 Nc3 c5 7 d5



Question: Hold on a minute, you said the idea of deflecting
the queen to c2 meant White could not play this move?

Answer: Well I did not say he ‘could not’ do it, but it loses its strength. In fact here it is a pure pawn sacrifice, and
one that can be quite dangerous.
7 ... exd5 8 cxd5 Nxd5 9 Bg5
9 Qe4+ Qe7 10 Qxd5 is an interesting queen sacrifice variation that I once had
played against me in an internet game. It is the speciality of the creative Dutch IM
Manuel Bosboom. The line continues 10 ... Bxd5 11 Nxd5 Qd8 12 Bg5 f6 13 Bf4 Nc6
and here:
a) 14 Nc7+ Kf7 15 Nxa8 Qxa8 16 e4 Na5 and Black still keeps a material
advantage.
b) 14 e4 d6 15 Bb5 Rc8 16 h4 Kf7 17 Bc4 Na5 18 Ba2 and now Black has the key
move 18 ... c4! to block out the a2-bishop. After 19 0-0 Be7 20 Nd4 Re8 21 Rad1 Bf8
22 Rfe1 Kg8 Black had successfully regrouped in M.Bosboom-V.Anand, Wijk aan Zee
(blitz) 1999.
9 ... Be7

10 Qe4
Now the heat is turned up. Some of the following moves will appear strange, but
after avoiding all the pitfalls Black should emerge with a good position. White has also
played:
a) 10 Nb5 and Black has in turn tried:
a1) 10 ... d6 was Michael Adams’ choice here, so it should be taken seriously. 11
Bxe7 Nxe7 12 0-0-0 Nc8 13 e3 a6 14 Nc3 b5 15 Bd3 Qe7 16 Be4 Ra7 ½-½ was
J.Rowson-M.Adams, Turin Olympiad 2006.
a2) 10 ... 0-0 11 Nd6 Ne3!? 12 fxe3 Bxf3 13 exf3 (13 Bxe7 Qxe7 14 Nf5 Qe4 is
good for Black) 13 ... Bxg5 14 Bc4 Nc6 and White has some compensation for the
pawn, but there are also dangers lurking. For example 15 f4? Bxf4! 16 Nxf7 Qh4+ 17
g3 Bxg3+ 18 Kd2 Bf2 19 Kd1 d5 0-1 was a quick win for Black in L.Janjgava-
A.Panchenko, USSR Championship, Lviv 1987.
b) 10 Ne4 0-0 11 0-0-0 Na6 12 h4 f6 13 Bd2 Nac7 14 e3 Ne6 15 Bc3 f5 16 Neg5
Bxg5 17 Nxg5 Nxg5 18 Bc4 Ne4 19 Bxd5+ Bxd5 20 Rxd5 Qe7 21 Be5 Rad8 was
played in V.Topalov-K.Georgiev, Bulgarian Championship, Sofia 1989. White has
some compensation for the pawn, but I would rather be Black.
10 ... Nxc3 11 Qxb7 Nc6 12 Bxe7 Kxe7
The only move, as the queen is required to defend the a8-rook. It looks like Black is
just losing a piece here, but the queen on b7 is about to get itself trapped ...

13 Qa6
White alternatives at this point are:
a) 13 bxc3 Rb8 14 Qa6 b5 which transposes to the game.
b) 13 Ne5 Nxe5 14 bxc3 Nc6 15 Qa6 Re8 16 Qd3 which was played in
B.Tuvshintugs-I.Sukandar, Asian Women’s Championship, Ho Chi Minh City 2012, and
now shielding the h7-pawn with 16 ... g6 17 g3 Kf8 18 Bg2 Qe7 should be fine for
Black.
13 ... b5 14 bxc3 Rb8 15 a4
15 e4 Rb6 16 Qxb5 Rxb5 17 Bxb5 Qa5 18 Bxc6 Qxc3+ 19 Ke2 dxc6 just looks bad
for White, with no compensation for the sacrificed queen.
15 ... Rb6 16 Qxb5 Rxb5 17 axb5 Qb8
An interesting piece sacrifice, given that White has no development.
After 17 ... Na5 18 e3 Qb6 19 Be2 Rc8 20 0-0 c4 21 Nd4 White is quite solid and it
might be difficult for Black to translate this position into a win.
18 bxc6 Qb2 19 Rd1 Qxc3+ 20 Nd2 dxc6 21 h4 Rd8 22 Rh3 Qc2 23 Ra3 c4 24
Rxa7+ Kf8 25 Raa1 c3 26 Nc4 Rxd1+ 27 Rxd1 Qb3 28 Ne3 c2 29 Nxc2 Qxc2
Black emerges with a winning advantage which he eventually converts.

The remaining moves were:
30 e3 Qc3+ 31 Rd2 Qc1+ 32 Rd1 Qa3 33 Rd4 Qc1+ 34 Rd1 Qc3+ 35 Rd2 c5 36 Bd3 Qc1+ 37 Ke2 c4
38 Be4 Qc3 39 f4 g6 40 Rc2 Qb3 41 g4 Ke7 42 g5 Kd6 43 Rd2+ Kc5 44 Rd7 Qa2+ 45 Rd2 Qa6 46 Rc2
Kb4 47 Rb2+ Kc3 48 Rd2 Qe6 49 Rd4 Kb2 50 Rd2+ Ka3 51 Rd4 Kb2 52 Bd5 Qg4+ 53 Bf3 Qc8 54 Rd2+
Ka3 55 Be4 c3 56 Rc2 Kb4 57 Rc1 Qg4+ 58 Bf3 Qf5 59 h5 Kb3 60 hxg6 hxg6 61 Kf2 Kb2 62 Re1 c2 0-1

Game 41
T.Radjabov-M.Adams
Prague (rapid) 2002

1 d4 Nf6 2 c4 e6 3 Nf3 b6 4 a3 Ba6 5 Qc2 Bb7 6 Nc3 c5 7 e4

This is the most natural and popular move, just getting on with central play. We’ll be
covering this line in depth in the remaining games in this chapter.
7 ... cxd4 8 Nxd4 Bc5
White has e4-e5 as a potential threat and Black must try to counter in the centre.
Alternatively, 8 ... d6 and going for a Hedgehog setup is fine, but getting our dark-
squared bishop into a more active role if we can is always a good idea.

Exercise: What should Black do now after the natural looking 9 Be3?


Answer: This is positionally suspect as after 9 Be3 Ng4! the d4-knight and e3-bishop are looking a little shaky,
even at this early stage. White can now play:
a) 10 Qd2 Nxe3 11 Qxe3 and now Black has 11 ... e5 winning a piece, which
exploits the dark-squared strategy started with 8 ... Bc5.
b) 10 b4 Nxe3 11 fxe3 Be7 12 Nf3 Nc6 13 b5 Na5 14 Bd3 Rc8 was fantastic for
Black in V.Mikhailovsky-N.Korostelev, Omsk 2008.
Instead, in the main game, White played:
9 Nb3

9 ... Nc6 10 Nxc5 bxc5

Question: You just spoke about the dark squares, so why did we allow this exchange?

Answer: The idea is to follow up with ... e5 and ... Nd4, dropping the c6-knight into a superb outpost.
11 Bd3 0-0 12 0-0 d6 13 f4 h6 14 Qd1 Rb8 15 Be3 Re8 16 Rb1 a5


Question: What is the point of Black’s last move 16 ... a5?

Answer: To clamp down on any b2-b4 pawn breaks which may open up the position for White’s bishops.
17 Nb5 e5 18 f5 Nd4
The knight reaches an excellent outpost. If White exchanges it off, he will give Black
a protected passed pawn in the centre.
19 Nc3 Bc6 20 Rf2
20 b3 Rb6 21 Nd5 Bxd5 22 cxd5 Qb8 23 Bc2 is equal but I would much rather be
Black, as the knights are better than bishops in closed positions.
20 ... Rb6 21 g4
Going for the attack. 22 g5 is a threat, which Adams seeks to defend against.
21 ... Nh7
21 ... Qe7 22 g5 hxg5 23 Bxg5 allows White some pressure.
22 Rg2 Ng5
22 ... Qh4 was better, for example 23 Nd5 Bxd5 24 cxd5 Ng5 25 Kh1 Reb8 when
Black has attacking chances on both sides of the board.
23 Rg3
23 h4 doesn’t win the knight because of 23 ... Ngf3+ 24 Kf1 Qxh4, and indeed
backfires as ... Qh1+ is a big threat.
23 ... Qa8 24 Bxg5
24 Nd5 might have been better.
24 ... hxg5 25 Qd2

25 ... Qd8
25 ... f6 26 Qe3 Reb8 27 Rg2 Rb3 28 Rd2 a4 29 Qf2 Qb7 is much easier for Black
to play – what is White going to do for an active plan here, since he is tied down to the
defence of the b2-pawn?
26 Rh3 Qe7 27 Ne2 Rb3
27 ... Reb8 is sensible, to attack the b2-pawn.
28 Nc1
28 Nxd4? exd4 and now the e4-pawn needs protecting.
28 ... Rb6 29 Qxa5 Qb7 30 Qd2 f6
The human move, to defend the g5-pawn and prevent the white queen getting in.
30 ... Bxe4 was playable, but needed to be calculated. After 31 Qxg5 Bh1! 32 Bf1
Nf3+ 33 Rxf3 Qxf3 34 Nd3 f6 35 Qg6 Re7 wins, but this was unnecessary, given this
was a rapid game.
31 Re3?
31 Ne2! Bxe4 32 Nxd4 exd4 33 b4 Re5 was White’s only try, but Black has regained
the pawn and is better.


Exercise: What should Black do now?

Answer: There is a tactic in the air, based on the idea of ... Nf3+ forking king and queen.
31 ... Bxe4! 32 Qf2
32 Bxe4 Qxe4 33 Qd3 (or 33 Rxe4 Nf3+ 34 Kg2 Nxd2 forks both rooks) and now
33 ... Qxg4+ mops up the kingside pawns.
32 ... Bc6 33 h4 Nf3+ 34 Rxf3
34 Kf1 Nxh4 wins another clear pawn.
34 ... Bxf3 35 hxg5 fxg5 36 Qe3 e4 37 Bf1 Qe7 38 b4 Qe5 39 b5?
39 Ne2 resisted just a bit longer.
39 ... Qg3+ 0-1

Game 42
G.Tallaksen-M.Carlsen
Gausdal 2004
1 d4 Nf6 2 c4 e6 3 Nf3 b6 4 a3 Ba6 5 Qc2 Bb7 6 Nc3 c5 7 e4 cxd4 8 Nxd4 Bc5 9 Nb3 Nc6

10 Bf4
This move makes a lot of sense. White delays the capture on c5 and homes in on the
d6-square.
We saw 10 Nxc5 in the previous game, while 10 Bg5 is covered in the next game.
10 Bd3 is a sensible developing move, getting ready to castle and worry about the
e5-square at a later date. 10 ... 0-0 11 0-0 Be7 12 Bf4 d6 13 Be2 Qc8 14 Rac1 Ne5 15
Nd2 Rd8 16 Rfd1 Qc7 with our by now familiar Hedgehog setup, was played in
O.Cvitan-K.Georgiev, Sibenik 2008.
10 ... e5
10 ... 0-0 is actually playable, despite not appearing to deal with White’s threat
behind 10 Bf4. White can then play:
a) 11 Nxc5 bxc5 when fast development is the key idea behind Black’s sacrifice of
the c5-pawn, for example 12 Bd6 Nd4 13 Qd3 Re8 14 b4 e5 was played in L.Van
Wely-J.Granda Zuniga, Wijk aan Zee 1997, and if now 15 Bxc5 then 15 ... a5! with all
sorts of threats beginning with ... d6, ... Nb3 and ... axb4.
b) 11 Rd1 e5 12 Bg5 Nd4 13 Nxd4 Bxd4 14 Bd3 h6 was also good for Black in
M.Lacrosse-J.Hammer, Cappelle la Grande 2008.
11 Bg5 h6

12 Bh4

Question: Wouldn’t it make more sense for White to
exchange on f6 and gain the wonderful outpost on d5?

Answer: This is certainly an option, but the knight on d5 can eventually be exchanged off. The d4-square is also
an outpost for Black, so White isn’t having things all his own way.
12 Bxf6 Qxf6 13 Nd5 Qd8 14 Be2 0-0 15 0-0 a5 with a level game was seen in
I.Solomunovic-I.Farago, Boeblingen 1999.
12 ... 0-0 13 f3
The idea is to defend the e4-pawn, as Black has ... Nd4 on the cards, unleashing the
b7-bishop. Alternatively, White has played:
a) 13 0-0-0 Be7 14 f3 Ne8 15 Bf2 Nc7 16 Kb1 Ne6 17 Qd2 Qb8 18 Nd5 Bc5 with a
balanced game, V.Zvjaginsev-K.Lerner, Rostov on Don 1993.
b) 13 Rd1 is natural, so the following game is worth examining. 13 ... Nd4 14 Nxd4
exd4!? (14 ... Bxd4 as usual is absolutely fine) 15 Bxf6 Qxf6 16 Nd5 Bxd5 17 cxd5
Rac8 18 Qd3 Rfe8 19 Be2 Qe5 20 Rd2 f5! 21 exf5 Bxa3! with advantage Black, was
I.Sokolov-Z.Efimenko, Serbian League 2010. It says it all when a world class exponent
of 1 d4, GM Ivan Sokolov, lands himself in trouble from this variation.
13 ... Rc8 14 0-0-0 a6

15 Qd2
Other White tries here:
a) 15 Bxf6 Qxf6 16 Rxd7 Ba8 gives Black full compensation, as White cannot do
much with the extra pawn and the kingside pieces on f1 and h1 cannot participate in the
game effectively.
b) 15 Nd5 looks good, but Black can defend with 15 ... Be7 16 Bxf6 Bxf6 17 Be2
Bg5+ 18 Kb1 Ne7.
c) 15 Kb1 Nd4 16 Nxd4 Bxd4 17 Bf2 Bxf2 18 Qxf2 was roughly equal, and now the
typical sacrifice 18 ... b5 spiced things up. After 19 cxb5 axb5 20 Nxb5 d5 21 exd5
Nxd5 22 Bd3 Qe7 23 Rhe1 Rfd8 24 g3 there were chances for both sides in G.Kamsky-
J.Friedel, Philadelphia 2009.

Exercise: White’s last move 15 Qd2 was understandable, given the
apparent weakness of the d7-pawn. How should Black deal with this?

Answer: Actually there are two ways which are efficient here.
15 ... Nxe4
This doesn’t win a pawn, but relieves the pressure with a neat trick.
15 ... Nd4 16 Nxd4 Bxd4 sealing the d-file gives Black a nice position, for example
17 Bf2 Bxf2 18 Qxf2 b5! and the counterattack is on! 19 cxb5 axb5 20 Bxb5 d5 21 exd5
Nxd5 and suddenly the black pieces spring to life, and the open c-file is a huge cause of
concern for White.
16 Nxe4
16 Bxd8 Nxd2 17 Rxd2 Rfxd8 nets a pawn for Black.
16 ... Qxh4 17 Nexc5 bxc5 18 Qxd7 Ba8
The d7-pawn has swapped itself for White’s e-pawn, but the features of the position
remain. The d4-square is in Black’s control, and he is ahead in development. It is not
surprising that Carlsen is able to turn the screw here.
19 Qf5 Rfd8 20 g3 Rxd1+ 21 Kxd1 Qd8+ 22 Kc1 Rb8 23 Nd2 Nd4 24 Qd3
24 Qxe5 Nxf3 25 Nxf3 Bxf3 threatens both ... Bf3xh1 and ... Qd1 mate.
24 ... Nxf3 25 Nxf3

25 ... Qf6
This is even stronger than the obvious queen exchange 25 ... Qxd3 26 Bxd3 Bxf3 and
the pawn-up endgame should eventually be winning.
26 Qc3
26 Bg2 e4 is crushing.
26 ... Bxf3 27 Rg1 Be4 28 Bd3 Bxd3 29 Qxd3 Qf2 0-1
White cannot defend both the b2-pawn and g1-rook. A great example of the potential
of Black’s position if White tries to open up the game.

Game 43
I.Khenkin-M.Adams
German League 2002

1 d4 Nf6 2 c4 e6 3 Nf3 b6 4 a3 Ba6 5 Qc2 Bb7 6 Nc3 c5 7 e4 cxd4 8 Nxd4 Bc5 9


Nb3
9 Nf3 is an interesting alternative to the main line 9 Nb3.


After 9 Nf3 in recent times, Black has played:
a) 9 ... Nc6 10 b4 Nd4 with a further split:
a1) 11 Qd3 Nxf3+ 12 gxf3 Be7 13 Rg1 0-0 14 Bb2 Qc7 15 Qe3 Ne8 16 Qh6 f6 17
Nb5 Qc8 18 Rd1 Rf7 when Black was holding firm in I.Rajlich-N.Dzagnidze, Turin
Women’s Olympiad 2006.
a2) 11 Qd2 Nxf3+ 12 gxf3 Be7 13 Bb2 d6 14 Rg1 S.Sethuraman-B.Adhiban, Indian
Championship 2010 and now 14 ... 0-0 15 Qh6 g6 16 0-0-0 e5 17 f4 Qc8 is very
unclear, since both kings are under fire.
b) 9 ... Ng4 is playable, and can be dangerous if White does not know how to react:
b1) 10 Nd1 Nc6 and already I much prefer Black. Now the key d4- and e5-squares
are both in Black’s possession. Black won the following two games in less than 20
moves!
b11) 11 h3 Nge5 12 Nxe5 Nxe5 13 Bf4 Ng6 14 Bg3 0-0 15 Bd3 f5 16 f3 Qg5 17
Bh2 fxe4 18 Bxe4 Bxe4 19 fxe4 Nh4 0-1 P.Lasinskas-Y.Gonzalez Vidal, Sabadell 2011.
b12) 11 Be2 Qf6 12 h3 Nge5 13 Nxe5 Qxe5 14 Nc3 Nd4 15 Qd3 f5 16 exf5 Bxg2
17 f4 Nf3+ 18 Kd1 Qxf5 0-1, S.Korotkjevich-J.Markos, Pardubice 2002.
b2) 10 Bg5 which I think is the only good reply. Now 10 ... f6 (if 10 ... Bxf2+ 11
Qxf2 Nxf2 12 Bxd8 Nxh1 13 Bg5 is unclear) 11 Bh4 Nc6 12 b4 Be7 13 Rd1 0-0 14
Be2 Nge5 15 0-0 Nxf3+ 16 Bxf3 a5! with queenside pressure was the continuation of
M.Krasenkow-V.Baklan, French League 2014.
Returning to the main game after 9 Nb3:

9 ... Nc6 10 Bg5


This is the main line. Instead of going after the d6-square with 10 Bf4 (see the
previous game), the bishop immediately heads to its most active square, and pins the
f6-knight.

Question: I don’t like pins very much. Can I not
just play 10 ... Be7 here to defend against this?

Answer: Yes you can, and this is solid. It is played less often than the game continuation, as there is more
dynamism with the bishop on c5 trying to come to the nice d4-square. After 10 ... Be7 11 Be2 d6 12 0-0 0-0 13 Rfd1
Qb8 14 Bf4 Ne5 15 Bg3 Rc8 16 Nd2 a6 17 Rac1 the players agreed to a draw in M.Devereaux-S.Gordon, British
League 2006.
In our main game Adams continued with:
10 ... h6
Instead, after 10 ... Nd4 11 Nxd4 Bxd4 White has:
a) 12 Bd3 Qb8 13 Ne2 h6 14 Bd2 Bc5 15 b4 Be7 16 0-0 0-0 17 f4 a5 with a typical
middlegame resulting from this line, which was played in I.Sokolov-N.Short, Malmo
2009.
b) 12 Nb5 Be5 13 f4 (as we will see, the point behind inserting 10 ... h6 is that after
the g5-bishop drops back to h4, this f2-f4 advance is ruled out) 13 ... Bb8 14 e5 h6 15
Bxf6 gxf6 16 Nd6+ Bxd6 17 exd6 Rc8 18 Qc3 Rg8 19 Rg1 f5 20 g3 Be4 21 Be2 Kf8
22 Kf2 Rg7 23 Rgd1 Kg8 24 Rac1 White’s chances were to be preferred in V.Ivanchuk-
D.Bocharov, Russian League 2011.
11 Bh4 Nd4 12 Nxd4 Bxd4
Now we see the point of including ... h6 and Bg5-h4.
13 Bd3
After 13 Nb5 Be5 White is unable to follow the Ivanchuk-Bocharov game above, as
he cannot play f2-f4 now.
13 ... Qb8

Question: This looks like an odd square for Black’s queen!

Answer: However it is well founded. The queen escapes the pin on the h4-d8 diagonal, and exerts great influence
on the important squares d6, e5 and f4 in the centre.
14 Bg3 Be5
Also Black has this key move, maintaining control of those key squares.
15 0-0-0
15 0-0 is less common, although there are f2-f4 ideas in the air. The following line
might be one reason it is not so popular: 15 ... Bxg3 16 hxg3 h5 with attacking ideas
down the h-file. 17 Rad1 h4 18 gxh4 Rxh4 19 g3 Rg4 20 Kg2 Ke7 21 Rfe1 Qc7 22 Re3
Rh8 23 Be2 Rg6 24 Bf3 Rgh6 25 Qd3 g5 was by no means unacceptable for Black in
H.Olafsson-Z.Hracek, Selfoss 2002.
15 ... 0-0 16 Kb1
After 16 Rhe1 a6 17 Kb1 b5! White can choose:
a) 18 cxb5 axb5 19 Nxb5 Bc6 20 Bxe5 Qxe5 21 Nc3 Rfb8 with plenty of attacking
compensation, Benko Gambit style.
b) 18 c5 Rc8 and already this is a bit tricky for White, and here he ‘went for it’ with
19 f4 Bxf4 20 e5 Bxg3 21 exf6 in I.Karim-J.Hammer, Internet (blitz) 2010 but now 21
... Bxe1 is winning for Black.
16 ... d6 17 Bxe5 dxe5

Question: You want me to accept doubled pawns?

Answer: Doubled pawns are always a dubious point with many players, but they are beneficial in positions such as
this. Firstly, they control the key d4- and d5-squares, and secondly they prevent a kingside pawn storm beginning with
f2-f4 etc.
18 Qe2 Rd8 19 Qe3 Rd4 20 f3
20 Ne2 Rxd3! 21 Rxd3 Bxe4 wins a pawn with a neat tactic.
20 ... Ba6 21 b3 Qc7 22 Kb2 Rad8 23 Ne2 R4d7
Black certainly has chances here. Those doubled e-pawns have given Black the open
d-file and he is already in control of it.
24 Bc2 b5 25 c5
If instead 25 cxb5 Bxb5 26 Nc3 Ba6 when ... Rd4 is coming and Black still has
pressure.

After the game move, if White could now get in b3-b4 he would have a protected
passed c-pawn ...
25 ... b4! 26 axb4 Rb8 27 Bd3 Bxd3 28 Rxd3 Rxb4 29 Rc1 Qc6
King safety in general is very important. Black’s king is tucked up safely on g8,
which cannot be said about White’s king on b2, as there are threats down the b-file and
... a5-a4 to worry about.
30 Rd6 Rxd6 31 cxd6 Qxd6 32 Qxa7 Nd7
32 ... Qd2+ is just a check and White can defend with 33 Rc2. Far better to improve
the least effective piece, in this case the f6-knight.
33 Qc7? Qd3 34 Qc2

Question: Can you spot the move White missed here?

Answer:
34 ... Nc5!
Now the b3-pawn really feels the heat. There is no defence.
35 Qxd3 Nxd3+ 36 Kc3 Nxc1 37 Nxc1
37 Kxb4 Nxe2 is just lost. Now it is simply a matter of technique, for which Adams
is up to the task. The game concluded:
37 ... Rb8 38 Nd3 Rc8+ 39 Kd2 f6 40 g4 g5 41 b4 Kf7 42 Nc5 Ke7 43 Kc3 h5 44 gxh5 Rh8 45 b5 Kd6
46 Nb7+ Kc7 47 Nc5 Rxh5 48 Nxe6+ Kb6 49 Kc4 Rh3 50 Nc5 Rxf3 51 Na4+ Kb7 52 Kd5 Rf2 53 h3 Rf3
54 Ke6 Rxh3 55 Nc5+ Kc8 56 Kxf6 g4 57 Ne6 g3 58 Ng5 Rh1 59 Kxe5 g2 60 Nf3 Rf1 0-1

Game 44
S.Lputian-S.Tiviakov
Kropotkin 1995

1 d4 Nf6 2 c4 e6 3 Nf3 b6 4 a3 Ba6 5 Qc2 Bb7 6 Nc3 c5 7 e4 cxd4 8 Nxd4 Nc6


Black deviates from 8 ... Bc5 which we examined in the last few games. This is the
other way to play against the Petrosian Variation, to add to your repertoire. The key
squares are the same: d4, e5 and f4.
9 Nxc6
9 Be3 Ng4! 10 Be2 Nxe3 11 fxe3 Qg5 was already excellent for Black in M.Lupik-
M.Purtseladze, Herceg Novi 2005.
9 ... Bxc6 10 Bf4

10 ... Bc5
Instead 10 ... Nh5 11 Be3 and now:
a) 11 ... Bd6?? 12 Qd1! 1-0 was L.Christiansen-A.Karpov, Wijk aan Zee 1993, the
famous Karpov game which highlighted that tactics can occur when you least expect
them!
b) 11 ... Qb8 is the correct way to control the f4-square, as we saw in the previous
game Khenkin-Adams.
11 Be2 0-0 12 0-0 a5


Question: Why is Black pushing the a-pawn like this?

Answer: As it prevents any b2-b4-b5 ideas harassing the black bishops. Whilst that doesn’t win any material in
this position, it could be a possibility within the next few moves. So Tiviakov takes time out to defend against it
immediately.
13 Rad1 Qe7 14 e5
White’s other options at this point are:
a) 14 Bf3 e5 15 Bc1 a4 as played in S.Marjanovic-P.Popovic, Clichy 1986.
b) 14 Bg5 e5 15 Bf3 Qe6 16 Nd5 Bxd5 17 cxd5 Qd6 18 Rd3 Rfb8 which was seen
in S.Yudin-S.Tiviakov, Moscow 2009.
In both cases White cannot play b2-b4 and Black stands well.
14 ... Nh5

Exercise: Did Grandmaster Tiviakov just blunder a piece?

Answer: No. In fact Black has a tactic in mind.
After 15 Bxh5 Qh4 Black’s double attack on the two white bishops is not easy to
deal with, and in any case Black recovers the sacrificed piece:
a) 16 g4? Qh3 17 Nd5 (or 17 Ne4 Qf3) 17 ... exd5 18 cxd5 Bb5 19 Rfe1 g6 even
wins a piece for Black.
b) 16 Bg3 Qxh5 17 Na4 f5 18 exf6 Rxf6 19 Nxc5 bxc5 is probably equal.
c) 16 Qe2 Qxf4 17 g3 Qg5 18 Bf3 Bxf3 19 Qxf3 Qxe5 20 Rxd7 Rad8 and again
White ‘managed’ equality in I.Khenkin-M.Adams, Elenite 1993.
In the main game, White instead continued with:
15 Bc1?! Qh4 16 Rd3
With the threat of Rd3-h3, which is easily parried.
16 ... Nf4

17 Rg3
Instead:
a) 17 Bxf4? Qxf4 wins the e5-pawn.
b) 17 g3 Nh3# would be a nice way to end the game!
17 ... f5!
An excellent move, the strength of which can be underestimated. Alternatives are no
good:
a) 17 ... f6?? 18 Rg4 Nxe2+ 19 Qxe2 Qh5 20 Rxg7+ and White wins Black’s queen.
b) 17 ... Nxe2+? 18 Qxe2 with the annoying threat of 19 Bg5 trapping the queen.
18 exf6?!
Other White tries here are:
a) 18 Bf3 trying to exchange the c6-bishop is better, although Black is still in the
driving seat. After 18 ... Bxf3 19 Rxf3 Ng6 (hitting the e5-pawn) 20 Qe2 Bd4 21 Rh3
Qg4, White’s weak e5-pawn is again feeling the heat.
b) 18 Rd1 Nxe2+! 19 Qxe2 f4 20 Rh3 Qg5 also looks good for Black, as 21 Ne4
drops the important e5-pawn.
18 ... Rxf6


Exercise: Can you see what to do as Black here if White plays 19 Rg4?

Answer: Black has a neat tactic 19 ... Bxf2+! with two lines:
a) 20 Kh1 there are two attractive mates, namely 20 ... Qxh2+ 21 Kxh2 Rh6+ 22 Rh4
Rxh4# and 20 ... Qxg4 21 Bxg4 Bxg2#.
b) 20 Rxf2 Nh3+ 21 gxh3 Qxf2#.
The game continued:
19 Bxf4 Rxf4 20 Na4


Exercise: What move would you play here
as Black? a) 20 ... Raf8, b) 20 ... Bd4 or c) 20 ... Bxa4?

Answer:
20 ... Bd4!
Every black piece is stronger placed than its white counterpart. The d4-bishop
dominates the errant a4-knight. Both other moves 20 ... Bxa4 21 Qxa4, and 20 ... Raf8
21 Nxc5 bxc5 are still playable, but I don’t want to exchange off one of the powerful
bishops which are raking across the board towards White’s king.
21 Bf3
A sad necessity, weakening the king, but the threat of 21 ... Raf8 adding to the attack
on the f2-pawn was too much.
21 ... Bxf3 22 gxf3 Raf8 23 Kg2 Qf6 24 Qd3 Rh4 25 Nc3 Be5 26 Ne4
26 Qxd7 Bxg3 27 fxg3 Rxc4 should be winning for Black.
26 ... Qf5 27 Kg1

27 ... Kh8
27 ... Bxg3? 28 fxg3 Rh6 29 Qxd7 loses the attacking initiative.
28 Rd1
Or 28 Qxd7 Bxg3 29 fxg3 Rxe4! and Black wins a rook.
28 ... Bxg3 29 fxg3 Rh6 30 f4
30 Qxd7 Qxf3 is different to the note above, as the attack is crashing through.
30 ... e5 31 Rf1 Qh3 32 Qe2 exf4 33 Rf2
After 33 gxf4 Re8 Black is an exchange up and with all the pressure, White is
doomed.
33 ... Re6 34 Qf3 Qf5 35 Nc3 fxg3! 0-1
Game 45
A.Goldin-V.Chuchelov
Cappelle la Grande 2001

1 d4 Nf6 2 c4 e6 3 Nf3 b6 4 a3 Ba6 5 Qc2 Bb7 6 Nc3 c5 7 e4 cxd4 8 Nxd4 Nc6 9 Nxc6 Bxc6

10 Be2
We saw 10 Bf4 in the previous game. Other White alternatives on the 10th move are:
a) 10 Qe2 is a strange move championed by French Grandmaster Vladislav
Tkachiev. It prepares e4-e5 and also g2-g3 to fianchetto the king’s bishop. Now Black
has tried:
a1) 10 ... Qc7 11 e5 Ng8 12 Bf4 Ne7 13 0-0-0 a6 14 h4 b5 getting on with queenside
play before deciding where to put the king, S.Atalik-T.Gelashvili, Greek League 2003.
a2) 10 ... d6 11 g3 Be7 12 Bg2 0-0 13 0-0 Nd7 14 Bf4 Ne5 15 Rfd1 Qb8 16 Rac1
a6 17 b4 Bf6 with our standard Hedgehog structure against the Maroczy bind was
V.Tkachiev-M.Adams, French League 1999.
b) 10 e5 is a rare attempt but can be quite dangerous.

Question: Where to move the f6-knight, 10 ... Ng8 or 10 ... Ng4?

Answer: Probably 10 ... Ng8 is safest. Although it ‘undevelops’, the idea is to bring the knight back to life with ...
Ne7-g6 and go after the advanced e5-pawn.
b1) 10 ... Ng4 11 Qe2 and now the g4-knight can be stranded in some lines, for
example:
b11) 11 ... f5 12 h3 Qh4 13 g3 Qh5 14 Rg1! Bc5 15 Be3 and White won a piece in
N.Pert-M.Chandler, 5th match game, London 2000.
b12) 11 ... Qh4 12 h3 Nh6 13 g3 Qd8 14 Ne4 Nf5 15 Bg2 Be7 16 0-0 0-0 17 Bf4 a5
18 Rad1 when Black was solid but White had some annoying pressure down the d-file
in J.Piket-P.Wells, German League 1999.
b2) 10 ... Ng8 11 Bf4 Ne7 12 Rd1 Qb8 13 Qd2 Ng6 14 Bg3 Nxe5 15 Be2 Be7 16 0-
0 0-0 17 b4 d6 18 f4 Ng6 and in J.Piket-V.Chuchelov, Mondariz Zonal 2000, White had
compensation for the pawn but probably not more.
Returning to the main game after 10 Be2:
10 ... Qc7
Steps must be taken to prevent e4-e5. Instead of this move, 10 ... Qb8 is the main
line but the following variations have shown that there are some very dangerous lines to
navigate. After 11 f4 Black has tried:
11 ... Bc5 which is designed to prevent the e4-e5 push, but it turns out White can
play this anyway! 12 e5! Bxg2 (12 ... Ng8 13 Ne4 was not Black’s plan) 13 exf6 Bxh1
14 fxg7 Rg8 15 Qxh7 Ke7 16 Bd2 and this position is fraught with danger for Black, for
example 16 ... Qb7 17 0-0-0 Rac8 18 f5! f6 19 Qg6 Qg2 20 Bg4 Bd4 21 fxe6 dxe6 22
Re1 e5 23 Qf5 Qc6 24 Nb5 Qxc4+ 25 Kb1 Rcd8 26 Bb4+ 1-0 S.Lputian-Z.Hracek,
Yerevan 1996.
11 ... e5 is an attempt to prevent the e4-e5 push, but here White still has good play,
as in E.Najer-R.Pogorelov, Moscow 2002, which continued 12 Rf1 Bd6 13 f5 Qb7 14
Bf3 a5 15 g4 h6 16 Be3 Bc5 17 Bxc5 bxc5 18 0-0-0 Ke7 19 h4 d6 20 g5 with a
dangerous attack.
11 f4
11 ... Bc5
11 ... e5 12 Rf1 Bd6 13 g3 0-0 14 Be3 Qb7 15 0-0-0 Nxe4 16 Bd3 Nf6 17 g4 was
extremely unclear in E.Najer-R.Ovetchkin, Russian Championship, Krasnoyarsk 2003.
12 b4
Other White options here:
a) 12 Bd2 e5 13 Rf1 Bd4 14 0-0-0 0-0 15 g4 exf4 16 g5 Bxc3 17 Bxf4 was
S.Lputian-J.Timman, European Club Cup, Belgrade 1999, and now after 17 ... Qb7 18
Qxc3 Nxe4 19 Qd4 Rae8 Black has a pawn extra for some play.
b) 12 e5 doesn’t work as well when Black’s queen is on c7, because Black can
castle queenside in some lines, for example 12 ... Bxg2 13 exf6 Bxh1 14 fxg7 Rg8 15
Qxh7 0-0-0! with an extra exchange.
12 ... Bd4 13 e5

Question: This isn’t a very ‘Queen’s Indian’ like position! What is going on here?

Answer: This is one of the most tactical lines in the book. Black’s next will come as a surprise ...
13 ... Nd5
Other moves:
a) 13 ... Bxc3+ 14 Qxc3 Ne4 15 Qe3 0-0 16 0-0 f6 17 exf6 Nxf6 18 Bb2 Qb7 19
Qg3 Be4 20 Rad1 Rac8 21 Rd6 Rf7 22 f5! was clearly better for White in N.Pert-
M.Chandler, 3rd matchgame, London 2000. When those two bishops work, they are
potent.
b) 13 ... Ne4 14 Nxe4 Bxa1 15 Nd6+ Ke7 16 Be3 Bxg2 17 Rg1 and there are too
many black pieces hanging here.
14 Ne4
14 cxd5 Ba4! is the trick. If 15 Qxa4 then 15 ... Qxc3+ followed by 16 ... Qxa1.
14 ... Nxf4
14 ... Bxa1 15 cxd5 exd5 16 Nd6+ Ke7 17 b5 will give White two pieces for the
rook, and the powerful d6-knight will make the difference.
15 Nd6+
This looks obvious, but is met with a strong reply. Instead the critical line is 15 Bxf4
and now:
a) 15 ... Bxa1 16 Nd6+ Kf8 17 0-0 Bd4+ 18 Kh1 gives White huge compensation for
the exchange. The f7-pawn is a prime target and he is fully developed.
b) 15 ... Bxe5 16 Bxe5 Qxe5 17 Nc3 Bxg2 18 Rg1 Bc6 19 0-0-0 0-0 could be the
way forward in this line. Black has three pawns for the piece, but whether that is good
or not time will tell.
15 ... Qxd6!
Looks brilliant, but it was almost forced. Instead 15 ... Kf8 16 Bxf4 Bxa1 17 0-0
transposes to 15 ... Bxa1 in the previous note which is just good for White.
16 exd6 Nxg2+ 17 Kd2 Bxa1
The position is roughly equal. For the queen Black has rook, knight and two pawns,
but anything can happen!
18 Bb2 Bxb2 19 Qxb2 0-0 20 Rg1 f6 21 Qc3 Nh4 22 b5 Bb7 23 c5 bxc5 24 Qxc5
Ng6 25 Qc7 Bd5 26 Rc1
26 Qxd7 Rf7! traps the queen.
26 ... Ne5 27 Qa5 f5 28 Rc7 f4 29 Qc3 Rf5

30 a4?! f3 31 Bf1 f2
The f-pawn has nearly made it to touchdown.
32 Kc1 Rg5 33 Rc8+ Kf7 34 Rxa8 Rg1?
34 ... Bxa8 is still good for Black.
35 Qh3?
Easy to criticise, but the computer calmly points out a draw with 35 Qxe5 Rxf1+ 36
Kd2 Rd1+ 37 Kc3 Rc1+, since if 37 ... f1Q 38 Qh5+ even wins for White! Very
difficult to spot, of course.
35 ... Bxa8 36 Kd2 Be4 37 Ke3 Bd3 38 Qh5+ Kf6 39 Bh3 f1Q 40 Bxf1 Rxf1 41
Kd2 Rf4 42 Qd1 Bxb5 0-1
As 43 axb5 Rd4+ wins the queen. Certainly an interesting game!
Chapter Seven
Petrosian Variation: 4 Nc3 Bb7 5 a3
1 d4 Nf6 2 c4 e6 3 Nf3 b6 4 Nc3
With this move order White more or less obliges Black to develop his light-squared
bishop on b7.
4 ... Bb7
4 ... Ba6 makes no sense here, as White could simply occupy the centre with 5 e4.
Meanwhile 4 ... Bb4 transposes to the Nimzo-Indian, so 4 ... Bb7 is Black’s main move
if he wants to stay within Queen’s Indian territory.
5 a3
Chapter Eight will cover the independent lines starting here with 5 Bg5 and 5 Qc2.

With 5 a3, White enters the Petrosian Variation, having prompted Black to play 4 ...
Bb7. Of course this position could also be reached after 4 a3 Bb7. In the previous
chapter we examined 4 a3 Ba6, and I believe that system is completely viable and
solid. However, to broaden your horizons on the Queen’s Indian and the variety of
pawn structures on offer, it is a good idea to be familiar with more than one main line
chosen by Grandmasters at the highest level. This gives you a choice but more
importantly, these games highlight the instructive Queen’s Indian middlegames that arise
and so they are certainly worth taking a look at. Black must understand what he is doing
here, as he can be steamrollered very fast if he doesn’t play carefully.
5 ... d5
This is the key move to play here for Black. Black argues that 5 a3 was not the most
useful move and thus a transposition to a Queen’s Gambit Declined (QGD) structure is
the way to go. Note that the other typical move ... c5 transposes to a Benoni structure
after d4-d5. This is not a pawn sacrifice (as in similar situations where the White queen
is on the c2-square) because the queen is still on d1 and defends the d-pawn. If you are
willing to play a QGD type of setup, and wish for a line a little more solid, then 5 ... d5
should probably be your weapon of choice.
In Game 46, we look at what happens if Black doesn’t occupy the centre but
continues development with 5 ... Be7?! which is answered by 6 d5!. The remaining
games in the chapter examine 5 ... d5. Before getting into the main line with 6 cxd5, we
look at Carlsen’s choice of 6 Bg5 in Game 47. My recommendation after 6 cxd5 is 6 ...
Nxd5 and here White’s key 7th move alternatives are 7 Qc2 (Game 48), 7 e3 (Game
49) and finally 7 Bd2 (Games 50-51).

Game 46
A.Gupta-Dao Thien Hai
Asian Team Championship, Vishakapatnam 2008

1 d4 Nf6 2 c4 e6 3 Nf3 b6 4 Nc3 Bb7 5 a3 Be7?!

Such a natural move, as the f8-bishop cannot go to b4 or any other decent square on
this turn. However, it fails to meet the demands of the position – it is crucial that Black
occupies the centre with a pawn on the 5th move, for reasons this game will
demonstrate.
6 d5!

Question: What is the reasoning behind this?

Answer: Just watch the game and you will see! We know the e4- and d5-squares are critical. How can Black
now prevent e2-e4 on the next move, or stop White building up a huge centre? This game is a lesson on how to use
that central space advantage to great effect.
6 ... 0-0 7 e4 d6
7 ... c6 8 d6 would be a disaster!
8 Bd3 Nbd7

9 Nd4
9 0-0 was a good alternative, but Gupta realises that with such a big space
advantage in the centre he can delay castling to build up an attack.
9 ... e5
9 ... Nc5 10 Bc2 with a space advantage, the general rule is: do not exchange pieces,
as this allows the side with the more cramped position to reduce the pressure.
10 Nf5 Re8 11 Be3 Nf8 12 Qf3 Bc8


Exercise: Think of what you would do here as White.

Answer: Almost anything sensible is good, but Gupta decides that with the centre closed he can go all aggressive
with:
13 g4! N6d7 14 h4 Nc5 15 Bc2 a5
15 ... Ba6 16 b4 Ncd7 17 Bd3 Black can hardly move, and must await his fate on the
kingside.
16 0-0-0 Ng6 17 g5 Bf8 18 Rdg1
The rook is not needed in the centre.
18 ... Nf4 19 h5
19 Bxf4 exf4 20 Qxf4 is just a safe pawn advantage.
19 ... g6 20 hxg6 fxg6

Exercise: Calculate what you would do here (tip: very
difficult to calculate the whole variation to the end).

Answer: White crashes through with:
21 Bxf4 exf4 22 Rxh7!! Bxf5
Or 22 ... Kxh7 23 Rh1+ Kg8 24 Qh3 and the attack down the h-file is decisive, for
example 24 ... Re7 25 Qh8+ Kf7 26 Rh7+ Ke8 27 Rxe7+ Qxe7 28 Nxe7 Kxe7 29 Qf6+
Kd7 30 e5 with an overwhelming position.
23 exf5
23 Rgh1 Bd7 24 Rh8+ Kf7 25 Qxf4+ was also crushing.
23 ... Kxh7 24 f6
So calm, despite being a rook down! The black king will not escape.
24 ... Kg8 25 Bxg6 Re7 26 Qh5 Bg7 27 Rh1
The f6-pawn is worth more than either of the pieces it can capture.
27 ... Kf8

Exercise: Calculate the mate from here.

Answer: With the queen sacrifice:
28 Qh8+! 1-0
A crushing attack. We saw how a lack of central control, and allowing the white
pawns to occupy the d5- and e4-squares with a big space advantage, can almost end
Black’s game before it has started. Do not let this happen!

Game 47
M.Carlsen-S.Karjakin
Wijk aan Zee 2012

1 d4 Nf6 2 c4 e6 3 Nf3 b6 4 Nc3 Bb7 5 a3 d5



Question: Why play ... Bb7 then ... d5? Are we
playing some kind of Queen’s Gambit Declined?

Answer: Black plays more classically in this line, with a traditional pawn centre. However, as we will see, Black
often meets cxd5 with ... Nxd5 (rather than ... exd5) to play more dynamically.
6 Bg5
The main line is 6 cxd5, which we will review in subsequent games.
6 ... Be7 7 e3 0-0 8 Rc1 h6
8 ... Nbd7 is commonly seen as well. The structure after 9 cxd5 exd5 10 Bd3 c5 11
0-0 Ne4 12 Bxe7 Qxe7 13 dxc5 bxc5 (V.Georgiev-Y.Kryvoruchko, Macedonian League
2012), is again a Queen’s Gambit Declined with the early a2-a3 move.
9 Bxf6
9 Bh4 was also possible, but the game move is designed to try and take Black’s e7-
bishop away from the defence of the c5-square. E.Relange-A.Onischuk, European Club
Cup, Kallithea 2002 continued 9 ... Nbd7 10 cxd5 exd5 11 Bd3 c5 12 0-0 Ne4 13 Bxe7
Qxe7 14 dxc5 bxc5 15 Bb1 Ndf6 16 Qc2 Rac8 17 Rfd1 Rfd8 and Black was fine here.
Notice Black even has the extra ... h6 tempo in, which is why Karjakin has played it so
early in the main game.
9 ... Bxf6 10 cxd5 exd5 11 Bd3 c5

Question: You mentioned the c5-square. Can White
not try and go after this pawn immediately?

Answer: White can, but without having castled, this might be a bit risky. After 12 dxc5 bxc5 13 Na4 c4 14 Bb1
Qa5+ 15 Nc3 Nc6 16 0-0 Rfd8 17 Qc2 g6 it is not clear what has been gained by this attack on the c-pawn. If
anything, it only helped Black.
12 0-0 Na6


Question: What is the point behind Black’s
last move 12 ... Na6? Why not just go 12 ... Nd7?

Answer: Black’s move defends the c5-pawn but also enables the knight manoeuvre ... Na6-c7-e6, putting it on the
excellent e6-square where it puts pressure on d4. We saw this plan in Oliveira-Ibragimov (Game 16) from Chapter
Three.
13 Ne5
After 13 Bb1 Re8 14 Qd3 g6 15 Rfd1 c4 16 Qe2 Nc7 Black has ideas of ... b5, ... a5
and ... b4 to attack on the queenside.
13 ... cxd4 14 exd4 Bxe5 15 dxe5 Nc5


Question: You mentioned pawn structures in the introduction, and I
see Black has an Isolated Queen’s Pawn (IQP). Is it good or bad here?

Answer: Normally such a pawn can become weak if all the minor pieces are exchanged off, but here White does
not have a grip on the d4-square in front of the pawn. The d-pawn can be quite strong if it is mobile and able to
advance quickly. How effective it is here is the main battleground for this game.
16 Re1!
‘The only try to fight for an advantage’ (GM Karjakin). Other tries were:
a) 16 Nb5 Nxd3 17 Qxd3 Ba6 which should be equal.
b) 16 Bb1 d4 17 Ne2 Qd5 and now we see the d-pawn being very strong and gaining
space. White must be wary of this.
16 ... Re8
The most logical move 16 ... d4 does not work since 17 Ne4 was the point of
White’s 16th move. After 17 ... Bxe4 (17 ... Qd5? 18 Bc4! Qxe5 19 Nxc5 Qxc5 20
Bxf7+ wins the queen) 18 Bxe4 d3 19 Rxc5 bxc5 20 Bxa8 Qxa8 21 Qxd3 White has an
extra pawn.
17 f4
Defending the e5-pawn and playing aggressively. Carlsen never plays for a draw
(that is why he is World Champion), however here he should be trying to equalise the
position. 17 Bf1 a6 18 Rc2! (Karjakin) with ideas of Rc2-d2 to keep an eye on the d-
pawn was best.
17 ... d4

18 Ne4?
18 Nb5 Qd5 19 Rc2 Nxd3 20 Qxd3 Ba6 21 a4 Bxb5 22 axb5 Rac8 was roughly
equal.
18 ... Bxe4 19 Bxe4 d3! 20 Rc4
Other moves were no better:
a) 20 Bxa8 d2 this is the trick which recovers the rook. After 21 Bc6 Qd4+ 22 Kh1
dxe1Q+ 23 Qxe1 Nd3 24 Qf1 Re6 25 Rc2 Nxf4 Black has a dominating position.
b) 20 Re3! Qd4 21 Qf3 Rac8 22 Rd1 d2 23 Kf1 (Karjakin) but this looks terrible for
White as he is only just managing to hold everything together.
c) 20 Rxc5 Qd4+! is also good for Black.
20 ... Rc8 21 Bf5 Qd5 22 Rc3 Rcd8
Is that d-pawn strong or weak now? It is definitely strong, as it is far down the board
and has forced White on the defensive.
23 Qd2 Qd4+ 24 Kh1 a5!
This is a very nice move. The c5-knight must remain stable to defend the d3-pawn,
and this move prevents b2-b4 kicking it away. Such typical positional moves are a
hallmark of a great Queen’s Indian player.
25 Rb1 a4 26 Rd1


Exercise: Can you see how Black can win material here?

In the game, Black played:
26 ... Rd5
Answer: With 26 ... g6!, for example 27 Bxd3 (or 27 Bg4 Ne4 28 Rxd3 Qxd3 29 Qxd3 Rxd3 30 Rxd3 Nf2+ 31
Kg1 Nxd3 winning a rook) 27 ... Ne4 28 Bxe4 Qxd2 29 Rxd2 Rxd2 should be winning as the b2-pawn will fall soon.
27 h4 g6
27 ... Red8 28 h5 f6 ‘and White is defenceless’ (Karjakin).
28 Bxd3 Red8 29 Qe1 Qxf4 30 e6!
Carlsen never gives in and looks for chances.
30 ... Nxe6 31 Bc2 b5
31 ... Nd4! 32 Bxa4 Nf5 33 Rxd5 Rxd5 34 Qe8+ Kg7 leaves White’s king
hopelessly exposed, for example 35 Bb3 Qxh4+ 36 Kg1 Nd6! remaining a clear pawn
to the good.
32 Rxd5 Rxd5 33 Re3 Nd4

34 Bd3
34 Bxg6 looks dangerous, but Black’s central control holds it all together. After 34
... fxg6 35 Re8+ Kg7 36 Re7+ Kf6 there are no more checks and the sacrifice has
failed.
34 ... Kg7 35 Kg1 Qf6 36 Kh2 Rh5 37 Rh3 Ne6 38 Rf3 Rxh4+ 39 Kg1 Qd4+ 40
Qf2 Qxf2+ 41 Kxf2 b4
Now it is just a matter of technique.
42 Re3 Rd4 43 Bb5 Kf6 44 Rf3+ Ke7 45 Rd3 bxa3 46 bxa3 Rf4+ 47 Ke3 f5 48 Rd7+ Kf6 49 Rd6 Re4+
50 Kf2 Kg5 51 Be8 Nf4 52 Bb5 Re5 53 Bc4 Nh5 54 Ra6 Nf6 55 Rxa4 Ng4+ 56 Kf1 Kh4! 57 Be2 Kg3 58
Bxg4 fxg4 59 Rb4 h5 60 a4 Kh2 0-1

Game 48
D.Sharavdorj-A.Shomoev
Ulaanbaatar 2011

1 d4 Nf6 2 c4 e6 3 Nf3 b6 4 a3 Bb7 5 Nc3 d5 6 cxd5



6 ... Nxd5

Question: What is wrong with the natural recapture 6 ... exd5?

Answer: Nothing is wrong with it, but having defined the central pawn structure, White can revert to a kingside
fianchetto which is a good setup against the hanging pawns on c5 and d5. After 6 ... exd5 White has played:
a) 7 Bg5 which occurred back in a classic game from 1930. 7 ... Be7 8 e3 0-0 9 Bd3
Ne4 10 Bf4 Nd7 11 Qc2 f5 12 Nb5 Bd6 13 Nxd6 cxd6 14 h4 Rc8 15 Qb3 was better
for White in Sultan Khan-J.Capablanca, Hastings 1930, with the Cuban great eventually
going down with Black in this game.
b) 7 g3 Be7 8 Bg2 0-0 9 0-0 c5 10 Bf4 Na6 was seen in V.Ivanchuk-E.Tomashevsky,
Greek League 2009, with a position similar to Carlsen-Karjakin except White’s light-
squared bishop is better placed on g2 to attack the d5-pawn.
7 Qc2
There are of course many possible moves here, which will be examined in the next
few games.
7 e4 Nxc3 8 bxc3 Bxe4 loses the pawn, so 7 Qc2 helps prepare this e2-e4 push.

7 ... Nxc3
7 ... Be7 8 e4 Nxc3 9 bxc3 c5 is another playable move-order. Note that in the
current game, Black will opt for a more aggressive and riskier ... Bd6 idea.
8 bxc3
8 Qxc3 looks odd given control over the e4-square is the name of the game, but the
idea is to keep the c-file open so that ideas of Bc1-f4 and Ra1-c1 come into the
equation. After 8 Qxc3, Black’s options are:
a) 8 ... c5 doesn’t make sense, as the g7-pawn is lost after 9 dxc5 Bxc5 10 Qxg7.
b) 8 ... Nd7 9 Bg5 Be7 10 Bxe7 Kxe7 is acceptable, but having the king in the centre
is not to everyone’s taste.
c) 8 ... h6 9 Bf4 Bd6 10 Bg3 0-0 11 e3 Nd7 12 Bb5 Bxg3 13 hxg3 c6 14 Ba4 Rc8 15
Rd1 Ba6 16 Bc2 c5, as played in J.Markos-B.Macieja, Pardubice 2002, was fine for
Black, as an attempt to take advantage of the d-file with 17 dxc5 Rxc5 18 Qd2 fails to
18 ... Qc8! and Black is better, as White still cannot get his king to safety.
8 ... c5 9 e4 Nc6
9 ... Nd7 is the safer (and most popular) route, where the early d4-d5 push has less
effect. After 10 Bd3 Qc7 11 Qb1 Be7 12 0-0 0-0 13 e5 h6 14 a4 Rfd8 15 Be3 Bd5 16
Nd2 cxd4 17 cxd4 Qb7 18 f3 Rac8 there was rough equality in I.Sokolov-S.Tiviakov,
Dutch League 2005.

10 Bb2

Question: Why put the bishop here, rather than play 10 Be3?

Answer: White pre-empts the idea that Black will exchange on d4 to open up the c-file, and then the b2-bishop
will eye up the g7-square with a future d4-d5 break.
10 Be3 was played in this high level game I.Sokolov-V.Kramnik, Wijk aan Zee
2004, which continued 10 ... Be7 11 d5 exd5 12 Rd1 d4 13 cxd4 0-0 14 d5 Nd4 15
Qb2 Re8 16 Bxd4 cxd4 17 Bb5 Bxa3 18 Qxd4 Bc5 19 Qd3 Bb4+ 20 Kf1 Rf8 21 Nd4
Qc8 22 h4 and the position was hugely unclear.
10 ... Rc8 11 Rd1 cxd4 12 cxd4 Bd6
12 ... Nxd4 13 Qa4+ loses the knight.
13 d5 exd5 14 exd5
This position already looks dangerous – but for who? There is a tasty discovered
attack down the c-file, but both kings will come under fire quite quickly.
14 ... Qe7+ 15 Be2 Na5 16 Qa4+ Kd8
16 ... Qd7 17 Bb5 would not be recommended!
17 Kf1
Likewise, White follows suit by nudging his king out of the way. Of course, he could
not castle as then he would drop the e2-bishop.
17 ... Rc5 18 Nd4 Qd7
18 ... Rxd5 19 Bf3 Rc5 20 Bxb7 Qxb7 looks fraught with danger down the d-file,
even if the computer says Black can hold with variations like 21 Nb5 Qa6 22 Rxd6+
Kc8.
19 Bb5?!
19 Qxd7+ Kxd7 would enable Black’s king to feel relatively safe in the centre, now
that the queens are exchanged off, but is also safer for White.
19 ... Ba6!
A fantastic shot, possibly this was missed by White?
20 Nc6+ Qxc6
The queen sacrifice looks beautiful, but actually it was forced in this position as the
alternatives were worse: 20 ... Nxc6 21 Qxa6 and White is better, while after 20 ...
Rxc6 21 dxc6 Qxc6 22 Qh4+ Kc7 23 Bxa6 White is simply a rook ahead.
21 Qh4+ Kc7
21 ... Be7? 22 dxc6+ and White wins.
22 dxc6 Rxb5 23 Kg1
The b2-bishop could not be saved, due to the threat of discovered check by the b5-
rook.
23 ... Rxb2
Three pieces for the queen is acceptable material, but with both kings still open the
queen can be formidable in these types of positions.
24 Qg4 Nxc6?
24 ... Rd8 leaves Black better.
25 Qxg7 Be5 26 Qxf7+ Kb8 27 f4??
27 h4 was far better, and the position is roughly equal. The game move weakens the
king just too much.
27 ... Bd4+ 28 Rxd4 Nxd4 29 h4
29 Qg7 Re8 30 h3 Nf3+ 31 gxf3 Re1#.
29 ... Nf3+! 0-1
As after 30 gxf3 Rc8 followed by ... Rc1 mate cannot be prevented.

Game 49
E.Lobron-A.Karpov
Dortmund 1995

1 d4 Nf6 2 c4 e6 3 Nf3 b6 4 a3 Bb7 5 Nc3 d5


6 cxd5
Besides this main line, there are a few options for White here other than this.
a) 6 Bg5 was covered in Carlsen-Karjakin (Game 47).
b) 6 Qc2 is the kind of move we have discussed many times throughout this book. It
controls the e4-square and waits to see how Black sets up. There are a few options, but
I like taking with ... dxc4 (since White didn’t play cxd5 and fix the centre pawn
structure). We will later play ... c7-c5 to counter in the centre.


After 6 Qc2, Black can choose:
a) 6 ... Be7 7 cxd5 Nxd5 (7 ... exd5 is also possible, transposing to Queen’s Gambit
Declined structures with an early a2-a3) 8 Bd2 0-0 9 e4 Nxc3 10 Bxc3 Nd7 11 0-0-0
which transposes to Gordon-Morrison given later in this chapter. Given that I am not
the biggest fan of Black’s position there, I am recommending 6 ... dxc4 against 6 Qc2.
b) 6 ... dxc4 7 e4 c5 and now 8 d5 is White’s only real try for an advantage, but after
8 ... exd5 9 exd5 Bd6 10 Bxc4 0-0 11 0-0 h6 12 Re1 a6 13 a4 Nbd7 14 b3 Qc7 15 Bb2
Ne5 I quite liked Black’s position in V.Bologan-B.Macieja, Stepanakert 2005.
c) 6 Qa4+ is an attempt to disrupt Black’s coordination, rather than just giving a
check for the sake of it.


There are a number of blocking moves here, so I suggest picking the one that suits
you personally from this lot:
a) 6 ... c6 7 cxd5 exd5 8 Bg5 Be7 9 e3 0-0 10 Bd3 Nbd7 11 0-0 Re8 12 Rad1 h6 13
Bf4 Bf8 was played in M.Petursson-N.Short, Biel Interzonal 1985. Black has control
over the e4-square, so ideas of ... c5 and ... Ne4 are the way forward.
b) 6 ... Nbd7 7 cxd5 Nxd5 8 Nxd5 Bxd5 9 Bg5 Be7 10 Bxe7 Qxe7 11 Ne5 a6 12
Qxd7+ Qxd7 13 Nxd7 Kxd7 14 f3 f5 15 e3 was just equal in Z.Azmaiparashvili-
A.Grischuk, Moscow (rapid) 2002.
c) 6 ... Qd7 7 Qxd7+ Nbxd7 8 Nb5 Bd6 9 b3 Ke7 10 Nd2 a5 11 Nxd6 cxd6 12 a4
Nb8 13 Ba3 Na6 as played in A.Dreev-A.Karpov, Cap d’Agde (rapid) 2000, suits
players who prefer queenless positions.
So the resulting positions after 6 Qa4+ are a matter of taste, personally I like 6 ... c6
to maintain tension and keep the pieces on the board, but I’ll leave it to the reader to
decide their own way in the Queen’s Indian here!
Returning to the main game after 6 cxd5:
6 ... Nxd5 7 e3

Here White is not going for a quick domination of the centre, but will play Bf1-d3,
castle short and only then think about pushing in the centre. This slower approach has
its pros and cons.
7 ... g6
7 ... Be7 is more common, but I like Karpov’s approach in this game. After 8 Bb5+
c6 9 Bd3 Nxc3 10 bxc3 c5 11 e4 0-0 12 0-0 Nd7 13 Qe2 Rc8 will most likely
transpose back to lines with an early ... Nd7 mentioned in the previous game.
8 Bb5+ c6 9 Bd3


Question: What is the idea of playing 8 Bb5 then back
with 9 Bd3? Didn’t White gift Black the extra move 8 ... c6?

Answer: Yes and no. Whilst 8 ... c6 is an ‘extra’ move, it may not be a move Black wants to play as he has now
blocked in the b7-bishop, which can only be opened up again with the ... c6-c5 break. Timing is everything with that
pawn move, as after dxc5, ... bxc5 Black’s c-pawn can become weak and exposed (or sometimes strong, as Karpov
demonstrates).
9 ... Bg7 10 Ne2
Other options for White here:
a) 10 0-0 0-0 11 e4 Nxc3 12 bxc3 c5 and play now resembles a sort of Grünfeld
Defence, except a2-a3 has been played instead of a main line beginning with Ra1-b1
(which is far more dangerous).
b) White might try to head to dinner early and go for a quick attack with 10 h4!? 0-0
11 h5 but Black has 11 ... c5! which is perfect strategy – in principle you should counter
the opponent’s wing attack with a central punch. Following 12 Ne4 cxd4 13 exd4 Nc6
14 hxg6 hxg6 15 Nfg5 f5 the complications were not unfavourable for the second player
in P.Meyer-J.Lautier, Hameln 1987.
10 ... c5!
10 ... 0-0 11 e4 to force the knight back was the idea behind 10 Ne2. Karpov is
having none of it and immediately strikes out.
11 dxc5 bxc5 12 Qc2
Another possibility is 12 e4 Nb6 13 Bg5 Qc7 14 Bf4 e5 15 Be3 N8d7 16 0-0 0-0 17
Rc1 with approximate equality.
12 ... Nd7 13 e4 N5b6


Exercise: Playing Black, what would you do if White castles here?

Answer: There is the tactical trick 14 0-0? c4! 15 Bxc4 Rc8 (Dautov) 16 Nd2 Ba6 winning a piece since now 17
b3 hangs the a1-rook.
14 Nc3
White frees the e2-square for the retreat of his d3-bishop.
14 ... c4 15 Be2 Nc5 16 0-0 Qc7 17 Bg5 0-0 18 Nd2 Rfc8!


Question: Is the c4-pawn strong or weak?

Answer: As discussed earlier, this was what White was counting on with the Bf1-b5-d3 sortie. Here though it
appears that Black’s activity more than compensates for this relative weakness.
19 f4?! a5 20 e5
Trying to block out the g7-bishop, but this can leave holes which Karpov exploits.
20 ... Ba6 21 Nde4
21 Rac1 Nd3 brings the knight onto the ‘octopus’ square. From there it causes all
sorts of problems, for example ... Qc5+ is threatened, and if the knight is chopped off
with Be2xd3, then ... cxd3 will create a monster passed d3-pawn.
21 ... Nxe4 22 Nxe4 Nd5
An excellent square for the knight, controlling the f6-square and eyeing up the e3-
and c3-squares.
23 Qd2
23 Nd6 Rcb8 24 Rf2 h6 25 Bh4 c3 and the white pieces and pawns are hanging all
over the place.
23 ... Rab8 24 Nd6 Qb6+ 25 Kh1 Rc5 26 Rac1 Qxb2 27 Qxb2 Rxb2 28 Bxc4
Bxc4 29 Nxc4
This move was criticised, but White is worse in all variations. He just has too many
weaknesses, beginning with the f4-pawn.
If 29 Rxc4 Rxc4 30 Nxc4 Ra2 31 Nxa5 h6 32 Bh4 Nxf4! the knight is immune due to
mate on a1. Now the e5-pawn and/or the a3-pawn will fall as well.

Exercise: Black’s b2-rook is attacked, where to go with it?

Answer: The temptation is to keep it on the 2nd rank (which is still strong) but Karpov is precise with his ideas.
29 ... Rb8! 30 Nd6
30 g3 Rbc8 wins the knight.
30 ... Rxc1 31 Rxc1 h6 0-1
Ironically, White chose to resign after this move, which Karpov had threatened all
game. The f4-pawn will go, and the rest will collapse like a pack of cards.

Game 50
R.Bagirov-M.Palac
European Team Championship, Batumi 1999

1 d4 Nf6 2 c4 e6 3 Nf3 b6 4 a3 Bb7 5 Nc3 d5 6 cxd5 Nxd5 7 Bd2


This looks like a passive square for the bishop, but the idea is to recapture on c3
with the bishop once Black eventually plays ... Nxc3.
7 ... Nd7 8 Qc2
8 e4? Nxc3 9 Bxc3 Bxe4 again does not work for White.
8 ... c5 9 e4
9 Nxd5 Bxd5 10 e4 Bb7 11 Rd1 cxd4 12 Nxd4 a6 13 Bc3 Qc7 (A.Anastasian-
R.Dive, Moscow 1994) transposes to the position reached in the main game after 12 ...
Qc7.

9 ... Nxc3

Question: You are always exchanging on c3 in this chapter.
Why can’t we just retreat the knight, to say f6?

Answer: This loses key time and the following variations demonstrate that:
After 9 ... N5f6 White plays 10 d5! which is very thematic. Since the knight
retreated, White plays aggressively to blast open the centre. After 10 ... exd5 11 Nxd5
Black has a choice of recaptures:
a) 11 ... Nxd5 12 exd5 and if 12 ... Bxd5? 13 0-0-0 when White has tremendous play
down the d-file, for example 13 ... Be7 14 Bc3 Nf6 15 Bb5+ with a big advantage.
b) 11 ... Bxd5 12 exd5 Bd6 (or 12 ... Nxd5? 13 Bc4 Nc7 14 0-0-0 again with great
compensation for the pawn) 13 Bb5 0-0 14 0-0 Ne5 15 Ng5 h6 16 Ne4 Nxe4 17 Qxe4
f5 18 Qe2 gave a small edge to White in B.Jobava-Z.Almasi, Novi Sad 2009, as the
two bishops have a nice open position to work with.
10 Bxc3 cxd4 11 Nxd4 a6 12 Rd1

12 ... Qc7
A sensible square for the queen. Now in some lines the pin on the c3-bishop is a
useful defensive mechanism.
13 Be2 Be7 14 0-0 0-0 15 f4
15 f3 and playing in a more restrained fashion was to be considered.
15 ... Rac8

16 Kh1
The sort of ‘passing’ move one can easily make, to await developments. However
this is not really in keeping with his aggressive thrust 15 f4.
Instead after 16 e5 Black has:
a) 16 ... Nc5 17 Bf3 Bxf3 18 Rxf3 when White has dreams of f4-f5 and/or Rf3-h3 to
attack.
b) Maybe Bagirov was concerned with the simplifying 16 ... Bxa3 17 bxa3 Qxc3 18
Qxc3 Rxc3 19 Nxe6 fxe6 20 Rxd7 Bd5 21 Bxa6 Rxa3 when Black is probably a bit
better as he has the outside b-pawn and better bishop. Still, this is likely a draw with
best play.
16 ... Nc5
16 ... Bxa3 is tricky, but White has counter chances with 17 Qa4 Bd6 18 e5 Bc5 19
Nxe6 and if Bc6?! (if 19 ... fxe6 20 Rxd7 is about equal) 20 Nxc7 Bxa4 21 Ra1 Rxc7
22 Rxa4 a5 23 Rd1 when White’s two bishops and greater space far outweigh Black’s
queenside pawn majority, which isn’t going anywhere for a while.
17 e5
17 Bf3? defends the e4-pawn, but drops another pawn to 17 ... Qxf4.
17 ... Be4 18 Qd2 Qb7 19 Bg4
19 Bf3 Rfd8 is comfortable for Black.
19 ... Rfd8 20 Qe2 g6


Question: Why are we weakening our kingside structure like this?

Answer: This move prevents the f4-f5 attacking break, and it is not clear how White can take advantage of any
dark square weaknesses on the f6-, g7- and h6-squares any time soon. Grandmasters often make this ... g6 defensive
move if they are confident their opponent cannot make inroads on those key squares.
21 Bb4 Rc7 22 Rfe1
Now f4-f5 is off the cards (was it ever on?).
If 22 Rd2 a5 23 Bc3 Na4! then with the typical solid structure and slightly more
active pieces, Black can be confident of playing for a win here.
22 ... Bd5 23 Bf3 Rcd7
Now the f4- and e5-pawns are potential targets, just as we saw in the previous game
Lobron-Karpov.
24 Bxd5 Rxd5 25 Bc3
Or 25 Nf3 Nd3! 26 Rxd3 (26 Rf1 Bxb4 27 axb4 Nf2+! is a neat tactic to win
material) 26 ... Rxd3 27 Bd6 Rd5 with a clear extra exchange.
25 ... Na4! 26 Qg4
After 26 Nc2 Nxc3 27 bxc3 Rxd1 28 Rxd1 Rxd1+ 29 Qxd1 Qe4 Black is better as
the queen is very active, pressurising the weak f4-pawn. The e7-bishop is also superior
to the c2-knight, which has no real prospects. There would be a long defensive task
ahead of White from here on in.
26 ... Qd7

Such harmony deserves a diagram. All the black pieces are defending each other and
are placed for a purpose. Black has pressure down the d-file and on the c3-bishop and
d4-knight. White didn’t even do much wrong, but he has been slowly outplayed
‘Queen’s Indian’ style.
27 Rd3 Bc5
27 ... Bxa3! 28 bxa3 Nxc3 29 Rxc3 Rxd4 30 h3 Rd1 was also just a clear pawn
advantage, but the Croatian Grandmaster was probably having fun here.
28 Red1 Bxa3!
Finally!
29 Rh3
29 bxa3 Nxc3 30 Rxc3 Rxd4 31 Rf1 Rd1 is similar to 27 ... Bxa3 in the earlier note.
29 ... Bxb2 30 Qh4

Question: Uh oh, White has a threat ...

Answer: Never panic, our harmonious and solid position should offer defensive resources.
30 ... f5
30 ... h5 should be enough. Obviously 31 g4 to prise open the kingside is a
possibility, but it probably isn’t quick enough given White will lose most of his army in
the centre. After 31 g4 Bxc3 32 gxh5 Rxd4 33 Rg1 Qc6+ and Black gives mate with ...
Rd1.
31 Bb4
31 exf6 Bxc3 with no compensation for the piece.
31 ... Nc5!
Black is vigilant right to the end and denies deny the opponent counterplay. After 31
... Bxd4 32 Be7 h5 33 Rg3 the computer calmly says Black is still winning, but to us
mortals this looks a bit dangerous!
32 Qh6 Rxd4 33 Rxd4 Bxd4 0-1
I very much liked this game – Black did nothing spectacular, but slowly increased
his advantage in the middlegame until his opponent cracked.

Game 51
S.Gordon-G.Morrison
British League 2006

1 d4 Nf6 2 c4 e6 3 Nf3 b6 4 Nc3 Bb7 5 a3 d5 6 cxd5 Nxd5 7 Bd2 Be7


Personally, I prefer the main line with 7 ... Nd7 and 8 ... c5 that we saw in the
previous game, as here we see White get a big centre and castle early, which I feel
passes him the initiative.

8 Qc2 Nd7 9 e4 Nxc3 10 Bxc3


So this was the point of 7 Bd2, and now that the b2-pawn stays in its place, castling
queenside is an added option.
10 ... 0-0 11 0-0-0
White can choose to castle kingside and play it safer, for example 11 Rd1 Rc8 12
Bc4 Nf6 13 Bd3 c5 14 dxc5 Rxc5 15 0-0 Qa8 16 Rfe1 Rfc8 17 Qe2 was played in
A.Riazantsev-M.Carlsen, European Championship, Warsaw 2005, with an unclear and
tense middlegame to come.

Question: You are always advising Black to play ... c5, so can we do it here?

Answer: Yes, but with White’s rook on d1 our queen on d8 looks a little shaky.
After 11 ... c5 White can choose between:
a) 12 Bb5 Qc7 13 d5 exd5 14 exd5 Rfd8 15 Rhe1 Nf8 16 Qe4 Ng6 17 h4 which
gave White the initiative in J.Piket-L.Polugaevsky, 4th matchgame, Aruba 1994.
b) 12 d5 is also playable. Following 12 ... exd5 13 exd5 Bf6 14 Bd3 g6 15 Bb5
Bxc3 16 Qxc3 Nf6 17 Bc6 Ne4 18 Qe5 Nd6 19 h4 an attack was brewing on the
kingside in K.Hulak-I.Farago, Slovenian League 1996.
Another option for Black is 11 ... c6 with the idea of a pawn storm with ... b5, ... a5
and ... b4. This looks slow but since White has played a2-a3, Black has something to
hook onto. Following 12 h4 b5 13 Rh3 a5 14 d5 cxd5 15 Bxb5 Nf6 16 Ng5 Qb6 the
position was wholly unclear in G.Kamsky-V.Anand, 4th matchgame, Sanghi Nagar
1994. However, it could be argued that Black achieved a lot more play in this line
compared to the main game, so this line might justify further investigation.
In the main game, Black instead played:
11 ... Qc8


Question: What is the reason behind Black’s last move 11 ... Qc8?

Answer: To move the queen off the d-file, so ... c5 can be played. Black’s plan is a bit slow, however.
12 h4 c5?!
Black’s timing is off for this thrust, which now only seems to help White by opening
up the position. Instead 12 ... Rd8 followed by ... Nd7-f8 is more solid.
13 d5 exd5?!
This opens up more lines for the White pieces.
14 exd5
This d-pawn is mobile, and there is no counterplay down the c-file.
14 ... Ba6?!
Instead:
a) 14 ... Re8 or 14 ... Bf6 was relatively best, but White is still much better.
b) 14 ... h6 doesn’t help Black’s defences as White can play 16 d6 or even 16 Ng5!
anyway, and White has a big attack.
15 Ng5

15 ... f5
15 ... Bxg5+ is hardly an option, opening up even more channels towards the black
king after 16 hxg5.
15 ... g6 allows the thematic sacrifice 16 Nxh7! Kxh7 17 h5 with a huge attack.
16 Bxa6 Qxa6 17 Ne6
Black is already lost, as he cannot defend against the twin threats on g7 and c7.
17 ... Rf7
17 ... Rfc8 18 Nxg7 is crushing.
18 Nc7 Qb7 19 Nxa8 Qxa8 20 d6 Bf8 21 f3 Qc6 22 Qd3 c4 23 Qd5
Wisely swapping off to a winning endgame. The game concluded:
23 ... Qxd5 24 Rxd5 Nc5 25 Rhd1 Rd7 26 Kc2 g6 27 R1d4 Kf7 28 Rxc4 Rxd6 29
Rxd6 Bxd6 30 Bb4 Ke6 31 Bxc5 Bxc5 32 b4 Bd6 33 Rc8 a5 34 Kb3 1-0
A powerful example of what happens if Black does not challenge the centre quickly enough with ... c5.
Chapter Eight
4 Nc3 Bb7: Other Fifth Moves
1 d4 Nf6 2 c4 e6 3 Nf3 b6 4 Nc3 Bb7

In the previous chapter, we looked at 5 a3, when White takes the game into the
Petrosian Variation. In this chapter we focus on two of White’s 5th move alternatives
namely 5 Bg5, which is covered in Games 52-54, and 5 Qc2, which is reviewed in
Games 55-58. In the latter line, after 5 Qc2 Black has a choice between 5 ... c5 (Games
55-56), staying within pure Queen’s Indian territory, and 5 ... Bb4 (Games 57-58)
which transposes to a line of the Nimzo-Indian. In these games I give details of both
approaches so you can make an informed repertoire choice.

Game 52
J.Menadue-P.Wells
Penarth 2011

1 d4 Nf6 2 c4 e6 3 Nf3 b6 4 Nc3 Bb7 5 Bg5


Here White is trying to do without a3 and fight for the e4-square by pinning the f6-
knight.
5 ... Be7

Question: This doesn’t make sense. Surely if we can play the bishop to b4, we should?

Answer: Yes 5 ... Bb4 is fine, but this move breaks the pin and in some lines allows Black to play ... Ne4 like in
the Piket-Anand game (Game 5 in Chapter One). In fact 5 ... Bb4 transposes to the main lines of the Nimzo-Indian
and for this reason I have opted to recommend a Queen’s Indian style line instead.
6 e3
6 Bxf6 Bxf6 7 e4 gaining a big centre looks impressive, but giving up the dark-
squared bishop might be too much to ask given the position is likely to open up soon
enough. After 7 ... d6 8 Qc2 g6 9 Be2 0-0 10 h4 c5! White played 11 dxc5 dxc5 12 e5
Bg7 13 Nb5 but this is far too ambitious. A.Czebe-Z.Szabo, Budapest 1998 continued
13 ... Nd7 14 Rd1 Qe7 15 Qc3 Bxf3 16 Bxf3 Rad8 17 Qe3 Nxe5 and Black took over.
6 ... 0-0 7 Bd3


Exercise: It’s about time Black challenged the centre. Which pawn move to play?

Answer: Either of the usual two, 7 ... c5 or 7 ... d5 are playable, but I like the way Wells goes for a Hedgehog
style setup and leaves the d-pawn on the modest d6-square.
7 ... c5
7 ... d5 8 0-0 Nbd7 9 Rc1 h6 10 Bh4 c5 11 Qe2 transposes to the Tartakower
Variation of the Queen’s Gambit Declined, which unfortunately is outside the scope of
this book (but sound nonetheless).
8 0-0
8 d5? Nxd5! is the trick. Remember this one when the bishops are on e7 and g5!
8 ... d6
8 ... cxd4 9 exd4 d5 will likely give White an IQP on d4, but again I prefer the
Hedgehog setup which offers more winning chances for Black.
9 Qe2 Nbd7 10 Rfd1 Rc8 11 Rac1 Re8 12 b3 a6
A useful move, it prevents Nc3-b5 and at some point Black will aim for the ... b5
break.
13 dxc5
13 e4 cxd4 14 Nxd4 Qc7 would lead to a standard Hedgehog vs. Maroczy Bind
setup.
13 ... Rxc5 14 Bh4 Qa8 15 Bb1


Exercise: Black has a fine position, but how to proceed further?
Decide between the pawn breaks ... b5 and ... d5.

Answer: As usual, both are acceptable but if playing for a win, it is a good idea to try and unbalance the position a
bit. Therefore ... b5 will be the one to aim for, as it exchanges off a wing pawn for a more central pawn.
15 ... b5!
15 ... d5 16 cxd5 Nxd5 17 Nxd5 Bxd5 18 Bxe7 Rxe7 19 Rxc5 Nxc5 is no problem,
and a good alternative to have, but if you want to play for a win (and I am sure you do)
then unbalancing the position is what is required.
16 cxb5?! Rec8
This is the crucial follow up. Since White played b2-b3 earlier, the c3-knight is now
very unstable.
17 bxa6 Bxf3 18 Qxf3 Qxf3 19 gxf3 Rxc3 20 Rxc3 Rxc3

So Black has won a piece, and it now is a question of whether he can hold back the
queenside passed pawns.
21 Bxf6 gxf6 22 Rd4 d5 23 a7 Rc8 24 Rg4+ Kh8 25 Ra4 Ra8 26 b4
Preventing ... Bc5 winning the a7-pawn, but with care from Black the pawns should
drop now anyway.
26 ... Nb6 27 Ra6 Nc8 28 b5 Bd8
Avoiding the last trick. Black should not go for 28 ... Rxa7 29 b6 Rb7 (29 ... Rxa6?
30 b7 Rc6 31 b8Q) 30 Ra8 which gets White out of jail.
29 Rc6 Nb6
Not 29 ... Nxa7 30 Ra6! Rc8 31 Rxa7 Rc1+ 32 Kg2 Rxb1 33 Ra8 and White regains
the piece with approximate equality.
30 Rd6 Bc7 31 Rc6 Rxa7
Now it should be game over. The tricks are all gone!
32 Bc2 Kg7 33 a4 Nc4 34 Kg2 Ne5 35 Rc3 Ba5 36 Rb3 Rc7 37 Bd1 Rc1 38 Be2
Rc2 39 Kf1 Nc4 40 Bxc4 dxc4 41 Rb1 c3 42 b6 Rb2 0-1
Great play from Peter Wells who understands these Nimzo and Queen’s Indian
structures in great detail.

Game 53
Y.Yakovich-B.Jobava
Russian League 2007

1 d4 Nf6 2 c4 e6 3 Nf3 b6 4 Nc3 Bb7 5 Bg5 Be7 6 Qc2


The previous game saw the more modest 6 e3 and 7 Bd3, but here White harbours
the intention of possibly playing e2-e4 in one go. The drawback, as we have seen in
many lines throughout this book, is that the Queen on c2 means that after a ... c5 push,
the counter d4-d5 is not as strong since the queen will not be protecting the d5-pawn.
6 ... h6 7 Bh4 c5!
This is definitely the right moment to counter in the centre, for reasons explained
above. 7 ... 0-0 8 e4 was absolutely not the plan at all – White has achieved everything
he wants, and can look to attack immediately with Bf1-d3 and e4-e5 etc.
8 dxc5 bxc5 9 e3
If 9 e4 Nc6 and the weakness of the d4-square might cause White a few headaches in
the future.
9 ... 0-0 10 Be2 d6 11 0-0 Nbd7 12 Rfd1 Qb6
In my view, these types of positions are very pleasant for Black. Potential pressure
down the b-file, a chance to push ... d5 at the correct time, and though White has a
harmonious setup it is not clear how he can improve things further for himself.
13 Rd2 Rfd8 14 Rad1 Nf8 15 h3 Rd7 16 Ne1
Trying to exchange off the light-squared bishops with Be2-f3.
16 ... Rad8 17 Bf3 Ng6 18 Bg3 Bc6 19 Bxc6 Qxc6

Notice that Black still keeps an eye on the key d5- and e4-squares.
20 Nf3 a6 21 Bh2 Rb8 22 b3 Rbd8 23 Qd3
23 e4 preventing the ... d5 break was to be preferred.
23 ... d5
Of course, freeing up Black’s position. Whilst a hanging pawns structure now
occurs, this is not unfavourable for the second player because he is well developed and
everything is defended.
24 cxd5 exd5 25 Qf5?
The queen ends up being exposed here.
25 ... d4!
Jobava is ready to go forward like this, as his pieces are all optimally placed. Still,
it is hard to see how White could lose this game so quickly!
26 exd4 cxd4 27 Na4 d3 28 Nb2
Forgetting that tactics exist in such positions ...
28 ... Rd5 29 Ne5


Exercise: Can you see a tactic for Black here?

Answer: The answer lies in the g2-square!
29 ... Rxe5 0-1
After 30 Bxe5 Nh4! Black has a double attack threatening mate on g2 and the queen.

Game 54
A.Pettersson-S.Brynell
Rilton Cup, Stockholm 2010

1 d4 e6 2 c4 Nf6 3 Nf3 b6 4 Nc3 Bb7 5 Bg5 h6 6 Bh4 Be7 7 Qc2 c5


We saw this important pawn break in the previous game.

8 e4
Deviating from 8 dxc5 which was played in the previous game.
8 ... cxd4 9 Nxd4 d6 10 Be2 Nbd7
The d7-square is where this knight belongs in the Hedgehog setup.
11 0-0 a6 12 f3 0-0 13 Bf2 Qc7 14 Rac1 Rac8 15 a3 Qb8 16 b4 Rfe8

It is important to understand what the various plans are for each side within the
Hedgehog setup. White will play primarily on the queenside, and look to break with c4-
c5 and open up the position at some moment. For Black, the key is to try and go for ...
d5 and ... b5 to open up the position in his favour, though the Maroczy Bind structure
with the pawns on c4 and e4 are designed to try and prevent this. Often a game of cat
and mouse takes place over the following 10 moves or so, with both sides jostling to
prevent their opponent’s plan whilst simultaneously getting their own plan in. I find this
a great way to play for a win though, since most of the pieces remain on the board.
A final plan I should mention, whilst much rarer, is the idea that Black can in fact
play for a kingside attack with ... Kh8, ... Rg8 and ... g5-g4. This looks risky, but if
timed correctly, can be very effective. Generally this occurs when White moves around
aimlessly and demonstrates no active plan.
17 Qd2 Bd8
The idea from here is to play ... Bc7 and threaten a timely ... d6-d5 attacking the h2-
pawn.

18 Be3 Kh7 19 Qc2 Kh8 20 Qd2 Bc7


No repetition draw with 20 ... Kh7, Black is playing for a win!
21 Kh1 Rcd8
Breaking now with 21 ... d5 22 cxd5 Bxh2 is a bit too early, as it allows 23 f4 Bg3
24 Rf3 Nh5 25 b5 axb5 26 Bxb5 when the black minor pieces are scattered all over the
place.
22 Rcd1 Nf8 23 Bd3 Ng6
Now White is the first one to lose patience and go for a sacrifice.
24 Bxh6 gxh6 25 Qxh6+ Nh7 26 f4 Rg8
Black has his pieces ready for the defence. Now it is up to the first player to prove
he has compensation for the sacrificed piece.
27 f5 Ne5
An excellent square for the knight.
28 fxe6 Rg6 29 Qh5 fxe6 30 Nf3?
White could stay in the game with 30 Rf7!
30 ... Rg7 31 Nd4
31 Nxe5 dxe5 is no better, as there is no decent attacking continuation here.
31 ... Qc8 32 Qh3 Re8 33 Be2 Kg8 34 Rf2 Ng5 35 Qh4 Bd8

The defence is successful and White’s attack has run out of steam. Picking the
opponent off like this is typical Queen’s Indian play.
36 Nd5 exd5 37 exd5 Qd7 38 Rdf1 Ngf3 39 Qe4 Rg4
A few clever tactics and it is game over.
40 Bxf3 Rxe4 41 Bxe4 Bc8 42 Ne6 Qe7 0-1

Game 55
P.Short-S.Brady
Irish Championship 2009

1 d4 e6 2 c4 Nf6 3 Nf3 b6 4 Qc2 Bb7 5 Nc3


The last two moves make sense in that 6 e4 is now a threat to gain central
domination, but Black can cut across this plan in a number of ways.
5 ... c5
5 ... Bb4 transposes to a Nimzo-Indian line which is examined in Games 57-58.
6 e4
6 dxc5 is the subject of the next game.
6 ... cxd4 7 Nxd4 Bc5
7 ... d6 8 Be2 Be7 9 0-0 0-0 10 Rd1 Nbd7 11 Bg5 a6 12 Rac1 Rc8 13 Qb1 Re8 14
Nf3 Qc7 15 b4 Qb8 and in G.Kuzmin-Z.Ribli, Bled 1979, it was Hedgehog time again!
8 Nb3 Nc6
We saw this idea of controlling the d4-square in Chapter Six. Black is not concerned
with White exchanging with Nb3xc5 because it will only increase his control over the
d4-square after ... bxc5.
9 Bd3 0-0 10 0-0 Rc8 11 Qe2

Question: What is White’s reason for playing this move 11 Qe2?

Answer: To get the queen off the dangerous c-file, and threaten e4-e5.
11 ... e5
11 ... Be7 12 e5 Ne8 looks passive but even here Black can gain approximate
equality with 13 Bf4 f6! 14 exf6 Nxf6 and there are chances for active play down the f-
file, at the very least.
12 Be3 d6 13 Rad1 Ne8 14 Nd5 Nc7 15 Nxc7 Rxc7

16 Nd2
Black shouldn’t fear this exchange 16 Nxc5 bxc5, as we have seen before, as ... Nd4
is coming and the white bishops are not so effective in this blocked position.
16 ... Re8 17 Nb1 Re6 18 Nc3 Rd7 19 Nd5 Rg6
It looks like Black is harbouring ideas for an attack ...
20 Bc2 Nd4 21 Bxd4
21 Qd2?? Nf3+ would obviously be one to avoid.
21 ... Bxd4 22 Ba4


Exercise: Did Black just lose an exchange? The d7-rook is trapped.

Answer: No! Black gains a tempo to free the d8-square for the rook.
22 ... Qg5 23 g3 Rd8 24 Kg2 Bc8
Around about here, attacking ideas will certainly be swirling around in Black’s
head. White doesn’t have so many defenders and there is a chance to push the h-pawn
and prise open lines.
25 Qd2 Qh4 26 Kh1 Bh3??
Was this really played? 26 ... Qh5 loses the exchange to 27 Ne7+ but play is similar
to the game continuation.
27 Rfe1??
The queen could have been taken! After 27 gxh4 27 ... Bg2+ 28 Kg1 Bf3+ 29 Qg5!
Rxg5+ 30 hxg5 Bxd1 31 Rxd1 Bxb2 White is a piece up.
27 ... Bg4 28 Qd3 Qh5
If 28 ... Bxd1 White simply takes the queen with 29 gxh4.
29 Ne7+
Black loses an exchange, but it was more like a sacrifice as Black certainly had the
final combo in mind ...
29 ... Kf8 30 Nxg6+ hxg6 31 Rd2 Bf3+ 32 Kg1

32 ... Ke7
An innocuous king move?
33 Bd1??
33 h3 Qg5 34 Qxf3 Qxd2 35 Re2 was the only way for White to hold. Now White
goes down to a stunning combination.
33 ... Qxh2+! 34 Kxh2 Rh8+ 35 Kg1 Rh1# 0-1

Game 56
I.Sokolov-I.Stohl
Elenite 1992

1 d4
The game actually started from a Réti move-order 1 Nf3 Nf6 2 c4 b6 3 Nc3 Bb7 4 d4 e6 5 Qc2 c5 6 dxc5 Bxc5
reaching the diagram below.
1 ... Nf6 2 c4 e6 3 Nf3 b6 4 Nc3 Bb7 5 Qc2 c5 6 dxc5 Bxc5
7 Bg5
Other 7th moves for White:
a) 7 e4 Nc6 8 Bg5 h6 9 Bh4 Nd4 10 Nxd4 Bxd4 11 f3 Qb8 12 Bg3 Be5 was played
in D.Przepiorka-A.Alekhine, Bad Pistyan 1922, with play similar to Khenkin-Adams
(Game 43) from Chapter Six.
b) 7 Bf4 0-0 8 e3 d5 9 a3 Nbd7 10 cxd5 Nxd5 11 Nxd5 Bxd5 12 Bb5 Nf6 13 Rd1
Qc8 14 Qe2 a6 15 Bd3 Qb7 16 0-0 h6 17 b4 Be7 with a completely acceptable
position for Black, R.Kempinski-K.Edvardsson, European Club Cup, Rethymnon 2003.
7 ... 0-0 8 e3 Be7 9 Be2 h6 10 Bh4
10 ... d5
10 ... Na6 with the idea of ... Nc5 controlling the e4-square, similar to Piket-Anand
(Game 5 from Chapter One), was to be preferred. 11 0-0 Nc5 12 Rfd1 Nfe4 13 Bxe7
Qxe7 14 Nd2 Nxc3 15 Qxc3 d5 (15 ... d6 16 Bf3 a5 is also playable) 16 Bf3 Rfd8 17
h3 dxc4 18 Bxb7 Nxb7 19 Nxc4 Rac8 20 Qa3 Qxa3 21 Nxa3 Nc5 was completely
equal in T.Tolnai-A.Adorjan, Hungarian Championship 1992.
11 0-0 Nbd7 12 Rfd1 Rc8 13 Rac1

Both sides have developed logically, although the position of Black’s queen on d8
could be a slight cause for concern, given the effectual placement of White’s rooks on
c1 and d1.
13 ... Qe8?!
A natural reply to get the queen off the d-file, and now play becomes sharp.
13 ... dxc4 14 Ne5 Nxe5! 15 Rxd8 Rfxd8 would have given full compensation for
the queen. Black can expand on the queenside with ... a6 and ... b5, as well as throw the
knight into the excellent d3-square. For example 16 Rd1 a6 17 Bg3 Rxd1+ 18 Qxd1
Nd3 19 Bxd3 (or 19 Qc2 Nb4 20 Qb1 b5) 19 ... Rd8 and in both cases Black maintains
a strong position.
14 cxd5 Bxd5
14 ... Nxd5 was another option, however this is where the tactics begin to fly!
Here 15 Nxd5! an easy move to miss, and then Black has to find a move:
a) 15 ... Rxc2 16 Nxe7+ Kh8 17 Rxc2 g5 18 Rc7 Bxf3 19 Bxf3 Ne5 20 Bg3 Nxf3+
21 gxf3 with too many pieces for the queen.
b) 15 ... Bxd5 16 Qxc8 Qxc8 17 Rxc8 Rxc8 18 Bxe7 wins a piece.
c) 15 ... exd5 16 Qa4 when Black has a passive position and the d5-pawn is a
weakness, not a strength.
d) 15 ... Bxh4! is a rather unnatural move, but is the only way for Black to stay in the
game.
15 Qa4 Bc6
15 ... a5 only makes things worse, for example 16 Bb5 g5 17 Nxd5 exd5 18 Ne5
with an almost crushing position.

16 Qxa7?!
16 Qa6! is the computer’s suggestion, as if 16 ... Nc5 17 Qxa7 Ra8 18 Qc7 the queen
can escape with the loot (the a7-pawn) in the bag.
16 ... g5 17 Nd4
White sacrifices a piece, but 17 Bg3 allows a draw by repetition with 17 ... Ra8 18
Qc7 Rc8 19 Qa7 Ra8 which Black should accept in this position!
17 ... gxh4 18 Nxc6 Rxc6 19 Bb5 Rd6 20 Qa4
A key moment. Can White regain the sacrificed piece and take advantage of Black’s
weakened kingside?
20 ... h3!?
Black tries to open lines of his own leading to the White king. White would win the
piece back after, for example 20 ... Qd8 21 Rxd6 Bxd6 22 Bxd7 Nxd7 23 Rd1 Ne5 24
Ne4 Qa8 with a dynamically balanced position according to our silicon friend.
21 gxh3?
White should ignore this pawn and simply get his piece back with 21 Rxd6 Bxd6 22
Rd1.
21 ... Qd8 22 Rxd6 Bxd6 23 Bxd7 Nxd7 24 Rd1
24 ... Nc5?
The wrong square for the knight. Instead 24 ... Ne5! aims at the kingside and covers
critical squares. After, for example 25 Nb5 (or 25 Ne4 Qh4!) Qg5+ 26 Kh1 Qh5 27
Nxd6 Qf3+ 28 Kg1 Rd8! the tables have turned and Black has a winning position.
25 Qc6 Qg5+ 26 Kh1
Now 27 Rg1, pinning the black queen, is also a threat.
26 ... Bxh2?
26 ... Kh7 27 Qxd6 Rg8 28 Qc6 Nd3 29 Qf3 f5 was the last chance to continue with
counterplay. For the two pawns Black is creating problems which are not easy to solve.
27 Kxh2 Kh7 28 Qf3 Qe5+ 29 Kh1 f5 30 Qf4 Qxf4 31 exf4 Ne4 32 Nxe4 fxe4 33
Rd6
The rook ending is easily winning for White.
33 ... Rxf4 34 Kg2 b5 35 Rxe6 Kg7 36 Re5 b4 37 Rb5 e3 38 fxe3 Re4 39 Kf3 1-0

Game 57
U.Adianto-N.De Firmian
Biel 1995

1 d4 Nf6 2 c4 e6 3 Nf3 b6 4 Nc3 Bb7 5 Qc2 Bb4


Unlike in the Petrosian variation, White is trying to do without the move a2-a3. So 5
... Bb4 makes sense.
We saw 5 ... c5 in the previous two games.
6 a3 Bxc3+ 7 Qxc3 Ne4

Question: Why does Black play the knight to the e4-square so early?

Answer: We know that e4 is one of the key squares, and the fact that this knight move gains time by attacking the
queen is an added bonus. Finally, it offers an extra possibility to control the e4-square – by playing ... f5 (rather than ...
d5).
8 Qc2 0-0 9 g3
After 9 e3 f5 10 Be2 d6 11 0-0 Nd7 12 b4 Rf6 13 Bb2 Rg6 Black’s strong e4-knight
offers a platform for attack. I particularly liked the following game as an example of
how to play this: 14 g3 Qe7 15 Rad1 Rf8 16 Ne1 Qh4 17 Ng2 Qh3 18 Nf4 and now the
tactics rain over the board: 18 ... Nxg3! 19 fxg3 (or 19 Nxh3 Nxe2#) 19 ... Rxg3+ 20
Kf2 Qxh2+ 21 Ke1 Rxe3 22 Qd2 Qg3+ 23 Rf2 Nf6 24 Bc1 Ng4 0-1 T.Dao-E.Ghaem
Maghami, Asian Continental Championship, Sharjah 2014.
9 ... f5 10 Bg2
10 ... Nf6!

Question: That doesn’t make sense – we brought the
knight to the e4-square then move it back again!

Answer: Actually White had a threat down the h1-a8 diagonal which needs to be dealt with. Since White has
declared his hand with a kingside fianchetto, Black’s light-squared bishop is minor piece that Black wants to be posted
on the e4-square. Then it can fight with the g2-bishop for the central key squares.
If Black plays 10 ... d6 then White has 11 Nd2, and since the b7-bishop is not
defended, this is an annoying trick. Following 11 ... d5 Black does not lose material,
but would rather not to have to play this move. Black’s dark-squared bishop is likely to
be missed, and the e5-square in particular is a glaring weakness.
11 0-0 Be4 12 Qc3 Qe8
12 ... d6 13 b4 Qe8 transposes.
13 b4 d6 14 Re1 Nbd7 15 Bb2 h6
16 Nd2
Eventually this move would have to be played, to fight for the e4-square.
16 ... Bxg2 17 Kxg2 Ne4
17 ... e5 is thematic but maybe De Firmian was a little concerned about voluntarily
helping the b2-bishop to come into the game. After 18 dxe5 dxe5 19 Qf3 Qe6 20 e4 f4
the position is unclear.
18 Qd3
18 Nxe4 fxe4 19 f3 Nf6 with simmering tension – which minor piece is better here?
18 ... Nxd2 19 Qxd2 Nf6
19 ... Qh5 20 f3 Nf6 21 a4 transposes back to the game.
20 f3 Qh5

Exercise: Is Black preventing the 21 e4 break?

21 a4
Answer: Yes. De Firmian has it all worked out.
If 21 e4 then 21 ... fxe4 22 fxe4 Ng4! and Black threatens both 23 ... Rf2 and 23 ...
Qxh2#. After 23 h3 Rf2+ 24 Qxf2 Nxf2 25 Kxf2 Qxh3 with a winning position for
Black, as ... Rf8+ is coming next.
21 ... Rae8 22 a5 e5
So it was Black who got in the first central break! Is the imminently open a-file
going to achieve anything for White? The c7-pawn can easily be defended with a move
like ... Rf7 or ... Re7.
23 dxe5 dxe5 24 axb6 axb6 25 Ra7 Re7 26 b5 Rfe8
26 ... f4 27 gxf4 exf4 28 Qxf4 Nd5 29 Qg3 Nf4+ 30 Kh1 with some compensation
for the pawn.
27 Kg1 Qf7 28 Qc2 f4
Black was only temporarily going backwards. Now the attack is back on.
29 Rea1?
Moving away the last defensive piece from the king was never a good idea,
presumably he underestimated the impending attack?

29 ... e4
Of course!
30 Ra8
30 gxf4 exf3 31 exf3 Re2 and Black wins, as White’s queen can’t simultaneously
defend the b2-bishop and stop both 32 ... Qg6+ and 32 ... Qh5.
30 ... fxg3 31 hxg3 Nh5
What happened to the king’s protection? By committing pieces forward, White has
forgotten to defend his own king.
32 Rxe8+ Rxe8 33 fxe4
33 g4 Nf4 hardly helps.
33 ... Qg6
33 ... Nxg3 is strong, but when you see a good move, always look for an even better
one.
34 Qd3 Nxg3 35 Kh2 Rxe4 36 Rg1
36 Qxg3 is winning no matter what Black does, but one attractive mate is 36 ...
Rxe2+ 37 Kh3 Qe6+ 38 Qg4 Re3+ 39 Kh4 Qe7+ 40 Kh5 Rh3+ 41 Qxh3 Qg5#.
36 ... Rh4+ 37 Kg2 Ne4+ 0-1
As 38 Kf1 Rf4+ is curtains.

Game 58
E.Lobron-B.Thorfinnsson
Reykjavik 2002

1 d4 Nf6 2 c4 e6 3 Nf3 b6 4 Nc3 Bb7 5 Qc2 Bb4 6 Bg5


Fighting for the e4-square by pinning the knight.
6 ... h6 7 Bh4 g5


Question: This 7 ... g5 looks rash, where am I going to castle now?

Answer: Often Black can castle queenside here, but the main point is to get in the quick ... Ne4 and ... f5 like we
saw in the previous game.
8 Bg3 Ne4 9 e3
The most obvious move, but there are a few alternatives:
a) 9 Nd2 Bxc3 10 bxc3 and now there are two options:
a1) 10 ... f5 11 f3 Nxg3 12 hxg3 Nc6 13 g4 Qf6 14 gxf5 Qxf5 15 Qxf5 exf5 16 e4
fxe4 17 fxe4 0-0-0 18 Bd3 d6 19 Ke2 Rdf8 with approximate equality in P.Cech-
J.Parker, Berlin 1998.
a2) 10 ... Nxg3 11 hxg3 Qe7 12 a4 a5 13 e4 d6 14 Bd3 Nd7 15 Nf1 0-0-0 16 Ne3 h5
17 Qe2 was agreed drawn in A.Dreev-Z.Hracek, Sibenik 2009, before play could get
going.
b) 9 Be5 Bxc3+ 10 bxc3 Rg8 Since we are going to castle queenside, this move is
not a problem. Also, it takes the sting out of an early h4 followed by hxg5 as the rook is
not hanging on h8. 11 Nd2 f5 12 Nxe4 Bxe4 13 Qd2 d6 14 Bg3 Bb7 15 e3 Nd7 16 f3
Qe7 17 a4 h5 18 h3 h4 19 Bf2 a5 20 Bd3 0-0-0 with unclear play, T.Hillarp Persson-
J.Rowson, York 1999.

9 ... Bxc3+
9 ... d6 is OK but watch out for that b4-bishop as Qc2-a4+ can be troublesome in
some lines (fortunately not here). Here White can try:
a) 10 Bd3 Bxc3+ 11 bxc3 f5 transposes to the game.
b) 10 Qa4+ Nc6 and now 11 d5? Nxc3 12 bxc3 Bxc3+ 13 Nd2 Bxa1 14 dxc6 Bc8
wins for Black.
10 bxc3 d6 11 Bd3 f5 12 d5!
The best move. It clears the way for the f3-knight to come to the d4-square and attack
the e6- and f5-pawns.
12 ... exd5 13 Nd4 Qf6 14 cxd5 Bxd5 15 f3 Nxg3 16 hxg3 Nd7 17 Bxf5 0-0-0
So White won the pawn back, but an interesting middlegame has arisen. Whose king
is weaker? Was the loss of the f5-pawn so important? How weak is the h6-pawn?
18 Kf2
Quite a safe square for the king.
18 ... Kb8 19 a4


Exercise: As Black here, are we worried about White playing a4-a5?

Answer: I would say yes, we should be. Allowing White the open a-file would only benefit the first player, and
Black’s king would look a little shaky.
19 ... a5!
Now there is no b-pawn, the queenside cannot be prised open with b2-b4. So this is
quite safe.
20 Be4
20 g4 Ne5 is also unclear.
20 ... Qe5 21 Bxd5 Qxd5 22 Rab1 Rde8 23 Rb5 Qc4 24 Rhb1 Ne5 25 Kg1
If 25 Rxa5 Nd3+ 26 Kg1 Rxe3 and now the game gets interesting, for example 27
Rab5 Nc5 28 a5 Rxc3 with chances for both sides.
25 ... g4 26 Rxa5 gxf3 27 gxf3 Rhg8
28 Qh7?
White had to defend first with 28 Kg2, when any result is still possible.
28 ... Nc6!
28 ... Rxg3+ 29 Kf2 Rg7!! also wins, for example 30 Qxg7 Qa2+ picking up the b1-
rook.
29 Rab5
29 Nxc6+ Qxc6 and three pawns are hanging, on c3, f3 and g3.
29 ... Nxd4?
Perhaps due to time pressure, both players miss big opportunities.
29 ... Rxg3+ 30 Kh2 Nxd4! was the way to go.
30 cxd4?
30 Rxb6+! would turn the tables again.
30 ... Qe2!
30 ... Rxg3+ 31 Kf2 Reg8 32 Rxb6+ is similar to the line above.
31 g4
31 Rxb6+ Ka8 was the clever point, as 32 ... Rxg3+ and 32 ... cxb6 are both
threatened.
31 ... Qxf3 32 Rxb6+ cxb6 33 Rxb6+ Kc8 34 Qc2+ Kd7 0-1
The black king is safe, and now ... Rxg4+ will lead to mate.
Chapter Nine
4 e3 and Other Fourth Moves
1 d4 Nf6 2 c4 e6 3 Nf3 b6

There are some other, lesser played variations which are worth seeing in case you
encounter them. By eschewing the main lines from the earlier chapters of this book,
White is hoping to catch the second player off guard. Hopefully by now you will have a
fair idea of how Black should be playing in the Queen’s Indian, and these methods are
usually the same against the other moves (which is a major advantage of this opening).
... c5 and ... d5 are moves we have always considered, and they generally work well
here too. In my view, only 4 e3 (Games 59-64) is any sort of serious test, though strong
players have used 4 Bf4 (Game 65) and so that should be examined as well.
4 e3

This variation can contain more poison than it first appears. White has a simple plan
of Bf1-d3 and 0-0 to develop quickly, and often follows with b2-3 and Bc1-b2 for a
Colle-Zukertort type of position. In essence, White is trying to avoid sharp theory and
instead heads for simple piece development. Rather than going for the aggressive e2-e4
break which we know is White’s overall aim in the Queen’s Indian, there is nothing
fundamentally wrong with this more restrained way of playing. From Black’s point of
view, do not be lulled into thinking there is no harm to be done here. For those not
familiar with the Colle-Zukertort style of play, it is all about the kingside attack. Both
white bishops on b2 and d3 point towards the castled black king, and they might even
be willing to sacrifice themselves on the h7- and g7-squares if the opportunity ever
crops up. Otherwise, they will be adequate backup for a piece attack towards h7 or g7.
After 4 e3 Bb7 5 Bd3 Black has three major options, namely 5 ... Bb4+ (Games 59-
60), 5 ... d5 (Games 61-62) and 5 ... c5 (Game 63). Finally, the rare move 5 Nbd2 is
covered in Game 64.

Game 59
G.Kuzmin-O.Romanishin
USSR Championship, Frunze 1981

1 d4 Nf6 2 c4 e6 3 Nf3 b6 4 e3 Bb7


4 ... Bb4+ 5 Bd2 Be7 6 Bd3 Bb7 was how the game actually started.
5 Bd3 Bb4+


Question: Don’t we normally wait with this move, since
the b1-knight has not committed itself to the c3-square?

Answer: In the 4 e3 variation, White often plays b2-b3 and Bc1-b2, and the b1-knight is actually quite flexible in
that it can go to either of the usual squares on d2 and c3. This 5 ... Bb4+ move however throws a mini spanner in the
works, as 6 Nc3 immediately transposes to the Nimzo-Indian Rubinstein Variation, so if White wants to keep a
Queen’s Indian flavour to the position, he should block on d2.
6 Bd2
6 Nbd2 is examined in the next game.
6 ... Be7
6 ... Bxd2+ 7 Nbxd2 d6 8 0-0 Nbd7 9 Qc2 0-0 10 Rad1 Qe7 was seen in
F.Saemisch-A.Alekhine, Dresden 1926, however I want to recommend something with
a few more winning chances for Black. We have encountered a few examples in this
book already where Black can exchange on the d2-square without too many problems,
but to play for more it is advisable to keep as many pieces on as possible.
7 Nc3


Exercise: White might be going e3-e4 next. Which pawn
break in the centre should Black go for: a) 7 ... c5 or b) 7 ... d5?

Answer: Both are good, since the bishops on d3 and d2 block the White queen’s influence on the d-file. This
means that the d4-d5 push won’t have such a great effect.
7 ... c5
In general, I like ... c5 if White cannot successfully meet this with d4-d5. This is one
reason why forcing the White bishop to the d2-square is a positive for Black. It is clear
this bishop would rather be on c1, where it can go to b2 and influence the centre from
there.
7 ... d5 and going to the centre with the other pawn is quite standard also. Then 8
cxd5 exd5 9 Qa4+ c6 10 Ne5 0-0 11 b4 Bd6 12 f4 b5 13 Qb3 a5 14 bxa5 b4 15 Ne2
Na6 16 Rc1 c5 with interesting play, was seen in J.Hebert-V.Korchnoi, Montreal 2004.
8 dxc5
Other moves for White here are:
8 0-0 0-0 9 Qe2 d6 10 Rad1 Nbd7 11 Bc1 is a bonus for Black, as White’s bishop
has to waste time rerouting itself to the b2-square. Now 11 ... cxd4 12 exd4 d5 was
played in P.Short-E.Rozentalis, Kilkenny 2000 with a nice IQP position to come for
Black.
8 d5 exd5 9 cxd5 Nxd5 10 Nxd5 Bxd5 11 Bc3 Bf6 12 Bxh7 wins the pawn back, but
the h-pawn is not useful as the king on e8 has not committed itself to the kingside yet.
After 12 ... Bxc3+ 13 bxc3 Bb7 14 Bc2 Qf6 Black has threats on c3 and f3. The king on
e8 can be evacuated to the queenside quite easily, however White’s king on e1 has
fewer options. If White castles kingside, there is an open h-file as an autobahn of attack
for Black, and the queenside is hardly something to be considered given the lack of
pawn protection on that side.
8 ... bxc5
8 ... Bxc5 was also fine, but we have seen many times this idea of ... bxc5 giving
greater central control.
9 e4 d6 10 0-0

Question: I never know where to put my b8-knight.
Where does it belong in this position?

Answer: Normally it goes to d7, but here Romanishin understood that since the d4-square has been newly
weakened, moving the knight to c6 would be stronger.
10 ... Nc6! 11 Ne2 0-0 12 Ng3 Nd7
Excellent dark-squared strategy. A future ... Bf6 will control the key squares on d4
and e5.
13 Rb1 Nb4 14 Qb3 a5 15 Be2 Ne5 16 a3
After 16 Nxe5 dxe5 Black’s doubled pawns are not bad, in fact they control useful
squares such as d4 and f4.
16 ... a4 17 Qd1 Nbc6 18 Bc3 Bf6 19 Ne1 Nd4 20 f4
Not 20 Bxd4 when 20 ... cxd4 21 Qxd4? Nf3+ wins the queen.
20 ... Nec6
Black has successfully managed to consolidate his knight on the d4-square, and
stands better.
21 Bd3 g6 22 Nc2 Bg7 23 Ne3 f5!
I like this move, preventing any counterplay with f4-f5 and snuffing any attacking
chances.
24 Bc2 h5
Looks risky, but since Black has such great central control he can afford a move like
this.
25 Rf2 h4 26 Ngf1 Ne7 27 exf5 Nexf5 28 Qg4 Qf6 29 Re1 Qf7 30 Bd3 Rae8 31
Qh3 Bc8 32 Ng4 e5 33 fxe5 dxe5
It looks like Black has taken on voluntary pawn weaknesses, but he only did so
because he noticed the cramped nature of White’s pieces, especially that queen on h3.
There are tactical ideas in the air now via the c8-h3 diagonal.
34 Nd2 Qe7 35 Ref1 Nd6
35 ... Nh6 doesn’t feel right. 36 Rxf8+ Rxf8 37 Nxh6+ Bxh6 38 Rxf8+ Qxf8 39 Qxh4
Be3+ 40 Kh1 and White is fine.
36 Rxf8+ Rxf8 37 Rxf8+ Bxf8
All the tricks now revolve around whether Black can win the g4-knight.
38 Ne4 Nxe4 39 Bxe4 Qg5

40 Bxd4
Alternatively:
a) 40 Bf3 e4 41 Bd1 Qc1 is curtains.
b) It was key for Black to see that 40 Nf6+ did not work. Now 40 ... Qxf6 41 Qxc8
Ne2+ 42 Kh1 Qf1# is a nice finish.
40 ... Qc1+
40 ... cxd4 41 Nf6+ Qxf6 42 Qxc8 is not quite so clear.
41 Kf2 Qd2+ 0-1

Game 60
J.Song-A.Sokolov
French League 2011

1 d4 Nf6 2 c4 e6 3 Nf3 b6 4 e3 Bb7 5 Bd3 Bb4+


The alternatives to this move are 5 ... d5 and 5 ... c5, and these will be examined
later in this chapter.
6 Nbd2
We saw 6 Bd2 in the previous game.
6 ... c5
Again we see that this move tends to work if White cannot answer with d4-d5.

7 0-0
7 a3 takes a tempo to question the bishop immediately. After 7 ... Bxd2+ 8 Bxd2
Black can choose:
a) 8 ... d6 9 dxc5 bxc5 10 0-0 a5 11 b3 Nbd7 12 Qc2 Qc7 13 Bc3 Bxf3 14 gxf3 Ne5
15 Bxe5 dxe5 was played in A.Miles-U.Andersson, Niksic 1983, which was roughly
equal, but certainly not inferior for Black.
b) 8 ... Ne4 looks logical, occupying the e4-square immediately, but after 9 Bxe4
Bxe4 10 Bc3 f6 11 dxc5 bxc5 12 Nd2 Bg6 13 h4 h5 14 Qf3 Nc6 15 Ne4 Qe7 16 Rd1
Black’s position looked a little shaky in G.Kacheishvili-A.Graf, European
Championship, Ohrid 2001.
7 ... cxd4
I quite like this early capture on d4. It poses the early question to White about the
pawn structures that will arise, which will depend on how White recaptures on d4.
8 exd4
8 Nxd4 should normally lead to the Hedgehog structure we have seen before, and not
one Black should ever fear. That d2-knight in particular is not well placed – this knight
much prefers to be on the c3-square, and this is true for the Maroczy structures also.
8 ... 0-0 9 a3 Be7 10 b3


Exercise: Should Black play 10 ... d6 or 10 ... d5?

Answer: I think 10 ... d5 is best as it stops the d4-pawn from advancing and therefore the b2-bishop will be
blocked in. Of course, we ‘give away’ the e5-square, but if a white knight jumps there is can always be challenged
with ... Nc6 or ... Nbd7.
10 ... d5 11 Qe2 Nbd7
11 ... Nc6 12 Bb2 Rc8 13 Rac1 Re8 is also playable, although White can try to
engineer a later queenside push with c4-c5 followed by b4-b5.
12 Bb2 Rc8 13 Rad1 Rc7


Question: What is the idea behind Black’s last move 13 ... Rc7?

Answer: To prepare ... Qa8 which will attack down the long diagonal towards g2, or (as we will see), allow the
rook manoeuvre ... Rc7-d7 and put pressure on the potentially weak d4-pawn.
14 Ne5 dxc4 15 bxc4 Nxe5 16 Qxe5 Rd7 17 Rfe1 g6
This looks to be provoking White to attack along the long diagonal towards g7.
18 d5?

Exercise: What would you play here as Black?

Answer:
18 ... Bxd5!
Black is winning material.
Instead, after 18 ... exd5? 19 cxd5 Bxd5 20 Bb5 White has plenty of play for the
sacrificed pawn, in fact this position is probably already winning for White. After 20 ...
Rc7 21 Nf1 and Nf1-Ne3 cannot be stopped.
19 cxd5 Rxd5 20 Qe2 Rxd3 21 Ne4 Rd5
21 ... Rxd1 22 Rxd1 Nd5 is also adequate.
22 Rxd5 Nxd5 23 Qd2 f6
Blocking off the long diagonal. With care, the two extra pawns should be enough for
Black to win.
24 g4 e5 25 Qh6 Qc8 26 g5 Qg4+ 27 Kf1 Nf4
The counterattack is on!
28 Ng3 Qf3 29 Re3

29 ... Rd8! 0-1

Game 61
L.Portisch-A.Karpov
Novi Sad (rapid) 2006

1 d4 Nf6 2 Nf3 e6 3 c4 b6 4 e3 Bb7 5 Bd3 d5

This is my recommended setup in this system. Black puts the dark-squared bishop on
the active d6-square and keeps his options open for ... c5 or even a future ... e5.
6 b3 Bd6 7 0-0 0-0 8 Bb2 Nbd7 9 Nc3 a6
A useful move, preventing the annoying Nc3-b5, as we saw in Van Trotsenburg-
Euwe in Chapter One.
10 Rc1 Qe7

Quite a standard position, with plenty of tension and therefore winning chances for
both sides. The e4- and e5-squares are often vital here. Both sides attempt to put a
knight on one of those squares and free their f-pawns, as well as gain space. Timing is
everything when carrying out these ideas.
11 Na4
Instead 11 Ne2 dxc4 12 bxc4 Bxf3 13 gxf3 c5 14 f4 cxd4 15 Nxd4 Rac8 16 Qf3 Ba3
17 Bxa3 Qxa3 18 Bb1 Rc7 19 Kh1 Rfc8 was very nice strategic play from Black in
A.Hassan-P.Tregubov, Manama 2009. White’s queenside is feeling the pressure.
The other main plan 11 cxd5 exd5 12 Ne2 is featured in the next game.
11 ... dxc4
11 ... c5 12 Ne5 Rfc8 13 Nxd7 Nxd7 14 cxd5 Bxd5 15 dxc5 bxc5 16 Qe2 Bb7 17
Rfd1 looks pleasant for White, which got better after 17 ... Rc7 18 Bxh7+ Kxh7 19
Qd3+ Kg8 20 Qxd6 and White went on to win in P.Lukacs-L.Lengyel, Budapest 1990.
12 bxc4
12 Bxc4? b5 wins a piece.
12 ... c5 13 Ne5 Rfd8 14 Qc2
Eyeing up the h7-square.

14 ... h6 15 h3 Rac8 16 Qe2 Bc7
Black defends his b6-pawn.
17 Rfd1 cxd4 18 exd4 Nxe5 19 dxe5 Nd7 20 f4
This looks aggressive but could be quite weakening later on. We have already seen
other games in which this f4-e5 pawn chain was attacked.
20 ... Bc6 21 Nc3 Nc5 22 Bb1 Rxd1+ 23 Rxd1 Rd8
Exchanges are fine for Black, who is looking for an endgame edge. Now a couple of
slight inaccuracies from White give Karpov a definite edge.
24 Ba3?! Rxd1+ 25 Qxd1?! Qh4 26 Qd2 Qg3
The pressure is mounting. Now White has a lot of weaknesses, something Karpov is
the master at exploiting.
27 Bxc5
27 Bb2 g5! is similar to the game.
27 ... bxc5 28 Ne4

Exercise: Can you spot Karpov’s combination to gain material?

Answer:
28 ... Bxe4 29 Bxe4 g5!
Excellent play, undermining the central pawn structure. Now the e5-pawn will
certainly drop.
30 fxg5
After 30 Qd7 Qe1+ 31 Kh2 Qxe4 32 Qxc7 Qxf4+ 33 Kh1 Qxc4 Black hoovers up
all of the pawns.
30 ... hxg5 31 Qf2 Qxe5
31 ... Qxf2+ 32 Kxf2 Bxe5 might give White some drawing chances in the opposite
coloured bishop ending.
32 Qe2 Qd4+ 33 Kf1 Bg3 34 Qc2 Qe3 0-1
Karpov’s boa constrictor style play in this game is well worth examining. A top
class player was slowly outplayed.

Game 62
J.Adler-F.Jenni
Swiss League 2003

1 d4 Nf6 2 c4 e6 3 Nf3 b6 4 e3 Bb7 5 Bd3 d5 6 0-0 Nbd7 7 b3 Bd6 8 Bb2 0-0 9 Nc3
a6 10 Rc1 Qe7 11 cxd5
The first deviation from the previous game.
11 ... exd5 12 Ne2

Exercise: What should Black play now?

Answer:
12 ... Ne4
And why not? A great square for the knight, and a platform for a potential kingside
attack beginning with ... f5.
13 Ng3
13 Qc2 Rac8 14 Ng3 f5 15 Qe2 Ra8 16 Rc2 g6 is another way to reach a position
similar to the game.
13 ... f5
Preventing Ng3-f5 and getting Black’s kingside play going.
14 Rc2
White has also tried 14 Re1 g6 15 Re2 c5 16 Rec2 with play similar to the main
game, M.Krasenkow-R.Wojtaszek, Polish League 2009.
14 ... Rf7 15 Qa1 Re8 16 Rfc1 c6
16 ... c5 is the move White’s pieces are waiting for, as then the b2-bishop can join
the game. Still, this is playable for Black.
17 Re1
17 Rxc6 Bxc6 18 Rxc6 looks like an interesting exchange sacrifice to open things
up, though the computer shows the nice line 18 ... Bxg3 19 fxg3 (19 hxg3 Nxf2 wins for
Black) 19 ... Nf2! 20 Bxa6 (20 Kxf2 Qxe3+ 21 Kf1 Qxd3+ wins) 20 ... Nb8 as clearly
better for Black.
17 ... g6 18 Ree2
It appears White has run out of ideas here. He was waiting for ... c5 to open the
position, but since this never came he can only sit back and wait while Black gets on
with the kingside action.
18 ... h5 19 Nf1 h4 20 Ne1
20 h3 g5 and ... g4 will prise open lines towards the king.
20 ... Rh7 21 f3 Ng3! 22 hxg3?
Taking the knight looks suicidal, but the alternatives were not great.
22 Rf2 Nf6 (but not 22 ... Nxf1 23 Bxf1 Qxe3? when 24 Rce2 wins the e8-rook) 23
Qc1 might have offered the most resistance.
22 ... hxg3 23 f4 Qh4 24 Nxg3 Qh2+
24 ... Qxg3 25 Nf3 Nf6 was still strong.
25 Kf2 Nf6 26 Nf1 Ng4+ 27 Kf3 Qg1
There are just too many threats.
28 Rf2
28 ... Qxf1! 29 e4
29 Rxf1 Rxe3#.
29 ... dxe4+ 0-1

Game 63
A.Braun-T.Petrik
Mitropa Cup, Brno 2006

1 d4 Nf6 2 Nf3 e6 3 e3 b6 4 Bd3 Bb7 5 0-0 c5


In my opinion, Black has to defend quite carefully after this natural move.

6 c4 Be7 7 Nc3 cxd4
If Black plays 7 ... 0-0 then White has 8 d5! exd5 9 cxd5 and now:
a) 9 ... Nxd5 10 Nxd5 Bxd5 11 Bxh7+ Kxh7 12 Qxd5 Nc6 13 e4 was J.Gonzalez
Zamora-M.Leon Hoyos, Mexico City 2007, which looks a little better for White, as
Black’s d-pawn cannot get going and his king is weaker.
b) 9 ... d6 10 e4 Nbd7 11 h3 a6 12 a4 Re8 13 Bf4 Bf8 14 Re1 and White had a nice
Benoni style position in A.Yusupov-T.Kaliszewski, Warsaw (rapid) 2007. Observe
Yusupov’s technique here: 14 ... Rc8 15 Bc4 Qc7 16 Qd3 Ra8 17 Rad1 Nh5 18 Bh2 g6
19 e5! Nxe5 20 Bxe5 dxe5 21 d6 Qd7 22 Nxe5 with a completely winning position for
White.
8 exd4

8 ... d5
8 ... d6 looks normal, but again White has the space gaining 9 d5!, and after 9 ... e5
10 Ng5 Nbd7 11 f4 Rc8 12 b3 a6 13 a4 0-0 14 Ra2 h6 15 Raf2!, A.Yusupov-
E.Rozentalis, Elista Olympiad 1998, saw White gain a huge kingside attack. If the g5-
knight gets captured, the attacking plan is instructive: 15 ... hxg5 16 fxg5 Ne8 17 Qh5 g6
18 Bxg6 fxg6 19 Qxg6+ Ng7 20 Rf7 Rxf7 21 Qxf7+ Kh8 22 Rf3 with mate on h3 to
follow.
9 cxd5 Nxd5 10 Ne5

Question: I know bringing the knight to e5 is a good idea, but
why so early? Maybe White should develop the c1-bishop first?

Answer: For now 10 Bg5 is not playable, and White wishes to take advantage of the f6-knight moving by bringing
the queen out to the kingside and commencing attacking operations.
10 ... 0-0 11 Qg4
11 Qh5 Nf6 12 Qh4 will transpose to the game.
11 ... Nf6 12 Qh4 Ne4
12 ... Nc6 is a standard looking move, but gets hit by a thematic move worth
remembering, namely 13 Bg5 g6 14 Ba6!

Analysis Diagram

14 ... h6 15 Bxh6 Nxe5 16 Bxb7 Nfg4 17 Qh3 Qxd4 18 Bxa8 Rxa8 19 Rad1 Qb4 20
Bc1 and White had a clear extra exchange in S.Halkias-J.Polgar, Warsaw (rapid) 2010.
Black’s 14th move alternatives all lose immediately, for example:
a) 14 ... Bxa6 15 Nxc6 Qc7 16 Nxe7+ Qxe7 17 Bxf6.
b) 14 ... Nxe5 15 dxe5 Bxa6 16 exf6 Bd6 17 Qh6.
Returning to the position in the main game after 12 ... Ne4:

13 Qh3
13 Qg4 Nf6 14 Qh4 Ne4 is a draw by repetition, but of course White can play more
aggressively.
13 ... Qxd4 14 Bf4
I think this position is very dangerous for Black. Whilst he is a pawn to the good, he
is faced with a large initiative and it is not clear how to develop the b8-knight.
14 ... Nf6 15 Ne2 Qa4
Trying to get the queen out of the way, but White still retains the attack. After 15 ...
Qd8 16 Rfd1 Qe8 17 Nc3 Nc6 18 Nb5 Rc8 19 Ng4 g6 20 Nd6 was crushing in
A.Stefanova-S.Melia, Women’s Rapid World Championship, Batumi 2012.
16 Bg5
16 Rfc1 Na6 17 Rc4 Qe8 18 Ng4 g6 19 Nh6+ Kg7 20 Be5 Nc5 21 Rh4 (M.Hebden-
G.Morrison, British League 2012) is an example of how a huge attack that can be built
up.
16 ... h6
16 ... g6 was possibly better. Though 17 Rac1 Qa5 18 f4 Qd5 19 Bc4 Qe4 20 Nc3
Qf5 21 Qh4 Bc5+ 22 Kh1 Ne4 23 Bh6 Re8 24 Bd3 Qf6 25 Bg5 Nxg5 26 fxg5 Qxe5 27
Rxf7! Kxf7 28 Qxh7+ Kf8 29 Rf1+ and White won in P.Taylor-N.Donovan, London
2013, which was hardly a glowing reference for 16 ... g6 either!


Exercise: How should White proceed with the attack?

Answer: Sacrifice of course!
17 Bxh6! gxh6?
The computer gives 17 ... Nc6 as the only move, but White’s attack remains.
18 Qxh6 Ne4
Attempting to block the d3-bishop. 18 ... Rd8 19 Bg6! Rf8 (or 19 ... fxg6 20 Qxg6+
Kh8 21 Nf7#) 20 Nf4 fxg6 21 Nfxg6 1-0 was another quick win for White in C.Hoi-
E.Mednis, Copenhagen 1991.
19 Rae1

19 ... Qe8
19 ... Bg5 20 Qh5 continues the attack.
20 Bxe4 Bxe4 21 Nf4 f5
21 ... Bh7 22 Nh5 and mate on g7.
22 Re3 1-0
So the 5 ... c5 move is playable but quite dangerous, and that is why I have mainly
recommended 4 e3 Bb7 5 Bd3 d5 to avoid White having all the fun.

Game 64
K.Jakubowski-B.Macieja
Polish League 2006

1 d4 Nf6 2 Nf3 e6 3 c4 b6 4 e3 Bb7 5 Nbd2


I don’t feel an early knight sortie to d2 can be that good. Of course he knows 5 Nc3
may be met by 5 ... Bb4, but by committing this knight so early, Black can adjust his
development accordingly.
5 ... c5 6 Bd3
6 b3 Be7 7 Bb2 0-0 8 Bd3 cxd4 9 exd4 Nc6 10 0-0 Rc8 11 Rc1 d5 12 a3 dxc4 13
Nxc4 Rc7 was played in N.Kraiouchkine-A.Kovalyov, Canadian Championship 2012,
with a similar strategy of development as in the main game.
6 ... cxd4 7 exd4 d5 8 b3 Bb4 9 0-0 0-0 10 Bb2

10 ... Re8 11 Qc2 Nbd7


After 11 ... Nc6 12 a3 Bf8 13 Rfe1 Rc8 White has 14 c5!? and his idea of advancing
his queenside majority with b3-b4, is a little dangerous. The development of the b8-
knight to d7 puts paid to that idea.
12 Rad1 Rc8 13 Qb1 g6


Question: Why play 13 ... g6, doesn’t it weaken our kingside?

Answer: Yes but only temporarily, as the dark-squared bishop will return to g7 via f8. Anyway, it is not as if White
can utilise any dark square weaknesses any time soon.
14 a3 Bf8 15 Rfe1 Rc7 16 cxd5?!
Releasing the tension and giving Black play against the IQP. White should probably
play something non-committal. If he tries the forcing 16 Ne5 dxc4 17 bxc4 Nxe5 18
dxe5 Ng4 (18 ... Rd7 was simple and strong) then the tactics work in Black’s favour,
for example 19 h3 Nxf2 20 Kxf2 Bc5+ 21 Kf1 Qh4 22 Re2 Qf4+ 23 Ke1 Qg3+ 24 Kf1
Rd7 25 Nb3 Qf4+ 26 Ke1 Be7! and ... Bh4+ and all in all, Black is calling the shots
here.
16 ... Nxd5 17 Ne4 N7f6 18 Rc1 Bh6
19 Rc4 Ba6 20 Rxc7 Bxd3 21 Qxd3 Qxc7
The exchanges have only aided Black, as the weakness of the d4-pawn will be felt in
the endgame. For the IQP to work as an attacking feature, the player with it must retain
as many of the pieces on the board as possible, and certainly the rooks. He has failed to
achieve this here.
22 h3 Nxe4
More exchanges suit Black.
23 Qxe4 Rc8 24 Ne5 Bf4 25 Nd3 Bd2 26 Re2? Qc2
To add to the above comments, now White is tied up too and his minor pieces are in
a jam.

27 Kh2 Nc3 28 Bxc3 Rxc3
Mission complete – White must lose material.
29 Rxd2
29 Qa8+ Kg7 30 Ne5 Bf4+ wins the rook on e2.
29 ... Qxd2 30 Ne5 Qxf2 31 Nf3 Rxb3 0-1
A deceptively simple game which demonstrated excellent Queen’s Indian strategy
throughout.

Game 65
B.Socko-V.Durarbayli
Warsaw 2014

1 d4 Nf6 2 c4 e6 3 Nf3 b6 4 Bf4


4 Bg5 Bb7 5 e3 Be7 6 Nc3 is likely to transpose back into Chapter Eight (Game 52,
Menadue-Wells).

4 ... Bb7 5 e3

Exercise: What would you play here: a) 5 ... Bb4+, b) 5 ... c5 or c) 5 ... Be7?

Answer: they are all playable, but 5 ... c5 is quite risky.
5 ... Be7
I like this simple developing move. Black waits on which pawn move to attack the
centre with, and gets ready to castle. Here is a summary of the alternatives:
a) 5 ... Bb4+ 6 Nbd2 0-0 7 Bd3 d5 8 0-0 Nbd7 9 cxd5 Nxd5 10 Bg3 N5f6 11 a3
Bxd2 12 Nxd2 c5 13 Bd6 Re8 14 dxc5 Nxc5 15 Bxc5 Qxd3 16 Bd4 e5 17 Bc3 Rad8,
as played in R.Derichs-V.Kunin, German League 2009, looked adequate for Black also.
b) 5 ... c5 allows a nice trick with 6 d5! exd5 7 Nc3 and Black must be careful here.
Take a look at these lines which should convince you of the danger involved:
b1) 7 ... dxc4 just looks far too greedy. 8 Nb5 Na6 9 Bxc4 with a further split:
b11) 9 ... Be7 10 Nd6+ Bxd6 11 Qxd6 Ne4 12 Qe5+ Qe7 13 Qxg7 was very nice for
White in P.Karthikeyan-S.Naranayan, Indian League 2015.
b12) 9 ... d5 looks almost suicidal. 10 Qa4! Nd7 11 Ne5 f6 12 Nxd7 dxc4 13 0-0-0
with a crushing position for White in Z.Izoria-A.Naiditsch, European Championship,
Kusadasi 2006.
b2) 7 ... a6 8 cxd5 d6 9 e4 Qe7 10 Be2 Nbd7 11 Nd2 b5 12 0-0 was A.Miles-
V.Hort, Tilburg 1978 and now trying to grab the e4-pawn will bring only danger, for
example 12 ... b4 13 Na4 Nxe4 14 Re1 with excellent compensation and threats down
the e-file.


Exercise: What should Black do after the simple developing move 6 Nc3?

6 h3
Answer: After 6 Nc3, Black should hound the f4-bishop with 6 ... Nh5!.
After 7 Bg3 d6 8 Bd3 Nd7 9 Qc2 g6 10 Rd1 a6 11 Ne2 Nxg3 12 hxg3 Bf6 13 Be4
Black played 13 ... c6! keeping the bishop pair and therefore the tension in the position.
A.Eliseev-Y.Balashov, St Petersburg 2000, continued 14 b3 Qe7 15 g4 Rc8 16 Qb1 d5
17 cxd5 exd5 18 Bc2 c5 19 0-0 0-0 20 Rfe1 Bg7 21 Ng3 Rfe8 which was very
comfortable for Black, who has the two bishops and active pieces.
With 6 h3, White is securing a hole for the bishop on h2 should Black ever play ...
Nh5 and try to exchange this bishop off.
6 ... c5!
As we have seen countless times, if White cannot meet this with d4-d5 then it is
worth playing.
7 Nc3 cxd4 8 exd4
8 Nxd4 is the other way to recapture, but we should not fear this. After 8 ... a6 9 Be2
0-0 10 0-0 d6 11 Bg3 Nbd7 12 Bf3 Qc7 13 Rc1 Rac8 14 b3 Rfd8 15 Qe2 Qb8 16 Rfd1
Nc5, as seen in E.Levin-M.Panchanathan, Pardubice 2011, there was plenty of life in
this Hedgehog position.
8 ... 0-0 9 Be2
9 d5 trying to cut across Black’s plan is risky given that White’s king remains on e1.
9 ... exd5 10 cxd5 Re8 11 Be2 Bb4 and White’s d5-pawn is probably going to hang
here.


Exercise: As always, should Black play 9 ... d6 or 9 ... d5?

Answer: I like the way Black heads for an IQP in this position with 9 ... d5, as moves like 4 Bf4 and 6 h3 are not
really in keeping with the attacking stance White normally has with an IQP.
9 ... d5 10 0-0 Nc6
A much more active square than d7, although that would be OK here too.
11 cxd5 Nxd5 12 Bg3 Rc8
I think that Black already has a very nice position. IQP positions can be quite
dangerous but White has wasted time with h2-h3 and Bf4-Bg3, meaning his initiative is
not as strong compared to the typical attacking IQP lines.
13 Rc1 Nxc3 14 Rxc3 Bf6 15 Qa4 a6 16 Rd1 b5 17 Qb3 Qb6 18 Be5 Nxe5 19
Nxe5 Rxc3 20 Qxc3 Rc8 21 Qb4 Qc7
With the two bishops and a weak pawn to aim for in the centre, Black is a touch
better.
22 a4 Be7 23 Qd2 bxa4 24 Ra1 Qc2 25 Qxc2 Rxc2 26 Bf3 Bxf3 27 Nxf3 Rxb2
28 Rxa4 Rb1+ 29 Kh2 Rb6

Now the advantage has grown to a pawn, and from here it is technique in converting
this to a win. Durarbayli is up to the task.
30 g3 f6 31 Kg2 Kf7 32 Nd2 Ke8 33 Ra2 Bb4 34 Nc4 Rc6 35 Ne3 Kd7 36 Kf3 a5 37 d5 Ra6 38 Ra4 exd5
39 Nxd5 Bc5 40 Nc3 Kc6 41 Rc4 Rb6 42 Na4 Rb5 43 Kg2 g6 44 Rc2 Kd6 45 Rd2+ Ke6 46 Re2+ Kd7 47
Rd2+ Bd6 48 Nc3 Rb6 49 Nd5 Ra6 50 Nxf6+ Kc6 51 Nxh7 a4 52 Nf6 a3 53 Rc2+ Kb5 54 Nd5 Ka4 55 Re2
Kb3 56 Re3+ Kc4 57 Nc3 Rb6 58 Ne4 Bb4 59 Nd2+ Kd5 60 Rd3+ Kc5 61 Rd8 Kb5 62 Nb3 a2 63 Ra8
Ra6 64 Rb8+ Kc4 65 Na1 Kc3 66 Kf3 Ra4 67 Rd8 Kb2 68 Rd1 Ra6 69 Kf4 Re6 70 Kg5 Re1 71 Rd4 Bc3
72 Ra4 Rxa1 73 Kxg6 Rf1 74 f4 Rf3 75 g4 Rxh3 76 f5 Kb3 77 Rxa2 Kxa2 78 g5 Rf3 79 f6 Kb3 80 Kf7 Kc4
81 Ke6 Kb5 82 Ke7 Kc6 83 f7 Bg7 0-1
Chapter Ten
King’s Indian Attack and Réti Lines
1 Nf3 Nf6 2 g3 b6

In this final chapter, we will examine how the Queen’s Indian setup with ... b6 can
be employed against a variety of other lines, including the King’s Indian Attack and
Réti systems. These variations delay or completely omit the d2-d4 pawn move, but both
do involve a kingside fianchetto with White’s light-squared bishop coming to the g2-
square very early on. The main idea of delaying the d2-d4 move is that Black no longer
has the e4-square as a launch pad for either the f6-knight, or the b7-bishop. This is
because the move d2-d3 can always be played. Of course, the drawback is that white is
no longer going for early active operations in the centre, using ideas like d4-d5 which
we have encountered earlier in this book. Essentially, with these systems, White is
going for a slower build-up which can catch the second player unawares if he assumes
White is up to nothing.
Game 66 is an example of a King’s Indian Attack setup with an early d2-d3 for
White, countered by an early ... e6 and a Hedgehog structure for Black. In the remaining
games I cover the double-fianchetto approach, where Black plays both ... b6 and ... g6.
This can often transpose to Chapter Five should White play an early d2-d4 and Black
also plays an early ... e6. Game 67 shows how Black could benefit by saving a tempo
by refraining from ... e6 in this scenario. Finally, Games 68-69 cover the line 1 Nf3 Nf6
2 c4 b6 3 g3 c5 4 Bg2 Bb7 5 0-0 g6 6 Nc3 Bg7 which has been championed at the top
level by Sergey Karjakin.

Game 66
A.Suarez Uriel-J.Moreno Ruiz
Madrid 2010

1 Nf3
1 e4 e6 2 d3 c5 3 Nf3 b6 4 g3 Bb7 5 Bg2 d6 6 0-0 Nf6 7 Re1 Be7 8 Nbd2 0-0 was
how the game actually started, via a French Defence.
1 ... Nf6 2 g3 b6 3 Bg2 Bb7 4 0-0 e6 5 d3 c5
5 ... d5 is of course playable, but I like the Hedgehog structure Black aims for here.
6 e4 d6 7 Re1 Be7 8 Nbd2 0-0 9 c3


Question: Where should I put my b8-knight? In the
Hedgehog I know it meant to go on d7, but is c6 better here?

Answer: They are both playable here, as this Hedgehog is not taking on a Maroczy Bind structure (which does
require the knight on d7 and not c6) hence Black can go slightly more active.
9 ... Nbd7
9 ... Nc6 trying to play more actively is also possible. After 10 a3 Rc8 11 b4 Ne5 12
Nxe5 dxe5 13 Nc4 Qc7 the d3-pawn was a potential target for Black in A.Miles-
M.Nedobora, Seville 1994.
10 a4 Qc7 11 Nc4 a6 12 Bf4 b5!
This is a typical move to try and get in when playing the Hedgehog system. It gains
space and pushes the c4-knight back.
13 Ncd2 e5 14 Bg5 h6 15 Bxf6 Nxf6 16 Nf1

16 ... c4 17 Nh4 Rfd8 18 d4 exd4 19 Qxd4


19 cxd4 was probably to be preferred, since White at least occupies the centre.
After 19 ... Bf8 20 Qd2 d5 21 e5 Ne4 22 Qe3 Qd7 Black is still fine, his e4-knight will
not be trapped as it has an escape route via the g5- and e6-squares. In the meantime,
there is a clear plan to push forward on the queenside with ... b4.
19 ... d5 20 exd5
If 20 e5 Ne4 21 Ne3 Bc5 22 Qd1 Black has 22 ... d4! thematically opening lines for
the two bishops, and after 23 cxd4 Rxd4 Black obtains an excellent position.
20 ... Bc5 21 Qd2 Bxd5
The opening of the position has mainly benefited Black. The c5-bishop is
particularly strong.
22 Qc2 Bxg2 23 Nxg2 Ng4 24 Nge3


Exercise: Should Black exchange knights on e3?

Answer: He could, but the d3-square is a very tasty destination for the black knight. Also, the white knights on e3
and f1 are superfluous – they both want to go to the same square, so best to leave them fighting one another for it!
24 ... Ne5 25 Qe4 Nd3 26 Re2 Re8 27 Qf3 Rad8 28 axb5 axb5 29 Rb1 Re6 30
b3?
An attempt at generating play, which loses at once.
30 ... cxb3 31 Rd2
31 Rxb3 Nc1! 32 Rxb5 Qc6! is the crushing blow. The white rooks are hanging all
over the place.
31 ... Bxe3 32 Nxe3 Qxc3
The bishop was only given up for material gain. The rest is painful for White.
33 Qd1 b2 34 Rdxb2 Qd4 35 Re2 Nf4 36 Re1 Nh3+ 37 Kg2 Qe4+ 38 Qf3 Nf4+ 39 gxf4 Rg6+ 0-1

Game 67
N.Getz-M.Vachier Lagrave
Gibraltar Masters 2012

1 Nf3 Nf6 2 c4 b6 3 g3 Bb7 4 Bg2 g6



As we have seen elsewhere in the book, the surprising fianchetto of the f8-bishop is
often a more ambitious plan than settling for standard development with ... Be7. Here,
the fact that White hasn’t yet played d2-d4 makes a quick d4-d5 thrust less of a threat
than in some other lines we have looked at.
5 0-0 Bg7 6 d4 0-0 7 Nc3 Ne4
7 ... e6 would transpose to lines from Chapter Five, but there is no need to play ... e6
here. Vachier Lagrave sets about gaining the e4-square as a base for a kingside attack.
8 Nxe4 Bxe4 9 Be3 e6 10 Nd2 Bxg2 11 Kxg2 d6
The position looks dull and dry, but I think Black has easier play with a clear plan of
... f5, ... e5 and ... f4 to get an attack going. Whilst White’s setup is very solid, he will
just have to sit and wait for the attack to come.
12 Nf3 Nd7 13 Qd2 Qe7 14 Bh6 Bxh6 15 Qxh6 e5 16 Qd2 Rae8 17 e3

Question: How is Black going to drum up
any play? We don’t want to settle for a draw.

Answer: Play for a kingside attack! ... f5, similar to King’s Indian style play, will form the basis of the attack.
First, we should close the centre to prevent immediate counterplay.
17 ... e4
17 ... f5 18 dxe5 dxe5 19 Rad1 gives White some play down the d-file which could
distract Black from his potential kingside pawn storm.
18 Ng1 f5 19 h4 h6 20 Qe2 Qg7 21 Nh3 Nf6 22 Nf4 g5 23 hxg5 hxg5 24 Nh5 Qg6
25 Nxf6+ Rxf6 26 Rh1 f4
27 Qg4
27 Qh5 Kg7 28 Qxg6+ Kxg6 would have been roughly equal, and safer.
27 ... fxe3 28 fxe3 Ref8
Now there is some danger for White.
29 Raf1 Rf3 30 Qd7 g4
30 ... Qf7 31 Qxf7+ R8xf7 32 Rxf3 Rxf3 33 Rh5 Rxe3 34 Rxg5+ Kf7 is a
complicated rook ending which is likely drawish with accurate play from White. By
keeping the queens on, there are greater attacking chances.
31 d5
31 Qxc7? Qf5! and White’s position suddenly collapses, with 32 ... Rf2+ the chief
threat.
31 ... b5
A surprising shot and the sort of move Grandmasters make that we often don’t
understand. The deep ideas behind this are explained in the following variations.
32 b3?
Alternatively:
a) 32 Qxb5? Qf5 with a mating attack, similar to the variation after White’s 31st
above.
b) 32 cxb5 Qf5 33 Qxf5 R8xf5 and the idea is that the d5-pawn is also weak.
c) 32 Rc1 is the computer’s recommendation (another far from obvious move),
which may enable White to hang on.
32 ... bxc4 33 bxc4
33 ... Rb8!
The infiltration down the b-file is decisive.
34 Rh4
34 Rb1 runs into the spectacular 34 ... Rf2+! 35 Kxf2 Qf6+ with forced mate after 36
Ke2 Qf3+ 37 Kd2 Qf2+ 38 Kc3 Qxe3+ 39 Kc2 Qd3+ 40 Kc1 Rxb1#.
34 ... Rb2+ 35 Kg1 Rxf1+ 0-1
As 36 Kxf1 Qf6+ will be mate very quickly.

Game 68
J.Sofrevski-A.Planinc
Skopje 1971

1 Nf3 Nf6 2 c4 b6 3 g3 c5 4 Bg2 Bb7 5 0-0 g6 6 Nc3 Bg7 7 d3


Here the Botvinnik triangle setup of pawns on c4, d3 and e4 is utilised. It could be
argued though, that the f3-knight is slightly misplaced (it tends to want to be on e2 in the
Botvinnik setup, as f4-f5 can be played quickly).
7 d4 is the subject of the next game.
7 ... 0-0 8 e4

8 ... Nc6

Question: Can’t we put the knight on d7?

Answer: Yes, but the d4-square is an outpost the knight is eyeing up. Of course, there is nothing wrong with the
knight on d7, but it has fewer prospects from there.
In the Hedgehog systems, the knight has a reason to be on d7, namely to defend the
b6-pawn. That is not so important here, so the knight heads to a more active square on
c6.
9 h3 d6 10 Be3 Rc8
An understated move that is designed to slow down the d4 pawn break, as then the
c4-pawn will be hanging at the end of it.
11 Qd2 Nd7 12 b3 Nde5 13 Ne1 Nd4 14 Rb1 f5
I really like Black’s strategy in this game. Deny the opponent the d3-d4 pawn break
and then go for play yourself.
15 f4 Nec6 16 Ne2 Rc7 17 Kh2 e5
The computer only thinks Black is ever so slightly better, but this is a tricky position
for White to play. He doesn’t have a plan and Black is far better coordinated.
18 exf5 Nxf5 19 Bg1 Ncd4 20 Bxb7 Rxb7
The white king is weakened further with this exchange of bishops.
21 Nxd4 cxd4 22 Ng2 Rbf7 23 Bf2 Bh6
White’s position starts to creak at the seams.
24 Qe2 exf4 25 gxf4 d5 26 Rbe1 dxc4 27 bxc4 Qc7 28 Qe4 Bg7 29 Kg1 Qd6 30
Qf3 g5 31 Re4
31 fxg5 Ne3 wins the exchange with ... Nxf1, whilst keeping the initiative.
31 ... gxf4 32 Qxf4 Qa3 33 Qg4 Qxd3 34 Bxd4
An attempt to complicate matters, which is easily rebuffed.
34 ... h5! 35 Qg6 Qxe4 0-1
In my view a classy game from Planinc who demonstrated how to handle Black’s
setup.

Game 69
I.Salgado Lopez-S.Karjakin
European Team Championship, Porto Carras 2011

1 Nf3 Nf6 2 c4 b6 3 g3 c5 4 Bg2 Bb7 5 0-0 g6 6 Nc3 Bg7 7 d4



7 ... cxd4 8 Qxd4
8 Nxd4 is less commonly seen. It is generally considered that by exchanging light-
squared bishops, Black stands at least equal. After 8 ... Bxg2 9 Kxg2 Nc6 10 b3 0-0 11
Bb2 Qc8! the queen heads to the nice b7-square, where it keeps an eye on the white
king as well as the usual d5- and b5-squares. M.Stean-A.Miles, Vienna 1980, continued
with 12 e4 Qb7 13 f3 a6 14 Qd2 Rfd8 15 Rad1 Nxd4 16 Qxd4 b5 which looked OK for
Black.
8 ... d6 9 Rd1 Nbd7 10 Be3 Rc8 11 Rac1 a6 12 b3 0-0 13 Qh4

13 ... Rc7

Question: What is the point of this move?

Answer: It is a multi-purpose move that allows the idea of ... Qa8 and ... Rfc8. This in turn helps Black to get in
one of the ... d5 or ... b5 pawn breaks. Clearly Black will be getting on with queenside play, although White’s ideas of
Be3-h6 and Nf3-g5 also have to be contended with.
14 Bh3 Qb8 15 Bh6 b5!
Well timed. The c3-knight, no longer defended by a pawn on b2, will look a little
shaky.
16 Bxg7
16 cxb5 Bxf3 17 exf3 Ne5 18 Bg2 Rfc8 followed by ... axb5 is better for Black, both
in the pawn structure and relative activity of the pieces.
16 ... Kxg7 17 Nd5


Exercise: Should Black take on d5 with the knight or bishop?

Answer: Black needs to avoid his b7-bishop being locked out of play, as it has little scope along the a8-h1 diagonal
should Black play 17 ... Nxd5 and White answers with 18 cxd5. So 17 ... Bxd5 is the most straightforward solution,
although it turns out that 17 ... Nxd5 is also good if Black follows up correctly (see the next note).
17 ... Bxd5
17 ... Nxd5 18 cxd5 Nf6 19 e4 Bc8! 20 Bg2 Bg4 is a little better for Black.
18 cxd5 Rxc1 19 Rxc1 Rc8 20 Rc6?!
Looks good, but the rook cannot remain here for long.

20 ... Qb7 21 Rxc8
21 Nd4 Nxd5 and Black wins a pawn as well as ousting the rook from c6 in any
case.
21 ... Qxc8
The white pieces look stranded on the kingside as Black’s queen eyes the c-file as an
avenue to start harvesting pawns.
22 Qg5 Qc2 23 Bxd7
23 Qd2 Qb1+ 24 Kg2 Nc5 and the h3-bishop is practically useless here, and the a2-
and d5-pawns are prime targets of attack for Black.
23 ... Qxe2! 24 Bg4 h5
Regaining the piece, and maintaining the pressure.
25 Bxh5 Nxh5 26 Nh4 Nf6 27 Nf5+ Kg8 28 Ne3
28 ... Qxa2
Now all the pawns fall.
29 h4 Qxb3 30 h5 Qb1+ 31 Kh2 Qb2 32 hxg6 Qxf2+ 33 Kh1 Qf3+ 34 Kg1 Ne4 0-1
Index of Complete Games
Adianto.U-De Firmian.N, Biel 1995
Adler.J-Jenni.F, Swiss League 2003
Akesson.R-Speelman.J, European Team Championship, Pula 1997
Andersson.T-Cramling.P, Swedish League 2014
Aronian.L-Agdestein.S, Norway Chess, Stavanger 2014
Bagirov.R-Palac.M, European Team Championship, Batumi 1999
Beliavsky.A-Topalov.V, Madrid 1997
Braun.A-Petrik.T, Mitropa Cup, Brno 2006
Bruno.F-Kovalyov.A, Sort 2008
Brynell.S-Hou Yifan, Wijk aan Zee 2007
Carlsen.M-Karjakin.S, Wijk aan Zee 2012
Gelfand.B-Ivanchuk.V, FIDE Grand Prix, London 2012
Gelfand.B-Kasparov.G, Novgorod 1997
Getz.A-Adams.M, World Open, Philadelphia 2011
Getz.N-Vachier Lagrave.M, Gibraltar Masters 2012
Gheorghiu.F-Hübner.R, Novi Sad Olympiad 1990
Goldin.A-Chuchelov.V, Cappelle la Grande 2001
Gomez Luesia.J-Adla.D, Zaragoza 2003
Gordon.S-Morrison.G, British League 2006
Grabarczyk.M-Yakovich.Y, Bad Wiessee 1999
Gupta.A-Dao Thien Hai, Asian Team Championship, Vishakapatnam 2008
Indjic.A-Yu Yangyi, World Junior Championship, Athens 2012
Istratescu.A-Werle.J, La Roche sur Yon 2006
Jakubowski.K-Macieja.B, Polish League 2006
Kasparov.G-Van der Wiel.J, Amsterdam 1988
Khenkin.I-Adams.M, German League 2002
Kozul.Z-Efimenko.Z, European Club Cup, Kemer 2007
Kuzmin.G-Romanishin.O, USSR Championship, Frunze 1981
Lautier.J-Karpov.A, Amber Rapid, Monaco 2000
Ledger.A-Alekseev.E, European Club Cup, Kallithea 2008
Lobron.E-Karpov.A, Dortmund 1995
Lobron.E-Thorfinnsson.B, Reykjavik 2002
Lputian.S-Tiviakov.S, Kropotkin 1995
Maisuradze.N-L'Ami.E, Maastricht 2014
Markus.R-Postny.E, European Championship, Dresden 2007
Menadue.J-Wells.P, Penarth 2011
Morchiashvili.B-Savchenko.B, European Championship, Plovdiv 2008
Morovic Fernandez.I-Adams.M, Istanbul Olympiad 2000
Oliveira.A-Ibragimov.I, Lisbon 2000
Osterman.R-Grosar.A, Slovenian Championship 1993
Palo.D-Brynell.S, Aarhus 2014
Pettersson.A-Brynell.S, Rilton Cup, Stockholm 2010
Piket.J-Anand.V, Cologne 1995
Ponomariov.R-Kramnik.V, Sofia 2005
Portisch.L-Karpov.A, Novi Sad (rapid) 2006
Psakhis.L-Lobron.E, Baden-Baden 1992
Radjabov.T-Adams.M, Prague (rapid) 2002
Radjabov.T-Gelfand.B, Odessa (rapid) 2007
Roiz.M-Savchenko.B, European Championship, Plovdiv 2008
Salgado Lopez.I-Karjakin.S, European Team Championship, Porto Carras 2011
Sargissian.G-Odeev.H, Dubai 2006
Sharavdorj.D-Shomoev.A, Ulaanbaatar 2011
Short.P-Brady.S, Irish Championship 2009
Socko.B-Durarbayli.V, Warsaw 2014
Sofrevski.J-Planinc.A, Skopje 1971
Sokolov.I-Stohl.I, Elenite 1992
Song.J-Sokolov.A, French League 2011
Suarez Uriel.A-Moreno Ruiz.J, Madrid 2010
Svidler.P-Ipatov.A, Blitz World Championship, Dubai 2014
Tallaksen.G-Carlsen.M, Gausdal 2004
Urkedal.F-Agdestein.S, Norwegian League 2014
Van Trotsenburg.B-Euwe.M, Amsterdam 1920
Vernay.C-Postny.E, French League 2010
Vovk.Y-Melkumyan.H, Lviv 2007
Vragoteris.A-Sokolov.A, Greek League 2003
Wang Yue-Bacrot.E, Nanjing Pearl Spring 2010
Wang Yue-Leko.P, FIDE Grand Prix, Elista 2008
Yakovich.Y-Jobava.B, Russian League 2007
Yusupov.A-Manolache.M, European Championship, Dresden 2007

You might also like